Psych Exam 1: Ch. 1-11, 20, & 25

Réussis tes devoirs et examens dès maintenant avec Quizwiz!

A patient says, "I've done a lot of cheating and manipulating in my relationships." Select a nonjudgmental response by the nurse. A. "How do you feel about that?" B. "It's good that you realize this." C. "That's not a good way to behave." D. "Have you outgrown that type of behavior?"

A. "How do you feel about that?" Rationale: Asking a patient to reflect on feelings about his or her actions does not imply any judgment about those actions, and it encourages the patient to explore feelings and values. The remaining options offer negative judgments.

An individual says to the nurse, "I feel so stressed out lately. I think the stress is affecting my body also." Which somatic complaints are most likely to accompany this feeling? Select all that apply. A. Headache B. Neck pain C. Insomnia D. Anorexia E. Myopia

A. Headache B. Neck pain C. Insomnia D. Anorexia Rationale: When individuals feel "stressed-out," they often have accompanying somatic complaints, especially associated with sleep, eating, and headache or back pain. Changes in vision, such as myopia, would not be expected.

The assumption most useful to a nurse planning crisis intervention for any patient is that the patient: A. Is experiencing a state of disequilibrium B. Is experiencing a type of mental illness C. Poses a threat of violence to others D. Has a high potential for self-injury

A. Is experiencing a state of disequilibrium Rationale: Disequilibrium is the only answer universally true for all patients in crisis. A crisis represents a struggle for equilibrium when problems seem unsolvable. Crisis does not reflect mental illness. The potential for self-violence or other-directed violence may or may not be a factor in crisis.

As part of the stress response, the HPA axis is stimulated. Which structures make up this system? A. Hippocampus, parietal lobe, and amygdala B. Hypothalamus, pituitary gland, and adrenal glands C. Hind brain, pyramidal nervous system, and anterior cerebrum D. Hepatic artery, parasympathetic nervous system, and acoustic nerve

B. Hypothalamus, pituitary gland, and adrenal glands Rationale: As part of the physiologic response of stress, the hypothalamus stimulates the HPA axis, which is made up of the hypothalamus, pituitary gland, antd adrenal glands.

A nurse uses Maslow's hierarchy of needs to plan care for a psychotic patient. Which problem will receive priority? The patient: A. Refuses to eat or bathe B. Reports feelings of alienation from family C. Is reluctant to participate in unit social activities D. Need to be taught about medication action and side effects

A. Refuses to eat or bathe Rationale: The need for food and hygiene is physiological and therefore takes priority over psychological or meta-needs in care planning.

Which patient statements identify qualities of nursing practice with high therapeutic value? (Select all that apply.) "The nurse: A. Talks in language I can understand." B. Helps me keep track of my medications." C. Is willing to go to social activities with me." D. Lets me do whatever I choose without interfering." E. Looks at me as a whole person with different needs."

A. Talks in language I can understand." B. Helps me keep track of my medications." E. Looks at me as a whole person with different needs." Rationale: Each correct answer demonstrates caring is an example of appropriate nursing foci: communicating at a level understandable to the patient, using holistic principles to guide care, and providing medication supervision. The incorrect options suggest a laissez-faire attitude on the part of the nurse, when the nurse should instead provide thoughtful feedback and help patients test alternative solutions or violate boundaries.

A person who has been unable to leave home for more than a week because of severe anxiety says, "I know it does not make sense, but I just can't bring myself to leave my apartment alone." Which nursing intervention is appropriate? A. Teach the person to use positive self-talk B. Assist the person to apply for disability benefits C. Ask the person to explain why the fear is so disabling D. Advise the person to accept the situation and use a companion

A. Teach the person to use positive self-talk Rationale: This intervention, a form of cognitive restructuring, replaces negative thoughts such as "I can't leave my apartment" with positive thoughts such as "I can control my anxiety." This technique helps the patient gain mastery over the symptoms. The other options reinforce the sick role.

Several nurses are concerned that agency policies related to restraint and seclusion practices are inadequate. Which statement about the relationship of substandard institutional policies and individual nursing practice should guide nursing practice? A. The policies do not absolve an individual nurse of the responsibility to practice according to the professional standards of nursing care B. Agency policies are the legal standard by which a professional nurse must act and therefore override other standards of care C. In an institution with substandard policies, the nurse has a responsibility to inform the supervisor and leave the premises D. Interpretation of policies by the judicial system is rendered on an individual basis and therefore cannot be predicted

A. The policies do not absolve an individual nurse of the responsibility to practice according to the professional standards of nursing care Rationale: Nurses are professionally bound to uphold the American Nurses Association (ANA) standards of practice, regardless of lesser standards established by a health care agency or state. Conversely, if the agency standards are higher than the ANA standards of practice, the agency standards must be upheld. The courts may seek to establish the standard of care through the use of expert witnesses when the issue is clouded.

A child is placed in a foster home after being removed from parental contact because of abuse. The child is apprehensive and overreacts to environmental stimuli. The foster parents ask the nurse how to help the child. What should the nurse recommend? Select all that apply. A. Use a calm manner and low voice B. Maintain simplicity in the environment C. Avoid repetition in what is said to the child D. Minimize opportunities for exercise and play E. Explain and reinforce reality to avoid distortions

A. Use a calm manner and low voice B. Maintain simplicity in the environment E. Explain and reinforce reality to avoid distortions Rationale: The child can be hypothesized to have moderate-to-severe trait (chronic) anxiety. A calm manner calms the child. A simple, structured, predictable environment is less anxiety provoking and reduces overreaction to stimuli. Calm, simple explanations that reinforce reality validate the environment. Repetition is often needed when the child is unable to concentrate because of elevated levels of anxiety. Opportunities for play and exercise should be provided as avenues to reduce anxiety. Physical movement helps channel and lower anxiety. Play also helps by allowing the child to act out concerns.

Which assessment questions are most appropriate to ask a patient with possible obsessive-compulsive disorder? Select all that apply. A. "Have you been a victim of a crime or seen someone badly injured or killed?" B. "Are there certain social situations that cause you to feel especially uncomfortable?" C. "Do you have to do things in a certain way to feel comfortable?" D. "Is it difficult to keep certain thoughts out of awareness?" E. "Do you do certain things over and over again?"

C. "Do you have to do things in a certain way to feel comfortable?" D. "Is it difficult to keep certain thoughts out of awareness?" E. "Do you do certain things over and over again?" Rationale: The correct questions refer to obsessive thinking and compulsive behaviors. The incorrect responses are more pertinent to a patient with suspected posttraumatic stress disorder or with suspected social phobia.

If a cruel and abusive person rationalizes this behavior, which comment is most characteristic of this person? A. "I don't know why it happens." B. "I have poor impulse control." C. "That person shouldn't have provoked me." D. "I'm really a coward who is afraid of being hurt."

C. "That person shouldn't have provoked me." Rationale: Rationalization consists of justifying one's unacceptable behavior by developing explanations that satisfy the teller and attempt to satisfy the listener. The abuser is suggesting that the abuse is not his or her fault; it would not have occurred except for the provocation by the other person.

When a nurse assesses an older adult patient, answers seem vague or unrelated to the questions. The patient also leans forward and frowns, listening intently to the nurse. An appropriate question for the nurse to ask would be: A. "Are you having difficulty hearing when I speak?" B. "How can I make this assessment interview easier for you?" C. "I notice you are frowning. Are you feeling annoyed with me?" D. "You're having trouble focusing on what I'm saying. What is distracting you?"

A. "Are you having difficulty hearing when I speak?" Rationale: The patient's behaviors may indicate difficulty hearing. Identifying any physical need the patient may have at the onset of the interview and making accommodations are important considerations. By asking if the patient is annoyed, the nurse is jumping to conclusions. Asking how to make the interview easier for the patient may not elicit a concrete answer. Asking about distractions is a way of asking about auditory hallucinations, which is not appropriate because the nurse has observed that the patient seems to be listening intently.

Which statement is mostly likely to be made by a patient with agoraphobia? A. "Being afraid to go out seems ridiculous, but I can't go out the door." B. "I'm sure I'll get over not wanting to leave home soon. It takes time." C. "When I have a good incentive to go out, I can do it." D. "My family says they like it now that I stay home."

A. "Being afraid to go out seems ridiculous, but I can't go out the door." Rationale: Individuals who are agoraphobic generally acknowledge that the behavior is not constructive and that they do not really like it. Patients state they are unable to change the behavior. Patients with agoraphobia are not optimistic about change. Most families are dissatisfied when family members refuse to leave the house.

A nurse talks with the caregiver of a combat veteran with severe traumatic brain injuries. The caregiver says, "I don't know how much longer I can do it. My whole life is consumed with taking care of my partner." Select the nurse's best response. A. "How are you taking care of yourself?" B. "Let's review your partner's diagnostic results." C. "I have some web-based programs for you to visit." D. "Your partner is lucky to have someone so devoted."

A. "How are you taking care of yourself?" Rationale: The caregiver is the focus of the nurse's attention. The caregiver is suffering. The nurse must be empathetic and assess how the caregiver is caring for self. Reassurance and isolated computer activities do not help. The partner is already aware of the diagnostic results.

Which comment by a person who is experiencing severe anxiety indicates the possibility of obsessive-compulsive disorder? A. "I check where my car keys are eight times." B. "My legs often feel weak and spastic." C. "I'm embarrassed to go out in public." D. "I keep reliving the car accident."

A. "I check where my car keys are eight times." Rationale: Recurring doubt (obsessive thinking) and the need to check (compulsive behavior) suggest obsessive-compulsive disorder. The repetitive behavior is designed to decrease anxiety but fails and must be repeated. The statement, "My legs feel weak most of the time," is more in keeping with a somatoform disorder. Being embarrassed to go out in public is associated with an avoidant personality disorder. Reliving a traumatic event is associated with posttraumatic stress disorder.

Which statements by a patient who is terminally ill give the nurse information relevant to spiritual assessment? Select all that apply. A. "I feel an inner peace with my decision to use hospice services." B. "I trust my health care provider to prescribe enough medication to keep me free of pain." C. "I have prepared advance directives to spare my children the need to make difficult decisions." D. "I plan to use these last weeks to experience the process of dying as fully as I experienced the richness of living." E. "Listening to hymns helps deepen my relaxation and the relief I get from my pain medication."

A. "I feel an inner peace with my decision to use hospice services." D. "I plan to use these last weeks to experience the process of dying as fully as I experienced the richness of living." E. "Listening to hymns helps deepen my relaxation and the relief I get from my pain medication." Rationale: Spirituality encompasses finding meaning in the process of living and dying, as well as hope and inner peace. Listening to hymns identifies an activity that connects the patient to his or her beliefs and is helpful in calming anxieties. The other options are not directly related to spiritual aspects.

After being notified that her husband died of heart failure, a wife approaches the nurse who cared for her husband and in the hospital hallway shouts angrily, "He'd still be alive if you'd given him your undivided attention!" Select the nurse's best response. A. "I understand you're feeling upset. Let's go to our conference room, and I'll stay with you until your family comes." B. "Your husband's heart was severely damaged and could no longer pump. There's nothing anyone could have done." C. "I will call the nursing supervisor to discuss this matter with you." D. "It will be all right if you cry. Crying is a normal grief response."

A. "I understand you're feeling upset. Let's go to our conference room, and I'll stay with you until your family comes." Rationale: When a bereaved family member behaves in a disturbed manner, the nurse should show patience and tact while offering sympathy and warmth. Moving the individual to a private area so as not to disturb others is important. The incorrect options are defensive, evasive, or placating.

A patient says, "Please don't share information about me with the other people." How should the nurse respond? A. "I won't share information with your family or friends without your permission, but I will share information about you with other staff members." B. "A therapeutic relationship is just between the nurse and the patient. It's up to you to tell others what you want them to know." C. "It really depends on what you choose to tell me. I will be glad to disclose at the end of each session what I will report to others." D. "I cannot tell anyone about you. It will be as though I am talking about my own problems, and we can help each other by keeping it between us."

A. "I won't share information with your family or friends without your permission, but I will share information about you with other staff members." Rationale: A patient has the right to know with whom the nurse will share information and that confidentiality will be protected. Although the relationship is primarily between the nurse and patient, other staff members need to know pertinent data. The other incorrect responses promote incomplete disclosure on the part of the patient, require daily renegotiation of an issue that should be resolved as the nurse-patient contract is established, and suggest mutual problem solving. The relationship must be patient centered.

A community member asks a nurse, "People with mental illnesses used to go to a state hospital. Why has that changed?" Select the nurse's accurate responses. Select all that apply. A. "Science has made significant improvements in drugs for mental illness, so now many people may live in their communities." B. "A better selection of less restrictive settings is now available in communities to care for individuals with mental illness." C. "National rates of mental illness have declined significantly. The need for state institutions is actually no longer present." D. "Most psychiatric institutions were closed because of serious violations of patients' rights and unsafe conditions." E. "Federal legislation and payment for treatment of mental illness have shifted the focus to community rather than institutional settings."

A. "Science has made significant improvements in drugs for mental illness, so now many people may live in their communities." B. "A better selection of less restrictive settings is now available in communities to care for individuals with mental illness." E. "Federal legislation and payment for treatment of mental illness have shifted the focus to community rather than institutional settings." Rationale: The community is a less restrictive alternative than hospitals for the treatment of people with mental illness. Funding for treatment of mental illness remains largely inadequate but now focuses on community rather than institutional care. Antipsychotic medications improve more symptoms of mental illness; hence, management of psychiatric disorders has improved. Rates of mental illness have increased, not decreased. Hospitals were closed because funding shifted to the community. Conditions in institutions have improved.

A nurse explains to the family of a patient who is mentally ill how the nurse-patient relationship differs from social relationships. Which is the best explanation? A. "The focus is on the patient. Problems are discussed by the nurse and patient, but solutions are implemented by the patient." B. "The focus shifts from nurse to patient as the relationship develops. Advice is given by both, and solutions are implemented." C. "The focus of the relationship is socialization. Mutual needs are met, and feelings are openly shared." D. "The focus is the creation of a partnership in which each member is concerned with the growth and satisfaction of the other."

A. "The focus is on the patient. Problems are discussed by the nurse and patient, but solutions are implemented by the patient." Rationale: Only the correct response describes the elements of a therapeutic relationship. The remaining responses describe events that occur in social or intimate relationships.

A nurse performing an assessment interview for a patient with a substance use disorder decides to use a standardized rating scale. Which scales are appropriate? Select all that apply. A. Addiction Severity Index (ASI) B. Brief Drug Abuse Screen Test (B-DAST) C. Abnormal Involuntary Movement Scale (AIMS) D. Cognitive Capacity Screening Examination (CCSE) E. Recovery Attitude and Treatment Evaluator (RAATE)

A. Addiction Severity Index (ASI) B. Brief Drug Abuse Screen Test (B-DAST) E. Recovery Attitude and Treatment Evaluator (RAATE) Rationale: Standardized scales are useful for obtaining data concerning substance use disorders. The ASI, B-DAST, and RAATE are scales related to substance abuse. The AIMS assesses involuntary movements associated with antipsychotic medications. The CCSE assesses cognitive function.

The family of a patient whose insurance will not pay for continuing hospitalization considers transferring the patient to a public psychiatric hospital. The family expresses concern that the patient will "never get any treatment." Which reply by the nurse would be most helpful? A. "Under the law, treatment must be provided. Hospitalization without treatment violates patients' rights." B. "That's a justifiable concern because the right to treatment extends only to the provision of food, shelter, and safety." C. "Much will depend on other patients because the right to treatment for a psychotic patient takes precedence over the right to treatment of a patient who is stable." D. "All patients in public hospitals have the right to choose both a primary therapist and a primary nurse."

A. "Under the law, treatment must be provided. Hospitalization without treatment violates patients' rights." Rationale: The right to medical and psychiatric treatment was conferred on all patients hospitalized in public mental hospitals with the enactment of the federal Hospitalization of Mentally Ill Act in 1964. Stating that the concern is justifiable supports the family's erroneous belief. The provisions mentioned in the third and fourth options are not part of this or any other statute governing psychiatric care.

A terminally ill patient says, "I know I'm not going to get well, but still..." and the patient's voice trails off. Which response by the nurse would be therapeutic? A. "What do you hope for?" B. "No, you're not going to get well." C. "Do you have questions about what is happening?" D. "I'm happy you are being realistic about your future."

A. "What do you hope for?" Rationale: This open-ended response is an example of following the patient's lead. It provides an opportunity for the patient to speak about whatever is on his or her mind. The incorrect options are not therapeutic; they block further communication, refocus the conversation, give advice, or suggest the nurse is uncomfortable with the topic.

Which statement shows a nurse has empathy for a patient who made a suicide attempt? A. "You must have been very upset when you tried to hurt yourself." B. "It makes me sad to see you going through such a difficult experience." C. "If you tell me what is troubling you, I can help you solve your problems." D. "Suicide is a drastic solution to a problem that may not be such a serious matter."

A. "You must have been very upset when you tried to hurt yourself." Rationale: Empathy permits the nurse to see an event from the patient's perspective, understand the patient's feelings, and communicate this to the patient. The incorrect responses are nurse centered (focusing on the nurse's feelings rather than the patient's), belittling, and sympathetic.

A patient has a fear of public speaking. The nurse should be aware that social phobias are often treated with which type of medication? A. (Beta)-blockers B. Antipsychotic medications C. Tricyclic antidepressant agents D. Monoamine oxidase inhibitors

A. (Beta)-blockers Rationale: Beta-blockers, such as propranolol, are often effective in preventing symptoms of anxiety associated with social phobias. Neuroleptic medications are major tranquilizers and not useful in treating social phobias. Tricyclic antidepressants are rarely used because of their side effect profile. MAOIs are administered for depression and only by individuals who can observe the special diet required.

A nurse assesses soldiers in a combat zone in Afghanistan. When is it most important for the nurse to screen for signs and symptoms of traumatic brain injury (TBI)? A. After a fall, vehicle crash, or exposure to a blast B. Before departing Afghanistan to return to the United States C. One year after returning to the United States from Afghanistan D. Immediately upon return to the United States from Afghanistan

A. After a fall, vehicle crash, or exposure to a blast Rationale: The military estimates that up to 20% of the combat veterans in both Afghanistan and Iraq suffer some degree of TBI. TBI exhibits signs shortly after the injury, and these signs usually resolve in days or weeks. Screening after an exposure to an explosion and before returning to the United States is important.

Which situations qualify as abandonment on the part of a nurse? (Select all that apply.) The nurse: A. Allows a patient with acute mania to refuse hospitalization without taking further action B. Terminates employment without referring a seriously mentally ill for aftercare C. Calls police to bring a suicidal patient to the hospital after a suicide attempt D. Refers a patient with persistent paranoid schizophrenia to community treatment E. Asks another nurse to provide a patient's care because of concerns about countertransference

A. Allows a patient with acute mania to refuse hospitalization without taking further action B. Terminates employment without referring a seriously mentally ill for aftercare Rationale: Abandonment arises when a nurse does not place a patient safely in the hands of another health professional before discontinuing treatment. Calling the police to bring a suicidal patient to the hospital after a suicide attempt and referring a patient with schizophrenia to community treatment both provide for patient safety. Asking another nurse to provide a patient's care because of concerns about countertransference demonstrates self-awareness.

Two staff nurses applied for promotion to nurse manager. The nurse not promoted initially had feelings of loss but then became supportive of the new manager by helping make the transition smooth and encouraging others. Which term best describes the nurse's response? A. Altruism B. Sublimation C. Suppression D. Passive aggression

A. Altruism Rationale: Altruism is the mechanism by which an individual deals with emotional conflict by meeting the needs of others and vicariously receiving gratification from the responses of others. The nurse's reaction is conscious, not unconscious. No evidence of aggression is exhibited, and no evidence of conscious denial of the situation exists. Passive aggression occurs when an individual deals with emotional conflict by indirectly and unassertively expressing aggression toward others.

A patient being treated in an alcohol rehabilitation unit reveals to the nurse, "I feel terrible guilt for sexually abusing my 6-year-old child before I was admitted." Based on state and federal law, the best action for the nurse to take is to: A. Anonymously report the abuse by telephone to the local child abuse hotline B. Reply, "I'm glad you feel comfortable talking to me about it." C. Respect nurse-patient relationship of confidentiality D. File a written report on the agency letterhead

A. Anonymously report the abuse by telephone to the local child abuse hotline Rationale: Laws regarding reporting child abuse discovered by a professional during a suspected abuser's alcohol or drug treatment differ by state. Federal law supersedes state law and prohibits disclosure without a court order except in instances in which the report can be made anonymously or without identifying the abuser as a patient in an alcohol or drug treatment facility. Anonymously reporting the abuse by telephone to the local child abuse hotline meets federal criteria. Respecting nurse-patient confidentiality and replying, "I'm glad you feel comfortable talking to me about it" do not accomplish reporting. Filing a written report on agency letterhead violates federal law.

A community mental health nurse has worked with a patient for 3 years but is moving out of the city and terminates the relationship. A new nurse who begins work with this patient will: A. Begin at the orientation phase B. Resume the working relationship C. Enter into a social relationship D. Return to the emotional catharsis phase

A. Begin at the orientation phase Rationale: After the termination of a long-term relationship, the patient and new nurse usually have to begin at ground zero, the orientation phase, to build a new relationship. If termination is successfully completed, then the orientation phase sometimes progresses quickly to the working phase. Other times, even after successful termination, the orientation phase may be prolonged.

A nurse says, "I'm the only one who truly understands this patient. Other staff members are too critical." The nurse's statement indicates: A. Boundary blurring B. Sexual harassment C. Positive regard D. Advocacy

A. Boundary blurring Rationale: When the role of the nurse and the role of the patient shift, boundary blurring may arise. In this situation, the nurse is becoming overinvolved with the patient as a probable result of unrecognized countertransference. When boundary issues occur, the need for supervision exists. The situation does not describe sexual harassment. Data are not present to suggest positive regard or advocacy.

Which action by a psychiatric nurse best supports patients' rights to be treated with dignity and respect? A. Consistently addressing each patient by title and surname B. Strongly encouraging a patient to participate in the unit milieu C. Discussing a patient's condition with another health care provider in the elevator D. Informing a treatment team that a patient is too drowsy to participate in care planning

A. Consistently addressing each patient by title and surname Rationale: A simple way of showing respect is to address the patient by title and surname rather than assuming that the patient would wish to be called by his or her first name. Discussing a patient's condition with a health care provider in the elevator violates confidentiality. Informing a treatment team that the patient is too drowsy to participate in care planning violates patient autonomy. Encouraging a patient to participate in the unit milieu exemplifies beneficence and fidelity.

Which benefits are most associated with the use of telehealth? Select all that apply. A. Cost savings for patients B. Maximization of care management C. Access to services for patients in rural areas D. Prompt reimbursement by third-party payers E. Rapid development of trusting relationships with patients

A. Cost savings for patients B. Maximization of care management C. Access to services for patients in rural areas Rationale: Use of telehealth technologies has shown that it can maximize health and improve disease management skills and confidence with the disease process. Many rural patients have felt disconnected from services; telehealth technologies can solve these problems. Although telehealth's improved health outcomes regularly show cost savings for payers, one significant barrier is the current lack of reimbursement for remote patient monitoring by third-party payers. Telehealth is not associated with rapid development of trusting relationships.

A Mexican-American patient puts a picture of the Virgin Mary on the bedside table. Under which section of the assessment should the nurse document this behavior? A. Culture B. Ethnicity C. Verbal communication D. Nonverbal communication

A. Culture Rationale: Cultural heritage is expressed through language, works of art, music, dance, ethnic clothing, customs, traditions, diet, and expressions of spirituality. This patient's prominent placement of the picture is an example of expression of cultural heritage.

A professor's 4-year-old child has a body temperature of 101.6° F, diarrhea, and complains of stomach pain. The professor is scheduled to teach three classes today. Which nursing diagnosis best applies to this scenario? A. Decisional conflict B. Unilateral neglect C. Disabled family coping D. Ineffective management of the therapeutic regimen

A. Decisional conflict Rationale: The caregiver is the focus of the nurse's attention. The professor is under stress, related to the conflict between his parenting and professional roles. This scenario presents a decisional conflict. No evidence suggests that family coping is disabled or that a therapeutic regimen is not managed. Unilateral neglect refers to the awareness of the body.

A nurse driving home after work comes upon a serious automobile accident. The driver gets out of the car with no apparent physical injuries. Which assessment findings would be expected from the driver immediately after this event? Select all that apply. A. Difficulty using a cell phone B. Long-term memory losses C. Fecal incontinence D. Rapid speech E. Trembling

A. Difficulty using a cell phone D. Rapid speech E. Trembling Rationale: Immediate responses to crisis commonly include shock, numbness, denial, confusion, disorganization, difficulty with decision making, and physical symptoms such as nausea, vomiting, tremors, profuse sweating, and dizziness associated with anxiety. Incontinence and long-term memory losses would not be expected.

A nurse assesses an older adult patient brought to the emergency department by a family member. The patient was wandering outside, saying, "I can't find my way home." The patient is confused and unable to answer questions. Select the nurse's best action. A. Document the patient's mental status. Obtain other assessment data from the family member B. Record the patient's answers to questions on the nursing assessment form C. Ask an advanced practice nurse to perform the assessment interview D. Call for a mental health advocate to maintain the patient's rights

A. Document the patient's mental status. Obtain other assessment data from the family member Rationale: When the patient (primary source) is unable to provide information, secondary sources should be used, in this case the family member. Later, more data may be obtained from other relatives or neighbors who are familiar with the patient. An advanced practice nurse is not needed for this assessment; it is within the scope of practice of the staff nurse. Calling a mental health advocate is unnecessary.

Which employer's health plan is required to include parity provisions related to mental illnesses? A. Employer with more than 50 employees B. Cancer thrift shop staffed by volunteers C. Daycare center that employs 7 teachers D. Church that employs 15 people

A. Employer with more than 50 employees Rationale: Under federal parity laws, companies with more than 50 employees may not limit annual or lifetime mental health benefits unless they also limit benefits for physical illnesses.

A student says, "Before taking a test, I feel a heightened sense of awareness and restlessness." The nursing intervention most suitable for assisting the student is to: A. Explain that the symptoms are the result of mild anxiety, and discuss the helpful aspects B. Advise the student to discuss this experience with a health care provider C. Encourage the student to begin antioxidant vitamin supplements D. Listen without comment

A. Explain that the symptoms are the results of mild anxiety, and discuss the helpful aspects Rationale: Teaching about the symptoms of anxiety, their relation to precipitating stressors, and, in this case, the positive effects of anxiety serves to reassure the patient.

A nurse ends a relationship with a patient. Which actions by the nurse should be included in the termination phase? Select all that apply. A. Focus dialogs with the patient on problems that may occur in the future B. Help the patient express feelings about the relationship with the nurse C. Help the patient prioritize and modify socially unacceptable behaviors D. Reinforce expectations regarding the parameters of the relationship E. Help the patient identify strengths, limitations, and problems

A. Focus dialogs with the patient on problems that may occur in the future B. Help the patient express feelings about the relationship with the nurse Rationale: The correct actions are part of the termination phase. The other actions are used in the working and orientation phases.

Which descriptors exemplify consistency regarding nurse-patient relationships? Select all that apply. A. Having the same nurse care for a patient on a daily basis B. Encouraging a patient to share initial impressions of staff C. Providing a schedule of daily activities to a patient D. Setting a time for regular sessions with a patient E. Offering solutions to a patient's problems

A. Having the same nurse care for a patient on a daily basis C. Providing a schedule of daily activities to a patient D. Setting a time for regular sessions with a patient Rationale: Consistency implies predictability. Having the same nurse see the patient daily, providing a daily schedule of patient activities, and setting a regular time for sessions help a patient to predict what will happen during each day and to develop a greater degree of security and comfort. Encouraging a patient to share initial impressions of staff and giving advice are not related to consistency and would not be considered a therapeutic intervention.

A nurse can best address factors of critical importance to successful community treatment for persons with mental illness by including assessments related to which of the following? Select all that apply. A. Housing adequacy and stability B. Income adequacy and stability C. Family and other support systems D. Early psychosocial development E. Substance abuse history and current use

A. Housing adequacy and stability B. Income adequacy and stability C. Family and other support systems E. Substance abuse history and current use Rationale: Early psychosocial developmental history is less relevant to successful outcomes in the community than the assessments listed in the other options. If a patient is homeless or fears homelessness, focusing on other treatment issues is impossible. Sufficient income for basic needs and medication is necessary. Adequate support is a requisite to community placement. Substance abuse undermines medication effectiveness and interferes with community adjustment.

A patient with major depression has lost 20 pounds in one month has chronic low self-esteem and a plan for suicide. The patient has taken an antidepressant medication for 1 week. Which nursing intervention is most directly related to this outcome: "Patient will refrain from gestures and attempts to harm self"? A. Implement suicide precautions B. Frequently offer high-calorie snacks and fluids C. Assist the patient to identify three personal strengths D. Observe patient for therapeutic effects of antidepressant medication

A. Implement suicide precautions Rationale: Implementing suicide precautions is the only option related to patient safety. The other options, related to nutrition, self-esteem, and medication therapy, are important but are not priorities.

At what point in the nurse-patient relationship should a nurse plan to first address termination? A. In the orientation phase B. In the working phase C. In the termination phase D. When the patient initially brings up the topic

A. In the orientation phase Rationale: The patient has a right to know the conditions of the nurse-patient relationship. If the relationship is to be time limited, then the patient should be informed of the number of sessions. If it is open ended, then the termination date will not be known at the outset and the patient will know that the issue will be negotiated at a later date. The nurse is responsible for bringing up the topic of termination early in the relationship, usually during the orientation phase.

A patient in the emergency department exhibits disorganized behavior and incoherence after a friend suggested a homosexual encounter. In which room should the nurse place the patient? A. Interview room furnished with a desk and two chairs B. Small, empty storage room with no windows or furniture C. Room with an examining table, instrument cabinets, desk, and chair D. Nurse's office, furnished with chairs, files, magazines, and bookcases

A. Interview room furnished with a desk and two chairs Rationale: Individuals who are experiencing a severe-to-panic level of anxiety require a safe environment that is quiet, nonstimulating, structured, and simple. A room with a desk and two chairs provides simplicity, few objects with which the patient could cause self-harm, and a small floor space around which the patient can move. A small, empty storage room without windows or furniture would be like a jail cell. The nurse's office or a room with an examining table and instrument cabinets may be overstimulating and unsafe.

A patient tells the nurse at the clinic, "I haven't been taking my antidepressant medication as directed. I leave out the midday dose. I have lunch with friends and don't want them to ask me about the pills." Select the nurse's most appropriate intervention. A. Investigate the possibility of once-daily dosing of the antidepressant B. Suggest to the patient to take the medication when no one is watching C. Explain how taking each dose of medication on time relates to health maintenance D. Add the nursing diagnosis—Ineffective therapeutic regimen management, related to lack of knowledge—to the plan of care

A. Investigate the possibility of once-daily dosing of the antidepressant Rationale: Investigating the possibility of once-daily dosing of the antidepressant has the highest potential for helping the patient achieve compliance. Many antidepressants can be administered by once-daily dosing, a plan that increases compliance. Explaining how taking each dose of medication on time relates to health maintenance is reasonable but would not achieve the goal; it does not address the issue of stigma. The self-conscious patient would not be comfortable doing this. A better etiologic statement would be related to social stigma. The question asks for an intervention, not analysis.

A patient is having difficulty making a decision. The nurse has mixed feelings about whether to provide advice. Which principle usually applies? Giving advice: A. Is rarely helpful B. Fosters independence C. Lifts the burden of personal decision making D. Helps the patient develop feelings of personal adequacy

A. Is rarely helpful Rationale: Giving advice fosters dependence on the nurse and interferes with the patient's right to make personal decisions. Giving advice also robs patients of the opportunity to weigh alternatives and to develop problem-solving skills. Furthermore, it contributes to patient feelings of personal inadequacy. It also keeps the nurse in control and feeling powerful.

1. A nurse wishes to teach alternative coping strategies to a patient who is experiencing severe anxiety. The nurse will first need to: A. Lower the patient's current anxiety level B. Verify the patient's learning style C. Create outcomes and a teaching plan D. Assess how the patient uses defense mechanisms

A. Lower the patient's current anxiety level Rationale: A patient experiencing severe anxiety has a significantly narrowed perceptual field and difficulty attending to events in the environment. A patient experiencing severe anxiety will not learn readily. Determining preferred modes of learning, devising outcomes, and constructing teaching plans are relevant to the task but are not the priority measure. The nurse has already assessed the patient's anxiety level. Using defense mechanisms does not apply.

A soldier returns to the United States from active duty in a combat zone in Afghanistan. The soldier is diagnosed with posttraumatic stress disorder (PTSD). The nurse's highest priority is to screen this soldier for which problem? A. Major depression B. Bipolar disorder C. Schizophrenia D. Dementia

A. Major depression Rationale: Major depression frequently occurs with PTSD. The incidence of the disorders identified in the distracters is similar to the general population.

While talking with a patient with severe depression, a nurse notices the patient is unable to maintain eye contact. The patient's chin lowers to the chest while the patient looks at the floor. Which aspect of communication has the nurse assessed? A. Nonverbal communication B. A message filter C. A cultural barrier D. Social skills

A. Nonverbal communication Rationale: Eye contact and body movements are considered nonverbal communication. Insufficient data are available to determine the level of the patient's social skills or whether a cultural barrier exists.

A patient with a high level of motor activity runs from chair to chair and cries, "They're coming! They're coming!" The patient does not follow the staff's directions or respond to verbal interventions. The initial nursing intervention of highest priority is to: A. Provide for patient safety B. Increase environmental stimuli C. Respect the patient's personal space D. Encourage the clarification of feelings

A. Provide for patient safety Rationale: Safety is of highest priority; the patient who is experiencing panic is at high risk for self-injury related to an increase in non-goal-directed motor activity, distorted perceptions, and disordered thoughts. The goal should be to decrease the environmental stimuli. Respecting the patient's personal space is a lower priority than safety. The clarification of feelings cannot take place until the level of anxiety is lowered.

The spouse of a patient in hospice care angrily tells the nurse, "The care provided by the aide and other family members is inadequate, so I must do everything myself. Why can't anyone do things right?" The palliative care nurse should: A. Provide teaching about anticipatory grieving B. Assign new personnel to the patient's care C. Arrange hospitalization for the patient D. Refer the spouse for crisis counseling

A. Provide teaching about anticipatory grieving Rationale: The behaviors described in this scenario are consistent with anticipatory grieving. The spouse needs to be taught about the process of anticipatory grieving and to receive counseling to validate what she is experiencing and to enhance coping. The other options are not appropriate to the situation.

An individual experiences sexual dysfunction and blames it on a partner by calling the person unattractive and unromantic. Which defense mechanism is evident? A. Rationalization B. Compensation C. Introjection D. Regression

A. Rationalization Rationale: Rationalization involves unconsciously making excuses for one's behavior, inadequacies, or feelings. Regression involves the unconscious use of a behavior from an earlier stage of emotional development. Compensation involves making up for deficits in one area by excelling in another area. Introjection is an unconscious, intense identification with another person.

A person who is speaking about a rival for a significant other's affection says in a gushy, syrupy voice, "What a lovely person. That's someone I simply adore." The individual is demonstrating: A. Reaction formation B. Repression C. Projection D. Denial

A. Reaction formation Rationale: Reaction formation is an unconscious mechanism that keeps unacceptable feelings out of awareness by using the opposite behavior. Instead of expressing hatred for the other person, the individual gives praise. Denial operates unconsciously to allow an anxiety-producing idea, feeling, or situation to be ignored. Projection involves unconsciously disowning an unacceptable idea, feeling, or behavior by attributing it to another. Repression involves unconsciously placing an idea, feeling, or event out of awareness.

As a nurse discharges a patient, the patient gives the nurse a card of appreciation made in an arts and crafts group. What is the nurse's best action? A. Recognize the effectiveness of the relationship and patient's thoughtfulness. Accept the card B. Inform the patient that accepting gifts violates the policies of the facility. Decline the card C. Acknowledge the patient's transition through the termination phase but decline the card D. Accept the card and invite the patient to return to participate in other arts and crafts groups

A. Recognize the effectiveness of the relationship and patient's thoughtfulness. Accept the card Rationale: The nurse must consider the meaning, timing, and value of the gift. In this instance, the nurse should accept the patient's expression of gratitude.

A soldier in a combat zone tells the nurse, "I saw a child get blown up over a year ago, but I keep seeing bits of flesh everywhere. I see something red and the visions race back to my mind." Which phenomenon associated with post-traumatic stress disorder (PTSD) is this soldier describing? A. Reexperiencing B. Hyperarousal C. Avoidance D. Psychosis

A. Reexperiencing Rationale: Spontaneous or cued recurrent, involuntary, and intrusive distressing memories of the traumatic events are often associated with PTSD. The soldier has described intrusive thoughts and visions associated with reexperiencing the traumatic event. This description does not indicate psychosis, hypervigilance, or avoidance.

Which principle takes priority for the psychiatric inpatient staff when addressing behavioral crises? A. Resolve behavioral crises with the least restrictive intervention possible B. Rights of the majority of patients supersede the rights of individual patients C. Swift intervention is justified to maintain the integrity of the therapeutic milieu D. Allow patients the opportunities to regain control without intervention if the safety of other patients is not compromised

A. Resolve behavioral crises with the least restrictive intervention possible Rationale: The rule of using the least restrictive treatment or intervention possible to achieve the desired outcome is the patient's legal right. Planned interventions are nearly always preferable. Intervention may be necessary when the patient threatens harm to self.

An adult has cared for a debilitated parent for 10 years. The parent's condition has recently declined, and the health care provider has recommended placement in a skilled care facility. The adult says, "I've always been able to care for my parents. Nursing home placement goes against everything I believe." Successful resolution of this person's crisis will most closely relate to: A. Resolving the feelings associated with the threat to the person's self-concept B. Maintaining the ability to identify situational supports in the community C. Relying on the assistance from role models within the person's culture D. Mobilizing automatic relief behaviors by the person

A. Resolving the feelings associated with the threat to the person's self-concept Rationale: The patient's crisis clearly relates to a loss of (or threatened change in) self-concept. Her capacity to care for her parents, regardless of the deteriorating condition, has been challenged. Crisis resolution involves coming to terms with the feelings associated with this loss. Identifying situational supports is relevant but less so than coming to terms with the threat to self-concept. Reliance on lessons from role models can be helpful but is not the primary factor associated with resolution in this case. Automatic relief behaviors are not helpful and are part of the fourth phase of crisis.

Which technique will best communicate to a patient that the nurse is interested in listening? A. Restate a feeling or thought the patient has expressed B. Ask a direct question, such as "Did you feel angry?" C. Make a judgment about the patient's problem D. Say, "I understand what you're saying."

A. Restate a feeling or thought the patient has expressed Rationale: Restating allows the patient to validate the nurse's understanding of what has been communicated. Restating is an active listening technique. Judgments should be suspended in a nurse-patient relationship. Closed-ended questions such as, "Did you feel angry?" ask for specific information rather than show understanding. When the nurse simply states that he or she understands the patient's words, the patient has no way of measuring the understanding.

A patient with a high level of motor activity runs from chair to chair and cries, "They're coming! They're coming!" The patient is unable to follow staff direction or respond to verbal interventions. Which nursing diagnosis has the highest priority? A. Risk for injury B. Self-care deficit C. Disturbed energy field D. Disturbed thought processes

A. Risk for injury Rationale: A patient who is experiencing panic-level anxiety is at high risk for injury, related to an increase in non-goal-directed motor activity, distorted perceptions, and disordered thoughts. Existing data do not support the nursing diagnoses of Self-care deficit or Disturbed energy field. This patient has disturbed thought processes, but the risk for injury has a higher priority.

A new psychiatric nurse has a parent with bipolar disorder. This nurse angrily recalls embarrassing events concerning the parent's behavior in the community. Select the best ways for this nurse to cope with these feelings. Select all that apply. A. Seek ways to use the understanding gained from childhood to help patients cope with their own illnesses B. Recognize that these feelings are unhealthy, and try to suppress them when working with patients C. Recognize that psychiatric nursing is not an appropriate career choice, and explore other nursing specialties D. Begin new patient relationships by saying, "My own parent had mental illness, so I accept it without stigma." E. Recognize that the feelings may add sensitivity to the nurse's practice, but supervision is important

A. Seek ways to use the understanding gained from childhood to help patients cope with their own illnesses E. Recognize that the feelings may add sensitivity to the nurse's practice, but supervision is important Rationale: The nurse needs to explore these feelings. An experienced psychiatric nurse is a resource who may be helpful. The knowledge and experience gained from the nurse's relationship with a parent who is mentally ill may contribute sensitivity to a compassionate practice. Self-disclosure and suppression are not adaptive coping strategies. The nurse should not give up on this area of practice without first seeking ways to cope with the memories.

While conducting the initial interview with a patient in crisis, the nurse should: A. Speak in short, concise sentences B. Convey a sense of urgency to the patient C. Be forthright about time limits of the interview D. Let the patient know the nurse controls the interview

A. Speak in short, concise sentences Rationale: Severe anxiety narrows perceptions and concentration. By speaking in short, concise sentences, the nurse enables the patient to grasp what is being said. Conveying urgency will increase the patient's anxiety. Letting the patient know who controls the interview or stating that time is limited is nontherapeutic.

A nurse at the mental health center prepares to administer a scheduled injection of haloperidol decanoate (Haldol depot injection) to a patient with schizophrenia. As the nurse swabs the site, the patient shouts, "Stop, stop! I don't want to take that medicine anymore. I hate the side effects." Select the nurse's best initial action. A. Stop the medication administration procedure and say to the patient, "Tell me more about the side effects you've been having." B. Say to the patient, "Since I've already drawn the medication in the syringe, I'm required to give it, but let's talk to the doctor about skipping next month's dose." C. Proceed with the injection but explain to the patient that other medications are available that may help reduce the unpleasant side effects. D. Notify other staff members to report to the room for a show of force and proceed with the injection, using restraint if necessary.

A. Stop the medication administration procedure and say to the patient, "Tell me more about the side effects you've been having." Rationale: Patients with mental illness retain their civil rights unless clear, cogent, and convincing evidence of dangerousness exists. The patient in this situation presents no evidence of being dangerous. The nurse, an as advocate and educator, should seek more information about the patient's decision and should not force the medication.

A victim of spousal violence comes to the crisis center seeking help. The nurse uses crisis intervention strategies that focus on: A. Supporting emotional security and reestablishing equilibrium B. Offering a long-term resolution of issues precipitating the crisis C. Promoting growth of the individual D. Providing legal assistance

A. Supporting emotional security and reestablishing equilibrium Rationale: Strategies of crisis intervention are directed toward the immediate cause of the crisis and are aimed at bolstering the emotional security and reestablishing equilibrium, rather than focusing on underlying issues and long-term resolutions. The goal is to return the individual to the pre-crisis level of function. Crisis intervention is, by definition, short term. Promoting growth is a focus of long-term therapy. Providing legal assistance might be applicable.

A professor's 4-year-old child has a temperature of 101.6° F, diarrhea, and complains of stomach pain. The professor is scheduled to teach three classes today. Which actions by the professor demonstrate effective parenting? Select all that apply. A. Telephoning a grandparent to stay with the child at home for the day B. Telephoning a colleague to teach his classes and stays home with the sick child C. Taking the child to the university and keeps the child in a private office for the day D. Taking the child to a daycare center and hopes daycare workers will not notice the child is sick E. Giving the child one dose of ibuprofen (Motrin) and takes the child to the daycare center

A. Telephoning a grandparent to stay with the child at home for the day B. Telephoning a colleague to teach his classes and stays home with the sick child Rationale: The correct responses demonstrate fulfillment of the role as a parent. The distracters indicate the professor has not cared for the sick child in an effective way. Taking the child to a daycare center exposes other children to a potential infection. Taking the child to one's office does not keep the child comfortable or provide for the child while the professor is teaching.

When a female Mexican-American patient and a female nurse sit together, the patient often holds the nurse's hand. The patient also links arm and arm with the nurse when they walk. The nurse is uncomfortable with this behavior and thinks the patient is homosexual. Which alternative is a more accurate assessment? A. The patient is accustomed to touch during conversations, as are members of many Hispanic subcultures B. The patient understands that touch makes the nurse uncomfortable and controls the relationship based on that factor C. The patient is afraid of being alone. When touching the nurse, the patient is reassured and comforted D. The nurse is homophobic

A. The patient is accustomed to touch during conversations, as are members of many Hispanic subcultures Rationale: The most likely answer is that the patient's behavior is culturally influenced. Hispanic women frequently touch women they consider to be their friends. Although the other options are possible, they are less likely.

A nurse plans health teaching for a patient with generalized anxiety disorder who takes lorazepam (Ativan). What information should be included? Select all that apply. A. Use caution when operating machinery B. Allowed tyramine-free foods in diet C. Understand the importance of caffeine restriction D. Avoid alcohol and other sedatives E. Take the medication on an empty stomach

A. Use caution when operating machinery C. Understand the importance of caffeine restriction D. Avoid alcohol and other sedatives Rationale: Caffeine is a central nervous system stimulant that acts as an antagonist to the benzodiazepine lorazepam. Daily caffeine intake should be reduced to the amount contained in one cup of coffee. Benzodiazepines are sedatives, thus the importance of exercising caution when driving or using machinery and the importance of not using other central nervous system depressants such as alcohol or sedatives to avoid potentiation. Benzodiazepines do not require a special diet. Food will reduce gastric irritation from the medication.

A patient with mental illness asks a psychiatric technician, "What's the matter with me?" The technician replies, "Your wing nuts need tightening." The patient looks bewildered and wanders off. The nurse who overheard the exchange should take action based on: A. Violation of the patient's right to be treated with dignity and respect B. The nurse's obligation to report caregiver negligence C. Preventing defamation of the patient's character D. Supervisory liability

A. Violation of the patient's right to be treated with dignity and respect Rationale: Patients have the right to be treated with dignity and respect. Patients should never be made the butt of jokes about their illness. Patient emotional abuse has been demonstrated, not negligence. The technician's response was not clearly defamation. Patient abuse, not supervisory liability, is the issue.

The acronym QSEN refers to: A. Qualitative Standardized Excellence in Nursing B. Quality and Safety Education for Nurses C. Quantitative Effectiveness in Nursing D. Quick Standards Essential for Nurses

B. Quality and Safety Education for Nurses Rationale: QSEN represents national initiatives centered on patient safety and quality. The primary goal of QSEN is to prepare future nurses with the knowledge, skills, and attitudes to increase the quality, care, and safety in the health care setting in which they work.

A person whose husband is terminally ill says, "I don't want to cry in front of him. I don't want him to know how close he is to death or how sad I am." Which response by the nurse would be most therapeutic? A. "You're right to protect him at a time when he is so vulnerable." B. "He might be more reassured than disturbed by your tears." C. "It's important for you to know that time is running out." D. "You definitely need to be honest about your feelings."

B. "He might be more reassured than disturbed by your tears." Rationale: Many people try to protect the dying person from experiencing emotions; however, emotional honesty is important to both the patient and the family. The patient may be reassured knowing that the family is facing the inevitable. Giving advice and making judgmental statements are not helpful.

After the death of his wife, a man tells the nurse, "I can't live without her. She was my whole life." Which is the nurse's most therapeutic reply? A. "Each day will get a little better." B. "Her death is a terrible loss for you." C. "Remember, she's no longer suffering." D. "Your friends will help you cope with this."

B. "Her death is a terrible loss for you." Rationale: The correct response demonstrates the use of reflection, a therapeutic communication technique. A statement that validates the bereaved person's loss is more helpful than banalities and clichés; it signifies understanding. The other options are clichés.

A patient cries as the nurse explores the patient's relationship with a deceased parent. The patient says, "I shouldn't be crying like this. It happened a long time ago." Which responses by the nurse will facilitate communication? Select all that apply. A. "Why do you think you are so upset?" B. "I can see that you feel sad about this situation." C. "The loss of your parent is very painful for you." D. "Crying is a way of expressing the hurt you're experiencing." E. "Let's talk about something else because this subject is upsetting you."

B. "I can see that you feel sad about this situation." C. "The loss of your parent is very painful for you." D. "Crying is a way of expressing the hurt you're experiencing." Rationale: Reflecting ("I can see that you feel sad" or "This is very painful for you") and giving information ("Crying is a way of expressing hurt") are therapeutic techniques. "Why" questions often imply criticism or seem intrusive or judgmental, and they are difficult to answer. Changing the subject is a barrier to communication.

A soldier returns to the United States from active duty in a combat zone in Afghanistan. The soldier is diagnosed with post-traumatic stress disorder (PTSD). Which comment by the soldier requires the nurse's immediate attention? A. "It's good to be home. I missed my family and friends." B. "I saw my best friend get killed by a roadside bomb. It should have been me instead." C. "Sometimes I think I hear bombs exploding, but it's just the noise of traffic in my hometown." D. "I want to continue my education but I'm not sure how I will fit in with other college students."

B. "I saw my best friend get killed by a roadside bomb. It should have been me instead." Rationale: The correct response indicates the soldier is thinking about death and feeling survivor's guilt. These emotions may accompany suicidal ideation, which warrants the nurse's follow-up assessment. Suicide is a high risk among military personnel diagnosed with PTSD. One distracter indicates flashbacks, which is common with individuals with PTSD but is not solely indicative of further problems. The other distracters are normal emotions associated with returning home and change.

A patient comes to the clinic with superficial cuts on the left wrist. The patient paces around the room sobbing but cringes when approached and responds to questions with only shrugs or monosyllables. Select the nurse's best initial statement to this patient. A. "Everything is going to be all right. You are here at the clinic, and the staff will keep you safe." B. "I see you are feeling upset. I'm going to stay and talk with you to help you feel better." C. "You need to try to stop crying so we can talk about your problems." D. "Let's set some guidelines and goals for your visit here."

B. "I see you are feeling upset. I'm going to stay and talk with you to help you feel better." Rationale: A crisis exists for this patient. The two primary thrusts of crisis intervention are to provide for the safety of the individual and use anxiety-reduction techniques to facilitate the use of inner resources. The nurse offers therapeutic presence, which provides caring, ongoing observation relative to the patient's safety, and interpersonal reassurance.

A voluntarily hospitalized patient tells the nurse, "Get me the forms for discharge against medical advice so I can leave now." What is the nurse's best initial response? A. "I can't give you those forms without your health care provider's knowledge." B. "I will get them for you, but let's talk about your decision to leave treatment." C. "Since you signed your consent for treatment, you may leave if you desire." D. "I'll get the forms for you right now and bring them to your room."

B. "I will get them for you, but let's talk about your decision to leave treatment." Rationale: A patient who has been voluntarily admitted as a psychiatric inpatient has the right to demand and obtain release in most states. However, as a patient advocate, the nurse is responsible for weighing factors related to the patient's wishes and best interests. By asking for information, the nurse may be able to help the patient reconsider the decision. The statement that discharge forms cannot be given without the health care provider's knowledge is not true. Facilitating discharge without consent is not in the patient's best interest before exploring the reason for the request.

During the initial interview at the crisis center, a patient says, "I've been served with divorce papers. I'm so upset and anxious that I can't think clearly." What could the nurse say to assess personal coping skills? A. "What would you like us to do to help you feel more relaxed?" B. "In the past, how did you handle difficult or stressful situations?" C. "Do you think you deserve to have things like this happen to you?" D. "I can see you are upset. You can rely on us to help you feel better."

B. "In the past, how did you handle difficult or stressful situations?" Rationale: The correct answer is the only option that assesses coping skills. The incorrect options offer unrealistic reassurance, are concerned with self-esteem, and ask the patient to decide on treatment at a time when he or she "cannot think clearly."

A patient with paranoid schizophrenia tells the nurse, "The CIA is monitoring us through the fluorescent lights in this room. Be careful what you say." Which response by the nurse would be most therapeutic? A. "Let's talk about something other than the CIA." B. "It sounds like you're concerned about your privacy." C. "The CIA is prohibited from operating in health care facilities." D. "You have lost touch with reality, which is a symptom of your illness."

B. "It sounds like you're concerned about your privacy." Rationale: It is important not to challenge the patient's beliefs, even if they are unrealistic. Challenging undermines the patient's trust in the nurse. The nurse should try to understand the underlying feelings or thoughts the patient's message conveys. The correct response uses the therapeutic technique of reflection. The other comments are nontherapeutic. Asking to talk about something other than the concern at hand is changing the subject. Saying that the CIA is prohibited from operating in health care facilities gives false reassurance. Stating that the patient has lost touch with reality is truthful but uncompassionate.

A nurse's neighbor asks, "Why aren't people with mental illness kept in state institutions anymore?" What is the nurse's best response? A. "Many people are still in psychiatric institutions. Inpatient care is needed because many people who are mentally ill are violent." B. "Less restrictive settings are now available to care for individuals with mental illness." C. "Our nation has fewer persons with mental illness; therefore fewer hospital beds are needed." D. "Psychiatric institutions are no longer popular as a consequence of negative stories in the press."

B. "Less restrictive settings are now available to care for individuals with mental illness." Rationale: The community is a less restrictive alternative than hospitals for the treatment of people with mental illness. The remaining options are incorrect and part of the stigma of mental illness.

Which entry in the medical record best meets the requirement for problem-oriented charting? A. "A: Pacing and muttering to self. P: Sensory perceptual alteration, related to internal auditory stimulation. I: Given fluphenazine (Prolixin) 2.5 mg at 0900, and went to room to lie down. E: Calmer by 0930. Returned to lounge to watch TV." B. "S: States, 'I feel like I'm ready to blow up.' O: Pacing hall, mumbling to self. A: Auditory hallucinations. P: Offer haloperidol (Haldol) 2 mg . I: (Haldol) 2 mg at 0900. E: Returned to lounge at 0930 and quietly watched TV." C. "Agitated behavior. D: Patient muttering to self as though answering an unseen person. A: Given haloperidol (Haldol) 2 mg and went to room to lie down. E: Patient calmer. Returned to lounge to watch TV." D. "Pacing hall and muttering to self as though answering an unseen person. haloperidol (Haldol) 2 mg administered at 0900 with calming effect in 30 minutes. Stated, 'I'm no longer bothered by the voices.'"

B. "S: States, 'I feel like I'm ready to blow up.' O: Pacing hall, mumbling to self. A: Auditory hallucinations. P: Offer haloperidol (Haldol) 2 mg . I: (Haldol) 2 mg at 0900. E: Returned to lounge at 0930 and quietly watched TV." Rationale: Problem-oriented documentation uses the first letter of key words to organize data: S for subjective data, O for objective data, A for assessment, P for plan, I for intervention, and E for evaluation. The distracters offer examples of PIE charting, focus documentation, and narrative documentation.

An individual was killed during a store robbery 2 weeks earlier. The widowed spouse, who has schizoaffective disorder, cries spontaneously when talking about the death. Which is the nurse's most therapeutic comment? A. "I'm worried about how much you're crying. Your grief over your spouse's death has gone on too long." B. "The unexpected death of your spouse must be so painful. I'm glad you're able to talk to me about your feelings." C. "This loss is harder to accept because of your mental illness. Let's refer you to the partial hospitalization program." D. "Your crying shows me you aren't coping well. I made an appointment for you to see the psychiatrist for medication adjustment."

B. "The unexpected death of your spouse must be so painful. I'm glad you're able to talk to me about your feelings." Rationale: The patient is expressing feelings related to the loss, and this is an expected and healthy behavior. This patient is at risk for dysfunctional grieving because of the history of a severe psychiatric illness, but the nurse's priority intervention is to form a therapeutic alliance and support the patient's expression of feelings. The patient's crying 2 weeks after the spouse's death is expected and normal.

An appropriate question for the nurse to ask to assess situational support is: A. "Has anything upsetting occurred in the past few days?" B. "Who can be helpful to you during this time?" C. "How does this problem affect your life?" D. "What led you to seek help at this time?"

B. "Who can be helpful to you during this time?" Rationale: Only the correct answer focuses on situational support. The incorrect options focus on the patient's perception of the precipitating event.

A patient with acute depression states, "God is punishing me for my past sins." What is the nurse's best response? A. "Why do you think that?" B. "You sound very upset about this." C. "You believe God is punishing you for your sins?" D. "If you feel this way, you should talk to a member of your clergy."

B. "You sound very upset about this." Rationale: The nurse reflects on the patient's comment, a therapeutic technique to encourage sharing for perceptions and feelings. The incorrect responses reflect probing, closed-ended comments, and giving advice, all of which are nontherapeutic.

Which nursing intervention demonstrates false imprisonment? A. A confused and combative patient says, "I'm getting out of here and no one can stop me." The nurse restrains this patient without a health care provider's order and then promptly obtains an order. B. A patient has been irritating, seeking the attention of nurses most of the day. Now a nurse escorts the patient down the hall saying, "Stay in your room or you'll be put in seclusion." C. An involuntarily hospitalized patient with suicidal ideation runs out of the psychiatric unit. A nurse rushes after the patient and convinces the patient to return to the unit. D. An involuntarily hospitalized patient with suicidal ideation attempts to leave the unit. A nurse calls the security team and uses established protocols to prevent the patient from leaving.

B. A patient has been irritating, seeking the attention of nurses most of the day. Now a nurse escorts the patient down the hall saying, "Stay in your room or you'll be put in seclusion." Rationale: False imprisonment involves holding a competent person against his or her will. Actual force is not a requirement of false imprisonment. The individual needs only to be placed in fear of imprisonment by someone who has the ability to carry out the threat. The patient in one distracter is not competent, and the nurse is acting beneficently. The patients in the other distracters have been admitted as involuntary patients and should not be allowed to leave without permission of the treatment team.

What is the desirable outcome for the orientation stage of a nurse-patient relationship? The patient will demonstrate behaviors that indicate: A. Great sense of independence B. Rapport and trust with the nurse C. Self-responsibility and autonomy D. Resolved transference

B. Rapport and trust with the nurse Rationale: The development of rapport and trust is necessary before the relationship can progress to the working phase. Behaviors indicating a greater sense of independence, self-responsibility, and resolved transference occur in the working phase.

An adolescent comes to the crisis clinic and reports sexual abuse by an uncle. The patient told the parents about the uncle's behavior, but the parents did not believe the adolescent. What type of crisis exists? A. Maturational B. Adventitious C. Situational D. Organic

B. Adventitious Rationale: An adventitious crisis is a crisis of disaster that is not a part of everyday life; it is unplanned or accidental. Adventitious crises include natural disasters, national disasters, and crimes of violence. Sexual molestation falls within this classification. Maturational crisis occurs as an individual arrives at a new stage of development, when old coping styles may be ineffective. Situational crisis arises from an external source such as a job loss, divorce, or other loss affecting self-concept or self-esteem. Organic is not a type of crisis.

Which finding indicates the successful completion of an individual's grieving process? A. For 2 years, a person has kept the deceased spouse's belongings in their usual places B. After 15 months, a widowed person realistically remembers both the pleasures and disappointments of the relationship with the spouse C. Three years after the death, a person talks about the spouse as if the spouse were still alive and weeps when others mention the spouse's name D. Eighteen months after the spouse's death, a person says, "I never cry or have feelings of loss even though we were always very close."

B. After 15 months, a widowed person realistically remembers both the pleasures and disappointments of the relationship with the spouse Rationale: The work of grieving is over when the bereaved can remember the individual realistically and acknowledge both the pleasure and disappointments associated with the loved one. The individual is then free to enter into new relationships and activities. The other options suggest unresolved grief.

Shortly after a man's wife dies, the man approaches the nurse who cared for his wife during the final hours of life and says angrily, "If you had given her your undivided attention, she would still be alive." Which analysis applies? A. The comment summarizes the nurse's inadequacies B. Anger is a phenomenon experienced during grieving C. The patient had ambivalent feelings about his spouse D. In some cultures, grief is expressed solely through anger

B. Anger is a phenomenon experienced during grieving Rationale: Anger may protect the bereaved from facing the devastating reality of the loss. Anger expressed during mourning is not directed toward the nurse, personally, although accusations and blame may make him or her feel as though it is.

A patient comes to the crisis center saying, "I'm in a terrible situation. I don't know what to do." The triage nurse can initially assume that the patient is: A. Suicidal B. Anxious and fearful C. Misperceiving reality D. Potentially homicidal

B. Anxious and fearful Rationale: Individuals in crisis are universally anxious. They are often frightened and may be mildly confused. Perceptions are often narrowed.

A patient is very suspicious and states, "The FBI has me under surveillance." Which strategies should a nurse use when gathering initial assessment data about this patient? Select all that apply. A. Tell the patient that medication will help this type of thinking B. Ask the patient, "Tell me about the problem as you see it." C. Seek information about when the problem began D. Tell the patient, "Your ideas are not realistic." E. Reassure the patient, "You are safe here."

B. Ask the patient, "Tell me about the problem as you see it." C. Seek information about when the problem began E. Reassure the patient, "You are safe here." Rationale: During the assessment interview, the nurse should listen attentively and accept the patient's statements in a nonjudgmental way. Because the patient is suspicious and fearful, reassuring safety may be helpful, although trust is unlikely so early in the relationship. Saying that medication will help or telling the patient that the ideas are not realistic will undermine the development of trust between the nurse and patient.

A patient with paranoid schizophrenia believes evil spirits are being summoned by a local minister and verbally threatens to bomb a local church. The psychiatrist notifies the minister. The psychiatrist has: A. Released information without proper authorization B. Demonstrated the duty to warn and protect C. Violated the patient's confidentiality D. Avoided charges of malpractice

B. Demonstrated the duty to warn and protect Rationale: The duty of a health care professional is to warn or notify an intended victim after a threat of harm has been made. Informing a potential victim of a threat is a legal responsibility of the health care professional and not considered a violation of confidentiality.

A woman says, "I can't take it anymore! Last year my husband had an affair and now we don't communicate. Three months ago, I found a lump in my breast. Yesterday my daughter said she's quitting college." If this person's immediate family is unable to provide sufficient situational support, the nurse should: A. Suggest hospitalization for a short period B. Ask what other relatives or friends are available for support C. Tell the patient, "You must be strong. Don't let this crisis overwhelm you." D. Foster insight by relating the present situation to earlier situations involving loss

B. Ask what other relatives or friends are available for support Rationale: The assessment of situational supports should continue. Although the patient's nuclear family may not be supportive, other situational supports may be available. If they are adequate, admission to an inpatient unit will be unnecessary. Psychotherapy is not appropriate for crisis intervention. Advice is usually nontherapeutic.

Before assessing a new patient, a nurse is told by another health care worker, "I know that patient. No matter how hard we work, there isn't much improvement by the time of discharge." The nurse's responsibility is to: A. Document the other worker's assessment of the patient B. Assess the patient based on data collected from all sources C. Validate the worker's impression by contacting the patient's significant other D. Discuss the worker's impression with the patient during the assessment interview

B. Assess the patient based on data collected from all sources Rationale: Assessment should include data obtained from both the primary and reliable secondary sources. Biased assessments by others should be evaluated as objectively as possible by the nurse, keeping in mind the possible effects of counter-transference.

A patient approaches the nurse and impatiently blurts out, "You've got to help me! Something terrible is happening. My heart is pounding." The nurse responds, "It's almost time for visiting hours. Let's get your hair combed." Which approach has the nurse used? A. Distracting technique to lower anxiety B. Bringing up an irrelevant topic C. Responding to physical needs D. Addressing false cognitions

B. Bringing up an irrelevant topic Rationale: The nurse has closed off patient-centered communication. The introduction of an irrelevant topic makes the nurse feel better. The nurse is uncomfortable dealing with the patient's severe anxiety.

A nurse asks a patient, "If you had fever and vomiting for 3 days, what would you do?" Which aspect of the mental status examination is the nurse assessing? A. Behavior B. Cognition C. Affect and mood D. Perceptual disturbances

B. Cognition Rationale: Assessing cognition involves determining a patient's judgment and decision-making capabilities. In this case, the nurse expects a response of, "Call my doctor" if the patient's cognition and judgment are intact. If the patient responds, "I would stop eating" or "I would just wait and see what happened," the nurse would conclude that judgment is impaired. The other options refer to other aspects of the examination.

The relationship between a nurse and patient as it relates to status and power is best described by which term? A. Symmetric B. Complementary C. Incongruent D. Paralinguistic

B. Complementary Rationale: When a difference in power exists, as between a student and teacher or between a nurse and patient, the relationship is said to be complementary. Symmetrical relationships exist between individuals of like or equal status. Incongruent and paralinguistic are not terms used to describe relationships.

A nurse encourages an anxious patient to talk about feelings and concerns. What is the rationale for this intervention? A. Offering hope allays and defuses the patient's anxiety B. Concerns stated aloud become less overwhelming and help problem solving to begin C. Anxiety is reduced by focusing on and validating what is occurring in the environment D. Encouraging patients to explore alternatives increases the sense of control and lessens anxiety

B. Concerns stated aloud become less overwhelming and help problem solving to begin Rationale: All principles listed are valid, but the only rationale directly related to the intervention of assisting the patient to talk about feelings and concerns is the one that states that concerns spoken aloud become less overwhelming and help problem solving to begin.

A nurse assesses a confused older adult. The nurse experiences sadness and reflects, "The patient is like one of my grandparents...so helpless." What feelings does the nurse describe? A. Transference B. Countertransference C. Catastrophic reaction D. Defensive coping reaction

B. Countertransference Rationale: Countertransference is the nurse's transference or response to a patient that is based on the nurse's unconscious needs, conflicts, problems, or view of the world.

A patient who is preparing for surgery has moderate anxiety and is unable to understand preoperative information. Which nursing intervention is appropriate? A. Reassure the patient that all nurses are skilled in providing postoperative care B. Describe the procedure again in a calm manner using simple language C. Tell the patient that the staff is prepared to promote recovery D. Encourage the patient to express feelings to his or her family

B. Describe the procedure again in a calm manner using simple language Rationale: Providing information in a calm, simple manner helps the patient grasp the important facts. Introducing extraneous topics as described in the remaining options will further scatter the patient's attention.

After formulating the nursing diagnoses for a new patient, what is the next action a nurse should take? A. Design interventions to include in the plan of care B. Determine the goals and outcome criteria C. Implement the nursing plan of care D. Complete the spiritual assessment

B. Determine the goals and outcome criteria Rationale: The third step of the nursing process is planning and outcome identification. Outcomes cannot be determined until the nursing assessment is complete and the nursing diagnoses have been formulated.

As death approaches, a patient with acquired immunodeficiency syndrome (AIDS) says, "I don't want to see a lot of visitors anymore. Just my parents and my sibling can come in for a while each day." What action should the nurse take? A. Ask the patient to reconsider the decision because many interested and caring friends can be sources of support B. Discuss the request with the parents and sibling. Suggest that they explain the patient's decision to friends C. Suggest that the patient discuss these wishes with the health care provider D. Place a "no visitors" sign on the patient's door

B. Discuss the request with the parents and sibling. Suggest that they explain the patient's decision to friends Rationale: As many patients approach death, they begin to withdraw. In the stage of acceptance, many patients are exhausted and tired, and interactions of a social nature are a burden. Many prefer to have someone present at the bedside who will sit without constantly talking.

A patient tells the nurse, "I wanted my health care provider to prescribe diazepam (Valium) for my anxiety disorder, but buspirone (BuSpar) was prescribed instead. Why?" The nurse's reply should be based on the knowledge that buspirone: A. Does not produce blood dyscrasias B. Does not cause dependence C. Can be administered as needed D. Is faster acting than diazepam

B. Does not cause dependence Rationale: Buspirone is considered effective in the long-term management of anxiety because it is not habituating. Because it is long acting, buspirone is not valuable as an as-needed or as a fast-acting medication. The fact that buspirone does not produce blood dyscrasias is less relevant in the decision to prescribe buspirone.

A patient performs ritualistic hand washing. What should the nurse do to help the patient develop more effective coping strategies? A. Allow the patient to set a hand-washing schedule B. Encourage the patient to participate in social activities C. Encourage the patient to discuss hand-washing routines D. Focus on the patient's symptoms rather than on the patient

B. Encourage the patient to participate in social activities Rationale: Because patients with obsessive-compulsive disorder become overly involved in rituals, promoting involvement with other people and activities is necessary to improve the patient's coping strategies. Daily activities prevent the constant focus on anxiety and its symptoms. The other interventions focus on the compulsive symptom.

Psychotherapy for individuals at risk for complicated grief focuses on which goals? Select all that apply. A. Identifying ways to break bonds with the deceased B. Exploring emotional responses to a loss C. Solving problems related to moving forward in life D. Learning about the stages and symptoms of grieving E. Using antipsychotic medications for dysfunctional grief

B. Exploring emotional responses to a loss C. Solving problems related to moving forward in life D. Learning about the stages and symptoms of grieving Rationale: Grief therapy is offered when a problem—not necessarily dysfunctional grief—exists or is anticipated. It focuses on emotional responses to loss and problem solving related to moving forward in life. Anxiety and/or depression may develop, even with normal grief, and require the short-term use of anxiolytic or antidepressant medications; however, antipsychotic drugs would not be expected. Physical symptoms such as weakness, anorexia, shortness of breath, tightness of the chest, dry mouth, and gastrointestinal disturbances may accompany acute grief, but the development of actual complications indicates dysfunctional grief.

A nurse manager notices that a staff member spends minimal time with a patient who is terminally ill with AIDS. The patient says, "I'm having intense emotional reactions to this illness. Sometimes I feel angry, but other times I feel afraid or abandoned." The nurse manager can correctly hypothesize that the most likely reason for the staff member's avoidance is: A. High risk for infection transmission B. Feelings of inadequacy in dealing with complex emotional needs C. Knowledge that the patient needs time alone with family and friends D. Belief that the patient's former lifestyle included high-risk behaviors

B. Feelings of inadequacy in dealing with complex emotional needs Rationale: Many nurses tend to be more comfortable with meeting physical needs than in focusing on emotional needs. Standard precautions are necessary for all patients. The patient's lifestyle is irrelevant.

A soldier returned home from active duty in a combat zone in Afghanistan and was diagnosed with post-traumatic stress disorder (PTSD). The soldier says, "If there's a loud noise at night, I get under my bed because I think we're getting bombed." What type of experience has the soldier described? A. Illusion B. Flashback C. Nightmare D. Auditory hallucination

B. Flashback Rationale: Flashbacks are dissociative reactions in which an individual feels or acts as if the traumatic event were recurring. Illusions are misinterpretations of stimuli; although the experience is similar, the more accurate term is flashback because of the diagnosis of PTSD. Auditory hallucinations have no external stimuli. Nightmares commonly accompany PTSD, but this experience is stimulated by an actual environmental sound.

Cortisol is released in response to a patient's prolonged stress. Which initial effect would the nurse expect to result from the increased cortisol level? A. Diuresis and electrolyte imbalance B. Focused and alert mental status C. Drowsiness and lethargy D. Restlessness and anxiety

B. Focused and alert mental status Rationale: Cortisol is the primary stress hormone and is released in response to prolonged stress. Cortisol helps supply cells with amino acids and fatty acids for energy supply, as well as diverting glucose from muscles for use by the brain. As a result, the brain stays alert and focused. The distracters present effects that would not be expected.

Which communication technique is used more in crisis intervention than traditional counseling? A. Role modeling B. Giving direction C. Information giving D. Empathic listening

B. Giving direction Rationale: The nurse working in crisis intervention must be creative and flexible in looking at the patient's situation and suggesting possible solutions to the patient. Giving direction is part of the active role a crisis intervention therapist takes. The other options are used equally in crisis intervention and traditional counseling roles.

The mourning process is more difficult when the bereaved: A. Was relatively independent of the deceased B. Has experienced a number of previous losses C. Accepts that death is expected for older adults D. Had few unresolved conflicts with the deceased

B. Has experienced a number of previous losses Rationale: Factors that have negative effects on the mourning process include a high dependency on the deceased, ambivalence toward the deceased, a poor or absent support system, a high number of past losses or other recent losses, poor physical or mental health, and young age of the deceased. Data do not support the incorrect options.

A person whose spouse died 2 years earlier tells friends, "I think I'll begin to go out socially, maybe even take someone to dinner." This comment best demonstrates that the individual is: A. Denying the significance of the loss B. In a period of resolution of grief C. Actively working through grief D. Experiencing intrusion

B. In a period of resolution of grief Rationale: Toward the end of the grief process, the person renews his or her interest in people and activities. This behavior indicates resolution. At the same time, the person is released from the relationship with the deceased. The patient has progressed beyond grief. The patient is seeking to move into new relationships so that he or she is not alone.

Which scenario best demonstrates an example of eustress? An individual: A. Loses a beloved family pet B. Prepares to take a 1 week vacation to a tropical island with a group of close friends C. Receives a bank notice there were insufficient funds in their account for a recent rent payment D. Receives notification that their current employer is experiencing financial problems and some workers will be terminated

B. Prepares to take a 1 week vacation to a tropical island with a group of close friends Rationale: Eustress is beneficial stress; it motivates people to develop skills they need to solve problems and meet personal goals. Positive life experiences produce eustress. Going on a tropical vacation is an exciting, relaxing experience and an example of eustress. Losing the family pet, worrying about employment security, and having financial problems are examples of distress, a negative experience that drains energy and can lead to significant emotional problems.

A new nurse tells a mentor, "I want to convey to my patients that I am interested in them and that I want to listen to what they have to say." Which behaviors are helpful in meeting the nurse's goal? Select all that apply. A. Sitting behind a desk, facing the patient B. Introducing self to a patient and identifying own role C. Using facial expressions that convey interest and encouragement D. Assuming an open body posture and sometimes mirror imaging E. Maintaining control of the topic under discussion by asking direct questions

B. Introducing self to a patient and identifying own role C. Using facial expressions that convey interest and encouragement D. Assuming an open body posture and sometimes mirror imaging Rationale: Trust is fostered when the nurse gives an introduction and identifies his or her role. Facial expressions that convey interest and encouragement support the nurse's verbal statements to that effect and strengthen the message. An open body posture conveys openness to listening to what the patient has to say. Mirror imaging enhances patient comfort. A desk would place a physical barrier between the nurse and patient. A face-to-face stance should be avoided when possible, and a less intense 90- or 120-degree angle is used to permit either party to look away without discomfort. Once introductions have been made, the nurse focuses the interview on the patient by using open-ended questions such as, "Where should we start?"

The patient says, "My marriage is just great. My spouse and I usually agree on everything." The nurse observes the patient's foot moving continuously as the patient twirls a shirt button. What conclusion can the nurse draw? The patient's communication is: A. Clear B. Mixed C. Precise D. Inadequate

B. Mixed Rationale: Mixed messages involve the transmission of conflicting or incongruent messages by the speaker. The patient's verbal message that all is well in the relationship is modified by the nonverbal behaviors denoting anxiety. Data are not present to support the choice of the verbal message being clear, explicit, or inadequate.

A patient with a mass in the left upper lobe of the lung is scheduled for a biopsy. The patient has difficulty understanding the nurse's comments and asks, "What do you mean? What are they going to do?" Assessment findings include a tremulous voice, respirations at 28 breaths per minute, and a pulse rate at 110 beats per minute. What is the patient's level of anxiety? A. Mild B. Moderate C. Severe D. Panic

B. Moderate Rationale: Moderate anxiety causes the individual to grasp less information and reduces his or her problem-solving ability to a less-than-optimal level. Mild anxiety heightens attention and enhances problem-solving abilities. Severe anxiety causes great reduction in the perceptual field. Panic-level anxiety results in disorganized behavior.

A community mental health nurse has worked for 6 months to establish a relationship with a delusional, suspicious patient. The patient recently lost employment and stopped taking medications because of inadequate money. The patient says, "Only a traitor would make me go to the hospital." Which solution is best? A. Arrange a bed in a local homeless shelter with nightly onsite supervision B. Negotiate a way to provide medication so the patient can remain at home C. Hospitalize the patient until the symptoms have stabilized D. Seek inpatient hospitalization for up to 1 week

B. Negotiate a way to provide medication so the patient can remain at home Rationale: Hospitalization may damage the nurse-patient relationship even if it provides an opportunity for rapid stabilization. If medication can be obtained and restarted, the patient can possibly be stabilized in the home setting, even if it takes a little longer. A homeless shelter is inappropriate and unnecessary. Hospitalization may be necessary later, but a less restrictive solution should be tried first because the patient is not dangerous.

A patient with schizophrenia has been stable in the community. Today, the spouse reports the patient is delusional and explosive. The patient says, "I'm willing to take my medicine, but I forgot to get my prescription refilled." Which outcome should the nurse add to the plan of care? A. Nurse will obtain prescription refills every 90 days and deliver them to the patient B. Patient's spouse will mark dates for prescription refills on the family calendar C. Patient will report to the hospital for medication follow-up every week D. Patient will call the nurse weekly to discuss medication-related issues

B. Patient's spouse will mark dates for prescription refills on the family calendar Rationale: The nurse should use the patient's support system to meet patient needs whenever possible. Delivery of medication by the nurse should be unnecessary for the nurse to do if the patient or a significant other can be responsible. The patient may not need more intensive follow-up as long as he or she continues to take the medications as prescribed. No patient issues except failure to obtain medication refills were identified.

A nurse assesses a patient who reluctantly participates in activities, answers questions with minimal responses, and rarely makes eye contact. What information should be included when documenting the assessment? Select all that apply. A. Uncooperative patient B. Patient's subjective responses C. Only data obtained from the patient's verbal responses D. Description of the patient's behavior during the interview E. Analysis of why the patient is unresponsive during the interview

B. Patient's subjective responses D. Description of the patient's behavior during the interview Rationale: Both the content and process of the interview should be documented. Providing only the patient's verbal responses creates a skewed picture of the patient. Writing that the patient is uncooperative is subjectively worded. An objective description of patient behavior is preferable. Analysis of the reasons for the patient's behavior is speculation, which is inappropriate.

An adult comes to the crisis clinic after being terminated from a job of 15 years. The patient says, "I don't know what to do. How can I get another job? Who will pay the bills? How will I feed my family?" Which nursing diagnosis applies? A. Hopelessness B. Powerlessness C. Chronic low self-esteem D. Disturbed thought processes

B. Powerlessness Rationale: The patient describes feelings of the lack of control over life events. No direct mention is made of hopelessness or chronic low self-esteem. The patient's thought processes are not shown to be altered at this point.

After leaving work, a staff nurse realizes that documentation of the administration of a medication to a patient was omitted. This off-duty nurse telephones the unit and tells the nurse, "Please document the administration of the medication I forgot to do. My password is alpha1." The nurse should: A. Fulfill the request B. Refer the matter to the charge nurse to resolve C. Access the record and document the information D. Report the request to the patient's health care provider

B. Refer the matter to the charge nurse to resolve Rationale: At most hospitals, termination is a possible penalty for unauthorized entry into a patient record. Referring the matter to the charge nurse will allow the observance of hospital policy while ensuring that documentation occurs. Making an exception and fulfilling the request places the on-duty staff nurse in jeopardy. Reporting the request to the patient's health care provider would be unnecessary. Accessing the record and documenting the information would be unnecessary when the charge nurse can resolve the problem.

An African-American patient says to a Caucasian nurse, "There's no sense talking. You wouldn't understand because you live in a white world." The nurse's best action would be to: A. Explain, "Yes, I do understand. Everyone goes through the same experiences." B. Say, "Please give an example of something you think I wouldn't understand." C. Reassure the patient that nurses are in contact with people from all cultures. D. Change the subject to one that is less emotionally disturbing.

B. Say, "Please give an example of something you think I wouldn't understand." Rationale: Having the patient speak in specifics rather than globally helps the nurse understand the patient's perspective. This approach helps the nurse engage the patient.

A community psychiatric nurse assesses that a patient with a mood disorder is more depressed than on the previous visit a month ago; however, the patient says, "I feel the same." Which intervention supports the nurse's assessment while preserving the patient's autonomy? A. Arrange for a short hospitalization B. Schedule weekly clinic appointments C. Refer the patient to the crisis intervention clinic D. Call the family and ask them to observe the patient closely

B. Schedule weekly clinic appointments Rationale: Scheduling clinic appointments at shorter intervals will give the opportunity for more frequent assessment of symptoms and allow the nurse to use early intervention. If the patient does not admit to having a crisis or problem, this referral would be useless. The remaining options may produce unreliable information, violate the patient's privacy, and waste scarce resources.

A nurse should introduce the matter of a contract during the first session with a new patient because contracts: A. Specify what the nurse will do for the patient B. Spell out the participation and responsibilities of each party C. Indicate the feeling tone established between the participants D. Are binding and prevent either party from prematurely ending the relationship

B. Spell out the participation and responsibilities of each party Rationale: A contract emphasizes that the nurse works with the patient rather than doing something for the patient. "Working with" is a process that suggests each party is expected to participate and share responsibility for the outcomes. Contracts do not, however, stipulate roles or feeling tone, or that premature termination is forbidden.

A patient with liver failure has been on the transplant waiting list for 8 months. The patient says to the nurse, "Why is it taking so long to have the surgery? Maybe I'm meant to die for all the bad things I've done." The nurse should document the patient's comment in which section of the assessment? A. Physical B. Spiritual C. Financial D. Psychological

B. Spiritual Rationale: Stress can be evident in a person's spirituality. This patient's comment indicates questioning of one's place in the universe and consequences for wrongdoing, both of which are elements of spirituality. Stress can be related to psychologic, physical, or psychosocial well-being, but spirituality is the best answer.

A nurse wants to enhance the growth of a patient by showing positive regard. The action consistent with this wish is: A. Making rounds daily B. Staying with a tearful patient C. Administering daily medication as prescribed D. Examining personal feelings about a patient

B. Staying with a tearful patient Rationale: Staying with a crying patient offers support and shows positive regard. Administering daily medication and making rounds are tasks that could be part of an assignment and do not necessarily reflect positive regard. Examining feelings regarding a patient addresses the nurse's ability to be therapeutic.

Which situation demonstrates the use of primary care related to crisis intervention? A. Implementing suicide precautions for a patient with depression B. Teaching stress reduction techniques to a beginning student nurse C. Assessing coping strategies used by a patient who has attempted suicide D. Referring a patient with schizophrenia to a partial hospitalization program

B. Teaching stress reduction techniques to a beginning student nurse Rationale: Primary crisis intervention promotes mental health and reduces mental illness. The incorrect options are examples of secondary or tertiary intervention.

During the first interview with a parent whose child died in a car accident, the nurse feels empathic and reaches out to take the patient's hand. Select the correct analysis of the nurse's behavior. A. It shows empathy and compassion. It will encourage the patient to continue to express feelings B. The gesture is premature. The patient's cultural and individual interpretation of touch is unknown C. The patient will perceive the gesture as intrusive and overstepping boundaries D. The action is inappropriate. Patients in a psychiatric setting should not be touched

B. The gesture is premature. The patient's cultural and individual interpretation of touch is unknown Rationale: Touch has various cultural and individual interpretations. Nurses should refrain from using touch until an assessment can be made regarding the way in which the patient will perceive touch. The other options present prematurely drawn conclusions.

As a patient with mental illness is being discharged from a facility, a nurse invites the patient to the annual staff picnic. What is the best analysis of this scenario? A. The invitation facilitates dependency on the nurse B. The nurse's action blurs the boundaries of the therapeutic relationship C. The invitation is therapeutic for the patient's diversional activity deficit D. The nurse's action assists the patient's integration into community living

B. The nurse's action blurs the boundaries of the therapeutic relationship Rationale: The invitation creates a social relationship rather than a therapeutic relationship.

A nurse wants to demonstrate genuineness with a patient diagnosed with schizophrenia. The nurse should: A. Restate what the patient says B. Use congruent communication strategies C. Use self-revelation in patient interactions D. Consistently interpret the patient's behaviors

B. Use congruent communication strategies Rationale: Genuineness is a desirable characteristic involving an awareness of one's own feelings as they arise and the ability to communicate them when appropriate. The incorrect options are undesirable in a therapeutic relationship.

A patient experiences an episode of severe anxiety. Of these medications in the patient's medical record, which is most appropriate to administer as an as-needed (prn) anxiolytic medication? A. buspirone (Buspar) B. lorazepam (Ativan) C. amitriptyline (Elavil) D. desipramine (Norpramin)

B. lorazepam (Ativan) Rationale: Lorazepam is a benzodiazepine medication used to treat anxiety; it may be administered as needed. Buspirone is long acting and not useful as an as-needed drug. Amitriptyline and desipramine are tricyclic antidepressants and considered second- or third-line agents.

A patient discloses several concerns and associated feelings. If the nurse wants to seek clarification, which comment would be appropriate? A. "What are the common elements here?" B. "Tell me again about your experiences." C. "Am I correct in understanding that...?" D. "Tell me everything from the beginning."

C. "Am I correct in understanding that...?" Rationale: Asking, "Am I correct in understanding that...?" permits clarification to ensure that both the nurse and patient share mutual understanding of the communication. Asking about common elements encourages comparison rather than clarification. The remaining responses are implied questions that suggest the nurse was not listening.

As a nurse escorts a patient being discharged after treatment for major depression, the patient gives the nurse a necklace with a heart pendant and says, "Thank you for helping mend my broken heart." Which is the nurse's best response? A. "Accepting gifts violates the policies and procedures of the facility." B. "I'm glad you feel so much better now. Thank you for the beautiful necklace." C. "I'm glad I could help you, but I can't accept the gift. My reward is seeing you with a renewed sense of hope." D. "Helping people is what nursing is all about. It's rewarding to me when patients recognize how hard we work."

C. "I'm glad I could help you, but I can't accept the gift. My reward is seeing you with a renewed sense of hope." Rationale: Accepting a gift creates a social rather than a therapeutic relationship with the patient and blurs the boundaries of the relationship. A caring nurse will acknowledge the patient's gesture of appreciation, but the gift should not be accepted.

A patient who is experiencing moderate anxiety says, "I feel undone." An appropriate response for the nurse would be: A. "Why do you suppose you are feeling anxious?" B. "What would you like me to do to help you?" C. "I'm not sure I understand. Give me an example." D. "You must get your feelings under control before we can continue."

C. "I'm not sure I understand. Give me an example." Rationale: Increased anxiety results in scattered thoughts and an inability to articulate clearly. Clarification helps the patient identify his or her thoughts and feelings. Asking the patient why he or she feels anxious is nontherapeutic, and the patient will not likely have an answer. The patient may be unable to determine what he or she would like the nurse to do to help. Telling the patient to get his or her feelings under control is a directive the patient is probably unable to accomplish.

A patient says, "I'm still on restriction, but I want to attend some off-unit activities. Would you ask the doctor to change my privileges?" What is the nurse's best response? A. "Why are you asking me when you're able to speak for yourself?" B. "I will be glad to address it when I see your doctor later today." C. "That's a good topic for you to take up with your doctor." D. "Do you think you can't speak to a doctor?"

C. "That's a good topic for you to take up with your doctor." Rationale: Nurses should encourage patients to work at their optimal level of functioning. A nurse does not act for the patient unless it is necessary. Acting for a patient increases feelings of helplessness and dependency.

A nurse talks with a person whose spouse died while jogging. Which is the appropriate statement for the nurse? A. "At least your spouse did not suffer." B. "It's better to go quickly as your spouse did." C. "The loss of your spouse must be very painful for you." D. "You'll begin to feel better after you get over the shock."

C. "The loss of your spouse must be very painful for you." Rationale: The most helpful responses by others validate the bereaved person's experience of loss. Avoid banalities; they increase the individual's sense of isolation.

An adolescent asks a nurse conducting an assessment interview, "Why should I tell you anything? You'll just tell my parents whatever you find out." Select the nurse's best reply. A. "That isn't true. What you tell us is private and held in strict confidence. Your parents have no right to know." B. "Yes, your parents may find out what you say, but it is important that they know about your problems." C. "What you say about feelings is private, but some things, like suicidal thinking, must be reported to the treatment team." D. "It sounds as though you are not really ready to work on your problems and make changes."

C. "What you say about feelings is private, but some things, like suicidal thinking, must be reported to the treatment team." Rationale: The patient has a right to know that most information will be held in confidence but that certain material must be reported or shared with the treatment team, such as threats of suicide, homicide, use of illegal drugs, or issues of abuse. The first response is not strictly true. The second response will not inspire the confidence of the patient. The fourth response is confrontational.

A patient tells the nurse, "I don't think I'll ever get out of here." Select the nurse's most therapeutic response. A. "Don't talk that way. Of course you will leave here!" B. "Keep up the good work and you certainly will." C. "You don't think you're making progress?" D. "Everyone feels that way sometimes."

C. "You don't think you're making progress?" Rationale: By asking if the patient does not believe that progress has been made, the nurse is reflecting by putting into words what the patient is hinting. By making communication more explicit, issues are easier to identify and resolve. The remaining options are nontherapeutic techniques. Telling the patient not to "talk that way" is disapproving. Saying that everyone feels that way at times minimizes feelings. Telling the patient that good work will always result in success is falsely reassuring.

A patient with metastatic brain cancer says, "I'm dying, but I'm still living. I want to be in control as long as I can." Which reply shows the nurse was actively listening? A. "Our staff will do their best to help you feel comfortable." B. "Most people do not know how to help and are afraid of death." C. "Your mind and spirit are healthy, although your body is frail." D. "You want people to stop focusing on your weaknesses."

C. "Your mind and spirit are healthy, although your body is frail." Rationale: The patient is asking for acknowledgment that he or she is not totally sick; even in the terminal state, strengths and capabilities are present. The correct response provides that acknowledgment. The other responses are tangential.

A patient visiting the crisis clinic for the first time asks, "How long will I be coming here?" The nurse's reply should consider that the usual duration of crisis intervention is _____________ weeks. A. 1 to 2 B. 3 to 4 C. 4 to 6 D. 6 to 12

C. 4 to 6 Rationale: The disorganization associated with crisis is so distressing that it usually cannot be tolerated for more than 4 to 6 weeks. If the crisis is not resolved by that time, the individual usually adopts dysfunctional behaviors that reduce anxiety without solving the problem. Crisis intervention can shorten the duration.

In which situations does a nurse have a duty to intervene and report? Select all that apply. A. A peer is unable to write behavioral outcomes B. A health care provider consults the Physicians' Desk Reference C. A peer tries to provide patient care in an alcohol-impaired state D. A team member has violated the boundaries of a vulnerable patient E. A patient refuses a medication prescribed by a licensed health care provider

C. A peer tries to provide patient care in an alcohol-impaired state D. A team member has violated the boundaries of a vulnerable patient Rationale: Both instances jeopardize patient safety. The nurse must practice within the Code of Ethics for Nurses. A peer being unable to write behavioral outcomes is a concern but can be informally resolved. A health care provider consulting the Physicians' Desk Reference is acceptable practice.

To provide comprehensive care to patients, which competency is more important for a nurse who works in a community mental health center than a psychiatric nurse who works in an inpatient unit? A. Problem-solving skills B. Calm external manner C. Ability to cross service systems D. Knowledge of psychopharmacology

C. Ability to cross service systems Rationale: A community mental health nurse must be able to work with schools, corrections facilities, shelters, health care providers, and employers. The mental health nurse working in an inpatient unit needs only to be able to work within the single setting. Problem-solving skills are needed by all nurses. Nurses in both settings must have knowledge of psychopharmacology.

A nurse had cared for a terminally ill patient for over a month and always looked forward to spending time with the patient. When the patient died, the nurse experienced sadness and felt mildly depressed. Eventually, the nurse explains these feelings to a mentor. The mentor should counsel the nurse: A. About stress-reduction strategies B. To seek therapy for dysfunctional grief C. About the experience of disenfranchised grief D. To consider taking a leave of absence to pursue healing

C. About the experience of disenfranchised grief Rationale: The nurse is experiencing disenfranchised grief. Nurses often incur loss that is not openly acknowledged or publicly mourned. The loss of a patient may not be recognized or acknowledged by others; therefore the grief is solitary and uncomforted and may be difficult to resolve.

Which experiences are most likely to precipitate post-traumatic stress disorder (PTSD)? Select all that apply. A. A young adult jumps from a bridge with a bungee cord with a best friend B. An 8-year-old child watches an R-rated movie with both parents C. An adolescent is kidnapped and held for 2 years in the home of a sexual predator D. A passenger is in a bus that overturns on a sharp curve in the road, tumbling down an embankment E. An adult is trapped for 3 hours at an angle in an elevator after a portion of the supporting cable breaks

C. An adolescent is kidnapped and held for 2 years in the home of a sexual predator D. A passenger is in a bus that overturns on a sharp curve in the road, tumbling down an embankment E. An adult is trapped for 3 hours at an angle in an elevator after a portion of the supporting cable breaks Rationale: PTSD usually follows a traumatic event that is outside the range of usual experience. Examples are childhood physical abuse, torture or kidnapping, military combat, sexual assault, and natural disasters such as floods, tornados, earthquakes, and tsunamis; human disasters such as a bus or elevator accident or crime-related events such being taken hostage are additional examples. The common element in these experiences is the individual's extraordinary helplessness or powerlessness in the face of such stressors. Bungee jumping by adolescents is part of the developmental task and might be frightening but in an exhilarating way rather than a harmful way. A child may be disturbed by an R-rated movie, but the presence of the parents would modify the experience in a positive way.

A patient hurriedly tells the community mental health nurse, "Everything's a disaster! I can't concentrate. My disability check didn't come. My roommate moved out, and I can't afford the rent. My therapist is moving away. I feel like I'm coming apart." Which nursing diagnosis applies? A. Decisional conflict, related to challenges to personal values B. Spiritual distress, related to ethical implications of treatment regimen C. Anxiety, related to changes perceived as threatening to psychological equilibrium D. Impaired environmental interpretation syndrome, related to solving multiple problems affecting security needs

C. Anxiety, related to changes perceived as threatening to psychological equilibrium Rationale: Subjective and objective data obtained by the nurse suggest the patient is experiencing anxiety caused by multiple threats to security needs. Data are not present to suggest Decisional conflict, ethical conflicts around treatment causing Spiritual distress, or Impaired environmental interpretation syndrome.

A suspicious and socially isolated patient lives alone, eats one meal a day at a nearby shelter, and spends the remaining daily food allowance on cigarettes. Select the community psychiatric nurse's best initial action. A. Report the situation to the manager of the shelter B. Tell the patient, "You must stop smoking to save money." C. Assess the patient's weight; determine the foods and amounts eaten D. Seek hospitalization for the patient while a new plan is being formulated

C. Assess the patient's weight; determine the foods and amounts eaten Rationale: Assessment of biopsychosocial needs and general ability to live in the community is called for before any action is taken. Both nutritional status and income adequacy are critical assessment parameters. A patient may be able to maintain adequate nutrition while eating only one meal a day. Nurses assess before taking action. Hospitalization may not be necessary.

When alprazolam (Xanax) is prescribed for acute anxiety, health teaching should include instructions to: A. Report drowsiness B. Eat a tyramine-free diet C. Avoid alcoholic beverages D. Adjust dose and frequency based on anxiety level

C. Avoid alcoholic beverages Rationale: Drinking alcohol or taking other anxiolytic medications along with the prescribed benzodiazepine should be avoided because depressant effects of both drugs will be potentiated. Tyramine-free diets are necessary only with monoamine oxidase inhibitors (MAOIs). Drowsiness is an expected effect and needs to be reported only if it is excessive. Patients should be taught not to deviate from the prescribed dose and schedule for administration.

A soldier who served in a combat zone returned to the United States. The soldier's spouse complains to the nurse, "We had planned to start a family, but now he won't talk about it. He won't even look at children." The spouse is describing which symptom associated with post-traumatic stress disorder (PTSD)? A. Reexperiencing B. Hyperarousal C. Avoidance D. Psychosis

C. Avoidance Rationale: Physiologic reactions to reminders of the event include a persistent avoidance of the stimuli associated with the trauma; the individual avoids talking about the event or avoids activities, people, or places that arouse memories of the trauma. Avoidance is exemplified by a sense of foreshortened future and estrangement. No evidence suggests that this soldier is having a hyperarousal reaction or is reexperiencing war-related traumas. Psychosis is not evident.

Which finding indicates that a patient with moderate-to-severe anxiety has successfully lowered the anxiety level to mild? The patient: A. Asks, "What's the matter with me?" B. Stays in a room alone and paces rapidly C. Can concentrate on what the nurse is saying D. States, "I don't want anything to eat. My stomach is upset."

C. Can concentrate on what the nurse is saying Rationale: The ability to concentrate and attend to reality is increased slightly in mild anxiety and decreased in moderate-, severe-, and panic-level anxiety. Patients with high levels of anxiety often ask, "What's the matter with me?" Staying in a room alone and pacing suggest moderate anxiety. Expressing a lack of hunger is not necessarily a criterion for evaluating anxiety.

A patient's fiancé died in an automobile accident several days ago. The patient reports crying and experiencing feelings of guilt and anger. This behavior is characteristic of which stage of acute grief? A. Denial B. Reorganization C. Development of awareness D. Preoccupation with the lost object

C. Development of awareness Rationale: As denial fades, an awareness of the finality of the loss develops and is accompanied by painful feelings of loss, anger with others, and guilt for taking or not taking specific actions. Reorganization implies the movement toward healing. Denial is manifested by the inability to believe the reality of an event. Preoccupation with the lost object would involve the patient dwelling on thoughts of the deceased.

Termination of a therapeutic nurse-patient relationship with a patient has been successful when the nurse: A. Avoids upsetting the patient by shifting focus to other patients before the discharge B. Gives the patient a personal telephone number and permission to call after discharge C. Discusses with the patient changes that have happened during the relationship and evaluates the outcomes D. Offers to meet the patient for coffee and conversation three times a week after discharge

C. Discusses with the patient changes that have happened during the relationship and evaluates the outcomes Rationale: Summarizing and evaluating progress help validate the experience for the patient and the nurse and facilitate closure. Termination must be discussed; avoiding the discussion by spending little time with the patient promotes feelings of abandonment. Successful termination requires that the relationship be brought to closure without the possibility of dependency-producing ongoing contact.

A nurse works with a patient to establish outcomes. The nurse believes that one outcome suggested by the patient is not in the patient's best interest. What is the nurse's best action? A. Remain silent B. Educate the patient that the outcome is not realistic C. Explore with the patient possible consequences of the outcome D. Formulate an appropriate outcome without the patient's input

C. Explore with the patient possible consequences of the outcome Rationale: The nurse should not impose outcomes on the patient; however, the nurse has a responsibility to help the patient evaluate what is in his or her best interest. Exploring possible consequences is an acceptable approach.

A patient has the nursing diagnosis: Anxiety, related to __________, as evidenced by an inability to control compulsive cleaning. Which phrase correctly completes the etiologic portion of the diagnosis? A. Ensuring the health of household members B. Attempting to avoid interactions with others C. Having persistent thoughts about bacteria, germs, and dirt D. Needing approval for cleanliness from friends and family

C. Having persistent thoughts about bacteria, germs, and dirt Rationale: Many compulsive rituals accompany obsessive thoughts. The patient uses these rituals to relief anxiety. Unfortunately, the anxiety relief is short lived, and the patient must frequently repeat the ritual. The other options are unrelated to the dynamics of compulsive behavior.

A person with a fear of heights drives across a high bridge. Which structure will stimulate a response from the autonomic nervous system? A. Thalamus B. Parietal lobe C. Hypothalamus D. Pituitary gland

C. Hypothalamus Rationale: The individual will find this experience stressful. The hypothalamus functions as the command-and-control center when receiving stressful signals. The hypothalamus responds to signals of stress by engaging the autonomic nervous system. The parietal lobe is responsible for the interpretation of other sensations. The thalamus processes messages associated with pain and wakefulness. The pituitary gland may be involved in other aspects of the person's response but would not stimulate the autonomic nervous system.

A patient tells a nurse, "My new friend is the most perfect person one could imagine—kind, considerate, and good looking. I can't find a single flaw." This patient is demonstrating: A. Denial B. Projection C. Idealization D. Compensation

C. Idealization Rationale: Idealization is an unconscious process that occurs when an individual attributes exaggerated positive qualities to another. Denial is an unconscious process that calls for the nurse to ignore the existence of the situation. Projection operates unconsciously and results in blaming behavior. Compensation results in the nurse unconsciously attempting to make up for a perceived weakness by emphasizing a strong point.

An adult tells the nurse, "I can't take it anymore! Last year my husband had an affair and now we don't communicate. Three months ago, I found a lump in my breast. Yesterday my daughter told me she's quitting college and moving in with her boyfriend." What is the priority nursing diagnosis? A. Fear, related to impending surgery B. Deficient knowledge, related to breast lesion C. Ineffective coping, related to perceived loss of daughter D. Impaired verbal communication, related to spousal estrangement

C. Ineffective coping, related to perceived loss of daughter Rationale: This nursing diagnosis is the priority because it reflects the precipitating event associated with the patient's crisis. Data are not present to make the other diagnoses of Deficient knowledge, Fear, or Impaired verbal communication.

An adult who was widowed 18 months ago says, "I can now remember good times we shared without getting upset. Sometimes I even think about the disappointments. I've become accustomed to sleeping in our bed alone." The work of mourning: A. Is beginning B. Is progressing abnormally C. Is at or near completion D. Has not begun

C. Is at or near completion Rationale: The work of mourning has been successfully completed when the bereaved can remember both the positive and negative memories about the deceased and when the task of restructuring the relationship with the deceased is completed.

After celebrating a 40th birthday, an individual becomes concerned with the loss of youthful appearance. What type of crisis has occurred? A. Reactive B. Situational C. Maturational D. Adventitious

C. Maturational Rationale: Maturational crises occur when a person arrives at a new stage of development and finds that old coping styles are ineffective but has not yet developed new strategies. Situational crises arise from sources external to the individual, such as divorce and job loss. No classification called reactive crisis exists. Adventitious crises occur when disasters such as natural disasters (e.g., floods, hurricanes), war, or violent crimes disrupt coping styles.

When a new patient is hospitalized, a nurse takes the patient on a tour, explains the rules of the unit, and discusses the daily schedule. The nurse is engaged in: A. Counseling B. Health teaching C. Milieu management D. Psychobiologic intervention

C. Milieu management Rationale: Milieu management provides a therapeutic environment in which the patient can feel comfortable and safe while engaging in activities that meet the patient's physical and mental health needs. Counseling refers to activities designed to promote problem solving and enhanced coping and includes interviewing, crisis intervention, stress management, and conflict resolution. Health teaching involves identifying health educational needs and giving information about these needs. Psychobiologic interventions involve medication administration and monitoring response to medications.

Which agency provides coordination in the event of a terrorist attack? A. U.S. Food and Drug Administration (FDA) B. Environmental Protection Agency (EPA) C. National Incident Management System (NIMS) D. Federal Emergency Management Agency (FEMA)

C. National Incident Management System (NIMS) Rationale: The NIMS provides a systematic approach to guide departments and agencies at all levels of government, nongovernmental organizations, and the private sector during disaster situations.

Which actions by a nurse contribute to protecting the rights of patients who are terminally ill? Select all that apply. A. Maintain hope for a positive prognosis B. Hug the patient when sadness is expressed C. Offer choices that promote personal control D. Provide interventions that convey respect E. Support the patient's quest for spiritual growth

C. Offer choices that promote personal control D. Provide interventions that convey respect E. Support the patient's quest for spiritual growth Rationale: The answers support the rights of the individual who is dying. Touch should be nurturing but may leave the patient uncomfortable and confused if inappropriate. Acting on false information robs a patient of the opportunity for honest dialog and places barriers to achieving end-of-life developmental opportunities.

Documentation in a patient's chart shows, "Throughout a 5-minute interaction, patient fidgeted and tapped left foot, periodically covered face with hands, and looked under chair while stating, 'I enjoy spending time with you.'" Which analysis is most accurate? A. Patient is giving positive feedback about the nurse's communication techniques B. Nurse is viewing the patient's behavior through a cultural filter C. Patient's verbal and nonverbal messages are incongruent D. Patient is demonstrating psychotic behaviors

C. Patient's verbal and nonverbal messages are incongruent Rationale: When a verbal message is not reinforced with nonverbal behavior, the message is confusing and incongruent. Some clinicians call it a "mixed message." It is inaccurate to say that the patient is giving positive feedback about the nurse's communication techniques. The concept of a cultural filter is not relevant to the situation; a cultural filter determines what a person will pay attention to and what he or she will ignore. Data are insufficient to draw the conclusion that the patient is demonstrating psychotic behaviors.

A new staff nurse completes orientation to the psychiatric unit. This nurse will expect to ask an advanced practice nurse to perform which action for patients? A. Perform mental health assessment interviews B. Establish therapeutic relationships C. Prescribe psychotropic medications D. Individualize nursing care plans

C. Prescribe psychotropic medications Rationale: Prescriptive privileges are granted to Master's-prepared nurse practitioners who have taken special courses on prescribing medications. The nurse prepared at the basic level performs mental health assessments, establishes relationships, and provides individualized care planning.

Two staff nurses applied for a charge nurse position. After the promotion was announced, the nurse who was not promoted said, "The nurse manager had a headache the day I was interviewed." Which defense mechanism is evident? A. Introjection B. Conversion C. Projection D. Splitting

C. Projection Rationale: Projection is the hallmark of blaming, scapegoating, thinking prejudicially, and stigmatizing others. Conversion involves the unconscious transformation of anxiety into a physical symptom. Introjection involves intense, unconscious identification with another person. Splitting is the inability to integrate the positive and negative qualities of oneself or others into a cohesive image.

For a patient experiencing panic, which nursing intervention should be first? A. Teach relaxation techniques. B. Administer an anxiolytic medication C. Provide calm, brief, directive communication D. Gather a show of force in preparation for gaining physical control

C. Provide calm, brief, directive communication Rationale: Calm, brief, directive verbal interaction can help the patient gain control of the overwhelming feelings and impulses related to anxiety. Patients experiencing panic-level anxiety are unable to focus on reality; thus learning relaxation techniques is virtually impossible. Administering an anxiolytic medication should be considered if providing calm, brief, directive communication is ineffective. Although the patient is disorganized, violence may not be imminent, ruling out the intervention of preparing for physical control until other less-restrictive measures are proven ineffective.

Which issues should a nurse address during the first interview with a patient with a psychiatric disorder? A. Trust, congruence, attitudes, and boundaries B. Goals, resistance, unconscious motivations, and diversion C. Relationship parameters, the contract, confidentiality, and termination D. Transference, countertransference, intimacy, and developing resources

C. Relationship parameters, the contract, confidentiality, and termination Rationale: Relationship parameters, the contract, confidentiality, and termination are issues that should be considered during the orientation phase of the relationship. The remaining options are issues that are dealt with later.

Children of a widowed parent confer with the nurse; their surviving parent repeatedly relates the details of finding the deceased parent not breathing, performing cardiopulmonary resuscitation (CPR), going to the hospital by ambulance, and seeing the pronouncement of death. The family asks, "What can we do?" The nurse should counsel the family: A. They should share their feelings with the surviving parent and ask for the retelling to stop B. Retelling the story should be limited to once daily to avoid unnecessary stimulation C. Retelling memories is to be expected as part of the aging process D. Repeating the story is a helpful and a necessary part of grieving

C. Retelling memories is to be expected as part of the aging process Rationale: Nurses are encouraged to tell bereaved patients that telling the personal story of loss as many times as needed is acceptable and healthy; repetition is a helpful and necessary part of grieving.

A newly admitted patient with major depression has lost 20 pounds over the past month and has suicidal ideation. The patient has taken an antidepressant medication for 1 week without remission of symptoms. Select the priority nursing diagnosis. A. Imbalanced nutrition: Less than body requirements B. Chronic low self-esteem C. Risk for suicide D. Hopelessness

C. Risk for suicide Rationale: Risk for suicide is the priority diagnosis when the patient has both suicidal ideation and a plan to carry out the suicidal intent. Imbalanced nutrition, Hopelessness, and Chronic low self-esteem may be applicable nursing diagnoses, but these problems do not affect patient safety as urgently as a suicide attempt.

A person has minor physical injuries after an automobile accident. The person is unable to focus and says, "I feel like something awful is going to happen." This person has nausea, dizziness, tachycardia, and hyperventilation. What is this person's level of anxiety? A. Mild B. Moderate C. Severe D. Panic

C. Severe Rationale: The person whose anxiety is severe is unable to solve problems and may have a poor grasp of what is happening in the environment. Somatic symptoms such as those described are usually present. The individual with mild anxiety is only mildly uncomfortable and may even find his or her performance enhanced. The individual with moderate anxiety grasps less information about a situation and has some difficulty with problem solving. The individual in a panic level of anxiety demonstrates significantly disturbed behavior and may lose touch with reality.

Which principle should guide the nurse in determining the extent of silence to use during patient interview sessions? A. Nurses are responsible for breaking silences B. Patients withdraw if silences are prolonged C. Silence provides meaningful moments for reflection D. Silence helps patients know that what they have said is understood

C. Silence provides meaningful moments for reflection Rationale: Silence can be helpful to both participants by giving each an opportunity to contemplate what has transpired, weigh alternatives, and formulate ideas. A nurse breaking silences is not a principle related to silences. Saying that patients withdraw during long silences or that silence helps patients know that they are understood are both inaccurate statements. Feedback helps patients know they have been understood

A woman says, "I can't take it anymore! Last year my husband had an affair and now we don't communicate. Three months ago, I found a lump in my breast. Yesterday my daughter said she's quitting college." What type of crisis is this person experiencing? A. Maturational B. Adventitious C. Situational D. Recurring

C. Situational Rationale: A situational crisis arises from an external source and involves a loss of self-concept or self-esteem. An adventitious crisis is a crisis of disaster, such as a natural disaster or crime of violence. Maturational crisis occurs as an individual arrives at a new stage of development, when old coping styles may be ineffective. No classification of recurring crisis exists.

A patient tells the nurse, "I don't go to restaurants because people might laugh at the way I eat or I could spill food and be laughed at." The nurse assesses this behavior as consistent with: A. Acrophobia B. Agoraphobia C. Social phobia D. Posttraumatic stress disorder

C. Social phobia Rationale: The fear of a potentially embarrassing situation represents a social phobia. Acrophobia is the fear of heights. Agoraphobia is the fear of a place in the environment. Posttraumatic stress disorder is associated with a major traumatic event.

A nurse volunteers for a committee that must revise the hospital policies and procedures for suicide precautions. Which resources would provide the best guidance? Select all that apply. A. Diagnostic and Statistical Manual of Mental Disorders (fourth edition, text revision) (DSM-IV-TR) B. State's nurse practice act C. State and federal regulations that govern hospitals D. Summary of common practices of several local hospitals E. American Nurses Association Scope and Standards of Practice for Psychiatric-Mental Health Nursing Practice

C. State and federal regulations that govern hospitals E. American Nurses Association Scope and Standards of Practice for Psychiatric-Mental Health Nursing Practice Rationale: Regulations regarding hospitals provide information about the minimal standard. The American Nurses Association (ANA) national standards focus on elevating practice by setting high standards for nursing practice. The DSM-IV-TR and the state's nurse practice act would not provide relevant information. A summary of common practices of several local hospitals cannot be guaranteed to be helpful because the customs may or may not comply with laws or best practices.

A person with a fear of heights drives across a high bridge. Which division of the autonomic nervous system is stimulated in response to this experience? A. Limbic system B. Peripheral nervous system C. Sympathetic nervous system D. Parasympathetic nervous system

C. Sympathetic nervous system Rationale: The autonomic nervous system is made up of the sympathetic (fight or flight response) and parasympathetic (relaxation response) nervous systems. In times of stress the sympathetic nervous system is stimulated. A person fearful of heights would experience stress associated with the experience of driving across a high bridge. The peripheral nervous system responds to messages from the sympathetic nervous system. The limbic system processes emotional responses but is not specifically part of the autonomic nervous system.

After several therapeutic encounters with a patient who recently attempted suicide, which occurrence should cause the nurse to consider the possibility of countertransference? A. The patient's reactions toward the nurse seem realistic and appropriate. B. The patient states, "Talking to you feels like talking to my parents." C. The nurse feels oddly happy when the patient's mood begins to lift D. The nurse develops a trusting relationship with the patient

C. The nurse feels oddly happy when the patient's mood begins to lift Rationale: Strong positive or negative reactions toward a patient or an over-identification with a patient signals possible countertransference. Nurses must carefully monitor their own feelings and reactions to detect countertransference and then seek supervision. Realistic and appropriate reactions from a patient toward a nurse are desirable. One incorrect response suggests transference. A trusting relationship with the patient is desirable.

At the last contracted visit in the crisis intervention clinic, an adult says, "I've emerged from this a stronger person. You helped me feel like my life is back in balance." The nurse responds, "I think it would be worthwhile to have two more sessions to explore why your reactions were so intense." Which analysis applies? A. The patient is experiencing transference B. The patient demonstrates a need for continuing support C. The nurse is having difficulty terminating the relationship D. The nurse is empathizing with the patient's feelings of dependency

C. The nurse is having difficulty terminating the relationship Rationale: The nurse's remark is clearly an invitation to work on other problems and prolong contact with the patient. The focus of crisis intervention is on the problem that precipitated the crisis, not other issues. The scenario does not describe transference. The patient's need for continuing support is not demonstrated in the scenario. The scenario does not describe dependency needs.

A patient should be considered for involuntary commitment for psychiatric care when he or she: A. Is noncompliant with the treatment regimen B. Sold sells and distributes illegal drugs C. Threatens to harm self and others D. Fraudulently files for bankruptcy

C. Threatens to harm self and others Rationale: Involuntary commitment protects patients who are dangerous to themselves or others and cannot care for their own basic needs. Involuntary commitment also protects other individuals in society. The behaviors described in the other options are not sufficient to require involuntary hospitalization.

Which individual with a mental illness may need emergency or involuntary hospitalization for mental illness? The individual who: A. Resumes using heroin while still taking methadone B. Reports hearing angels playing harps during thunderstorms C. Throws a heavy plate at a waiter at the direction of command hallucinations D. Does not show up for an outpatient appointment with the mental health nurse

C. Throws a heavy plate at a waiter at the direction of command hallucinations Rationale: Throwing a heavy plate is likely to harm the waiter and is evidence of being dangerous to others. This behavior meets the criteria for emergency or involuntary hospitalization for mental illness. The behaviors in the other options evidence mental illness but not dangerousness.

A nurse assesses the health status of soldiers returning from Afghanistan. Screening will be a priority for signs and symptoms of which health problems? Select all that apply. A. Schizophrenia B. Eating disorder C. Traumatic brain injury D. Seasonal affective disorder E. Posttraumatic stress disorder

C. Traumatic brain injury E. Posttraumatic stress disorder Rationale: TBI and PTSD each occur in approximately 20% of soldiers returning from Afghanistan. Some soldiers have both problems. The incidence of disorders identified in the distracters would be expected to parallel the general population.

What information is conveyed by nursing diagnoses? Select all that apply. A. Medical judgments about the disorder B. Goals and outcomes for the plan of care C. Unmet patient needs currently present D. Supporting data that validate the diagnoses E. Probable causes that will be targets for nursing interventions

C. Unmet patient needs currently present D. Supporting data that validate the diagnoses E. Probable causes that will be targets for nursing interventions Rationale: Nursing diagnoses focus on phenomena of concern to nurses rather than on medical diagnoses.

Two hospitalized patients fight when they are in the same room. During a team meeting, a nurse asserts that safety is of paramount importance and therefore the treatment plans should call for both patients to be secluded to prevent them from injuring each other. This assertion: A. Reveals that the nurse values the principle of justice B. Reinforces the autonomy of the two patients C. Violates the civil rights of the two patients D. Represents the intentional tort of battery

C. Violates the civil rights of the two patients Rationale: Patients have a right to treatment in the least restrictive setting. Less restrictive measures should be tried first. Unnecessary seclusion may result in a charge of false imprisonment. Seclusion removes the patient's autonomy. The principle by which the nurse is motivated is beneficence, not justice. The tort represented is false imprisonment, not battery.

During which phase of the nurse-patient relationship can the nurse anticipate that identified patient issues will be explored and resolved? A. Preorientation B. Orientation C. Working D. Termination

C. Working Rationale: During the working phase, the nurse strives to assist the patient in making connections among dysfunctional behaviors, thinking, and emotions and offers support while alternative coping behaviors are tried.

An adult seeks counseling after the spouse is murdered. The adult angrily says, "I hate the beast that did this. It has ruined my life. During the trial, I don't know what I'll do if the jury doesn't return a guilty verdict." What is the nurse's highest priority response? A. "Would you like to talk to a psychiatrist about some medication to help you cope during the trial?" B. "What resources do you need to help you cope with this situation?" C. "Do you have enough support from your family and friends?" D. "Are you having thoughts of hurting yourself or others?"

D. "Are you having thoughts of hurting yourself or others?" Rationale: The highest nursing priority is safety. The nurse should assess suicidal and homicidal potentials. The incorrect options are important but not the highest priority.

A patient says to the nurse, "I dreamed I was stoned. When I woke up, I felt emotionally drained, as though I hadn't rested well." Which comment would be appropriate if the nurse seeks clarification? A. "It sounds as though you were uncomfortable with the content of your dream." B. "I understand what you're saying. Bad dreams leave me feeling tired, too." C. "So, all in all, you feel as though you had a rather poor night's sleep?" D. "Can you give me an example of what you mean by 'stoned'?"

D. "Can you give me an example of what you mean by 'stoned'?" Rationale: The technique of clarification is therapeutic and helps the nurse examine the meaning of the patient's statement. Asking for a definition of "stoned" directly asks for clarification. Restating that the patient is uncomfortable with the dream's content is parroting, a nontherapeutic technique. The other responses fail to clarify the meaning of the patient's comment.

Which assessment question would be most appropriate to ask a patient who has possible generalized anxiety disorder? A. "Have you been a victim of a crime or seen someone badly injured or killed?" B. "Do you feel especially uncomfortable in social situations involving people?" C. "Do you repeatedly do certain things over and over again?" D. "Do you find it difficult to control your worrying?"

D. "Do you find it difficult to control your worrying?" Rationale: Patients with generalized anxiety disorder frequently engage in excessive worrying. They are less likely to engage in ritualistic behavior, fear social situations, or have been involved in a highly traumatic event.

An adolescent hospitalized after a violent physical outburst tells the nurse, "I'm going to kill my father, but you can't tell anyone." Select the nurse's best response. A. "You're right. Federal law requires me to keep that information private." B. "Those kinds of thoughts will make your hospitalization longer." C. "You really should share this thought with your psychiatrist." D. "I am obligated to share information with the treatment team."

D. "I am obligated to share information with the treatment team." Rationale: Breach of nurse-patient confidentiality does not pose a legal dilemma for nurses in these circumstances because a team approach to the delivery of psychiatric care presumes communication of patient information to other staff members to develop treatment plans and outcome criteria. The patient should know that the team may have to warn the father of the risk for harm.

Which statement made by a patient during an initial assessment interview should serve as the priority focus for the plan of care? A. "I can always trust my family." B. "It seems like I always have bad luck." C. "You never know who will turn against you." D. "I hear evil voices that tell me to do bad things."

D. "I hear evil voices that tell me to do bad things." Rationale: The statement regarding evil voices tells the nurse that the patient is experiencing auditory hallucinations. The other statements are vague and do not clearly identify the patient's chief symptom.

Which remark by a patient indicates passage from the orientation phase to the working phase of a nurse-patient relationship? A. "I don't have any problems." B. "It is so difficult for me to talk about my problems." C. "I don't know how talking about things twice a week can help." D. "I want to find a way to deal with my anger without becoming violent."

D. "I want to find a way to deal with my anger without becoming violent." Rationale: Thinking about a more constructive approach to dealing with anger indicates a readiness to make a behavioral change. Behavioral change is associated with the working phase of the relationship. Denial is often seen in the orientation phase. It is common early in the relationship, before rapport and trust are firmly established, for a patient to express difficulty in talking about problems. Stating skepticism about the effectiveness of the nurse-patient relationship is more typically a reaction during the orientation phase.

A nurse interacts with a newly hospitalized patient. Select the nurse's comment that applies the communication technique of "offering self." A. "I've also had traumatic life experiences. Maybe it would help if I told you about them." B. "Why do you think you had so much difficulty adjusting to this change in your life?" C. "I hope you will feel better after getting accustomed to how this unit operates." D. "I'd like to sit with you for a while to help you get comfortable talking to me."

D. "I'd like to sit with you for a while to help you get comfortable talking to me." Rationale: "Offering self" is a technique that should be used in the orientation phase of the nurse-patient relationship. Sitting with the patient, an example of "offering self," helps build trust and conveys that the nurse cares about the patient. Two incorrect responses are ineffective and nontherapeutic. The other incorrect response is therapeutic but an example of "offering hope."

A soldier returned home last year after deployment to a war zone. The soldier's spouse complains, "We were going to start a family but now he won't talk about it. He will not look at children. I wonder if we're going to make it as a couple." Select the nurse's best response. A. "Posttraumatic stress disorder often changes a person's sexual functioning." B. "I encourage you to continue to participate in social activities where children are present." C. "Have you talked with your spouse about these reactions? Sometimes we just need to confront behavior." D. "Posttraumatic stress disorder often strains relationships. Here are some community resources for help and support."

D. "Posttraumatic stress disorder often strains relationships. Here are some community resources for help and support." Rationale: PTSD precipitates changes that often lead to divorce. Providing support to both the veteran and spouse is important. Confrontation will not be effective. Although providing information is important, ongoing support is more effective.

During an interview, a patient attempts to shift the focus from self to the nurse by asking personal questions. The nurse should respond by saying: A. "You've turned the tables on me." B. "Nurses direct the interviews with patients." C. "Do not ask questions about my personal life." D. "The time we spend together is to discuss your concerns."

D. "The time we spend together is to discuss your concerns." Rationale: When a patient tries to focus on the nurse, the nurse should refocus the discussion back onto the patient. Telling the patient that interview time should be used to discuss patient concerns refocuses discussion in a neutral way. Telling patients not to ask about the nurse's personal life shows indignation. Saying that nurses prefer to direct the interview reflects superiority. Saying "You've turned the tables on me" states the fact but does not refocus the interview.

A family asks the nurse, "What can we say when our terminally ill family member mentions death is coming soon?" Which response could the nurse suggest? A. "We think you will be around for a long time." B. "We don't want you to give up trying to get well." C. "We don't think we're ready to talk about this yet." D. "We feel so sad when we think of life without you."

D. "We feel so sad when we think of life without you." Rationale: This response is emotionally honest. It will allow the family opportunities to express emotions and further resolve issues in the relationship and explore end-of-life developmental opportunities. The incorrect options are evasive.

A patient comes to the clinic with superficial cuts on the left wrist. The patient is pacing and sobbing. After a few minutes with the nurse, the patient is calmer. What should the nurse ask to determine the patient's perception of the precipitating event? A. "Tell me why you were crying." B. "How did your wrist get injured?" C. "How can I help you feel more comfortable?" D. "What was happening just before you started to feel this way?"

D. "What was happening just before you started to feel this way?" Rationale: A clear definition of the immediate problem provides the best opportunity to find a solution. Asking about recent upsetting events permits the assessment of the precipitating event. Asking "why" questions is a poor communication technique.

During the first interview, a nurse notices that the patient does not make eye contact. The nurse can correctly analyze that: A. The patient is not truthful. B. The patient is feeling sad. C. The patient has a poor self-concept. D. More information is needed to draw conclusion.

D. More information is needed to draw conclusion. Rationale: The data are insufficient to draw a conclusion. The nurse must continue to assess.

Which scenario is an example of a tort? A. The primary nurse does not complete the plan of care for a patient within 24 hours of the patient's admission B. An advanced practice nurse recommends that a patient who is dangerous to self and others be voluntarily hospitalized C. A patient's admission status is changed from involuntary to voluntary after the patient's hallucinations subside D. A nurse gives an as-needed dose of an antipsychotic drug to a patient to prevent violent acting-out because a unit is short staffed

D. A nurse gives an as-needed dose of an antipsychotic drug to a patient to prevent violent acting-out because a unit is short staffed Rationale: A tort is a civil wrong against a person that violates his or her rights. Giving unnecessary medication for the convenience of staff members controls behavior in a manner similar to secluding a patient; thus false imprisonment is a possible charge. The other options do not exemplify torts.

A nurse assessing a new patient asks, "What is meant by the saying, 'You can't judge a book by its cover'?" Which aspect of cognition is the nurse assessing? A. Mood B. Attention C. Orientation D. Abstraction

D. Abstraction Rationale: Patient interpretation of proverbial statements gives assessment information regarding the patient's ability to abstract, which is an aspect of cognition. Mood, orientation, and attention span are assessed in other ways.

A patient with severe anxiety suddenly begins running and shouting, "I'm going to explode!" The nurse should: A. Ask, "I'm not sure what you mean. Give me an example." B. Chase after the patient, and give instructions to stop running C. Capture the patient in a basket-hold to increase feelings of control D. Assemble several staff members and state, "We will help you regain control."

D. Assemble several staff members and state, "We will help you regain control." Rationale: The safety needs of the patient and other patients are a priority. The patient is less likely to cause self-harm or hurt others when several staff members take responsibility for providing limits. The explanation given to the patient should be simple and neutral. Simply being told that others can help provide the control that has been lost may be sufficient to help the patient regain control. Running after the patient will increase the patient's anxiety. More than one staff member is needed to provide physical limits if they become necessary. Asking the patient to give an example is futile; a patient in panic processes information poorly.

A community psychiatric nurse facilitates medication compliance for a patient by having the health care provider prescribe depot medications by injection every 3 weeks at the clinic. For this plan to be successful, which factor will be of critical importance? A. Attitude of significant others toward the patient B. Nutritional services in the patient's neighborhood C. Level of trust between the patient and the nurse D. Availability of transportation to the clinic

D. Availability of transportation to the clinic Rationale: The ability of the patient to get to the clinic is of paramount importance to the success of the plan. The depot medication relieves the patient of the necessity to take medication daily, but if he or she does not receive the injection at 3-week intervals, noncompliance will again be the issue. Attitude toward the patient, trusting relationships, and nutrition are important but not fundamental to this particular problem.

A Puerto Rican-American patient uses dramatic body language when describing emotional discomfort. Which analysis most likely explains the patient's behavior? The patient: A. Has a histrionic personality disorder B. Believes dramatic body language is sexually appealing C. Wishes to impress staff with the degree of emotional pain D. Belongs to a culture in which dramatic body language is the norm

D. Belongs to a culture in which dramatic body language is the norm Rationale: Members of Hispanic-American subcultures tend to use high affect and dramatic body language as they communicate. The other options are more remote possibilities.

Nursing behaviors associated with the implementation phase of the nursing process are concerned with: A. Participating in the mutual identification of patient outcomes B. Gathering accurate and sufficient patient-centered data C. Comparing patient responses and expected outcomes D. Carrying out interventions and coordinating care

D. Carrying out interventions and coordinating care Rationale: Nursing behaviors relating to implementation include using available resources, performing interventions, finding alternatives when necessary, and coordinating care with other team members.

A woman says, "I can't take it anymore! Last year my husband had an affair and now we don't communicate. Three months ago, I found a lump in my breast. Yesterday my daughter said she's quitting college." What is the nurse's priority assessment? A. Identifying measures useful to help improve the couple's communication B. Discussing the patient's feelings about the possibility of having a mastectomy C. Determining whether the husband is still engaged in an extramarital affair D. Clarifying what the patient means by, "I can't take it anymore!"

D. Clarifying what the patient means by, "I can't take it anymore!" Rationale: During crisis intervention, the priority concern is patient safety. This question helps assess personal coping skills. The other options are incorrect because the focus of crisis intervention is on the event that occurred immediately before the patient sought help.

A patient checks and rechecks electrical cords related to an obsessive thought that the house may burn down. The nurse and patient explore the likelihood of an actual fire. The patient states that a house fire is not likely. This counseling demonstrates the principles of: A. Flooding B. Desensitization C. Relaxation technique D. Cognitive restructuring

D. Cognitive restructuring Rationale: Cognitive restructuring involves the patient in testing automatic thoughts and drawing new conclusions. Desensitization involves a graduated exposure to a feared object. Relaxation training teaches the patient to produce the opposite of the stress response. Flooding exposes the patient to a large amount of undesirable stimuli in an effort to extinguish the anxiety response.

A person who feels unattractive repeatedly says, "Although I'm not beautiful, I am smart." This is an example of: A. Repression B. Devaluation C. Identification D. Compensation

D. Compensation Rationale: Compensation is an unconscious process that allows an individual to make up for deficits in one area by excelling in another area to raise self-esteem. Repression unconsciously puts an idea, event, or feeling out of awareness. Identification is an unconscious mechanism calling for an imitation of the mannerisms or behaviors of another. Devaluation occurs when the individual attributes negative qualities to self or to others.

At what point in an assessment interview would a nurse ask, "How does your faith help you in stressful situations?" During the assessment of: A. Childhood growth and development B. Substance use and abuse C. Educational background D. Coping strategies

D. Coping strategies Rationale: When discussing coping strategies, the nurse might ask what the patient does when upset, what usually relieves stress, and to whom the patient goes to talk about problems. The question regarding whether the patient's faith helps deal with stress fits well here. It would seem out of place if introduced during exploration of the other topics.

A woman says, "I can't take it anymore! Last year my husband had an affair and now we don't communicate. Three months ago, I found a lump in my breast. Yesterday my daughter said she's quitting college and moving in with her boyfriend." Which issue should be the focus for crisis intervention? A. Possible mastectomy B. Disordered family communication C. Effects of the husband's infidelity D. Coping with the reaction to the daughter's events

D. Coping with the reaction to the daughter's events Rationale: The focus of crisis intervention is on the most recent problem—"the straw that broke the camel's back." The patient has coped with the breast lesion, the husband's infidelity, and the disordered communication. Disequilibrium occurs only with the introduction of the daughter leaving college and moving.

A widow grieving her husband's sudden death tells the nurse, "I'm not feeling well. Yesterday, I saw my husband walk through the door, stop, and smile at me. Then he just faded away." Which is the nurse's most appropriate action? A. Assess for recent substance use B. Suggest a referral to the mental health clinic C. Arrange for an evaluation for antidepressant medication D. Counsel the widow that visualizations are a normal part of grieving

D. Counsel the widow that visualizations are a normal part of grieving Rationale: Grieving patients often dream about, visualize, think about, or search for the lost loved one. The patient should be told that this is considered a normal phenomenon and not a sign of mental illness. Visualization does not suggest substance use or mental illness in this case.

A patient is undergoing diagnostic tests. The patient says, "Nothing is wrong with me except a stubborn chest cold." The spouse reports that the patient smokes, coughs daily, has lost 15 pounds, and is easily fatigued. Which defense mechanism is the patient using? A. Displacement B. Regression C. Projection D. Denial

D. Denial Rationale: Denial is an unconscious blocking of threatening or painful information or feelings. Regression involves using behaviors appropriate at an earlier stage of psychosexual development. Displacement shifts feelings to a more neutral person or object. Projection attributes one's own unacceptable thoughts or feelings to another.

A troubled adolescent opened fire in a high school cafeteria, fatally shooting three people and injuring many others. Hundreds of parents come to the high school after hearing the news reports. After the police arrest the shooter, which action should occur next? A. Ask the police to encircle the school campus with yellow tape to prevent parents from entering B. Announce over the loudspeakers, "The campus is now secure. Please return to your classrooms." C. Require parents to pass through metal detectors and then allow them to look for their children in the school D. Designate zones according to the alphabet, and direct students to the zones based on their surnames to facilitate reuniting them with their parents

D. Designate zones according to the alphabet, and direct students to the zones based on their surnames to facilitate reuniting them with their parents Rationale: Chaos is likely among students and desperate parents. A directive approach is best. Once the scene is secure, creative solutions are needed. Creating zones by letters of the alphabet helps anxious parents and their children to unite. Preventing parents from uniting with their children would further incite the situation.

A patient says, "People should be allowed to commit suicide without interference from others." A nurse replies, "You're wrong. Nothing is bad enough to justify death." What is the best analysis of this interchange? A. The patient is correct B. The nurse is correct C. Neither person is totally correct D. Differing values are reflected in the two statements

D. Differing values are reflected in the two statements Rationale: Values guide beliefs and actions. The individuals stating their positions place different values on life and autonomy. Nurses must be aware of their own values and be sensitive to the values of others.

A nurse caring for a withdrawn, suspicious patient recognizes the development of feelings of anger toward the patient. The nurse should: A. Suppress the angry feelings B. Express the anger openly and directly with the patient C. Tell the nurse manager to assign the patient to another nurse D. Discuss the anger with a clinician during a supervisory session

D. Discuss the anger with a clinician during a supervisory session Rationale: The nurse is accountable for the relationship. Objectivity is threatened by strong positive or negative feelings toward a patient. Supervision is necessary to work through a countertransference of feelings.

Which behavior shows that a nurse values autonomy? The nurse: A. Sets limits on a patient's romantic overtures toward the nurse B. Suggests one-on-one supervision for a patient who is suicidal C. Informs a patient that the spouse will not be in during visiting hours D. Discusses available alternatives and helps the patient weigh the consequences

D. Discusses available alternatives and helps the patient weigh the consequences Rationale: A high level of valuing is acting on one's belief. Autonomy is supported when the nurse helps a patient weigh alternatives and their consequences before the patient makes a decision. Autonomy or self-determination is not the issue in any of the other behaviors.

Which health care worker should be referred to critical incident stress debriefing? A. Nurse who works at an oncology clinic where patients receive chemotherapy B. Case manager whose patients are seriously mentally ill and are being cared for at home C. Health care employee who worked 8 hours at the information desk of an intensive care unit D. Emergency medical technician (EMT) who treated victims of a car bombing at a department store

D. Emergency medical technician (EMT) who treated victims of a car bombing at a department store Rationale: Although each of the individuals mentioned experiences job-related stress on a daily basis, the person most in need of critical incident stress debriefing is the EMT, who experienced an adventitious crisis event by responding to a bombing and provided care to victims of trauma.

A psychiatric nurse best implements the ethical principle of autonomy when he or she: A. Intervenes when a self-mutilating patient attempts to harm self B. Stays with a patient who is demonstrating a high level of anxiety C. Suggests that two patients who are fighting be restricted to the unit D. Explores alternative solutions with a patient, who then makes a choice

D. Explores alternative solutions with a patient, who then makes a choice Rationale: Autonomy is the right to self-determination, that is, to make one's own decisions. When the nurse explores alternatives with the patient, the patient is better equipped to make an informed, autonomous decision. Staying with a highly anxious patient or intervening with a self-mutilating patient demonstrates beneficence and fidelity. Suggesting that two fighting patients be restricted to the unit demonstrates the principles of fidelity and justice.

A soldier returned 3 months ago from Afghanistan and was diagnosed with post-traumatic stress disorder (PTSD). Which social event would most likely be disturbing for this soldier? A. Halloween festival with neighborhood children B. Singing carols around a Christmas tree C. Family outing to the seashore D. Fireworks display on July 4th

D. Fireworks display on July 4th Rationale: The exploding noises associated with fireworks are most likely to provoke exaggerated responses for this soldier. The distracters are not associated with offensive sounds.

After the death of a spouse, an adult repeatedly says, "I should have made him go to the doctor when he said he didn't feel well." This individual is experiencing: A. Preoccupation with the image of the deceased B. Sensations of somatic distress C. Anger D. Guilt

D. Guilt Rationale: Guilt is evident by the bereaved person's self-reproach. Preoccupation refers to dwelling on images of the deceased. Somatic distress would involve bodily symptoms. Anger is not evident from data given in this scenario.

A nurse cares for an older adult patient admitted for the treatment of depression. The health care provider prescribes an antidepressant medication, but the dose is more than the usual adult dose. The nurse should: A. Implement the order B. Consult a drug reference C. Give the usual geriatric dosage D. Hold the medication and consult the health care provider

D. Hold the medication and consult the health care provider Rationale: The dose of an antidepressant medication for older adult patients is often less than the usual adult dose. The nurse should withhold the medication and consult the health care provider who wrote the order. The nurse's duty is to intervene and protect the patient. Consulting a drug reference is unnecessary because the nurse already knows the dose is excessive. Implementing the order is negligent. Giving the usual geriatric dose would be wrong; a nurse without prescriptive privileges cannot change the dose.

The spouse of a patient who has delusions asks the nurse, "Are there any circumstances under which the treatment team is justified in violating the patient's right to confidentiality?" The nurse must reply that confidentiality may be breached: A. Under no circumstances B. At the discretion of the psychiatrist C. When questions are asked by law enforcement D. If the patient threatens the life of another person

D. If the patient threatens the life of another person Rationale: The duty to warn a person whose life has been threatened by a patient under psychiatric treatment overrides the patient's right to confidentiality. The right to confidentiality is not suspended at the discretion of the therapist or for legal investigations.

A nurse documents: "Patient is mute, despite repeated efforts to elicit speech. Makes no eye contact. Is inattentive to staff. Gazes off to the side or looks upward rather than at the speaker." Which nursing diagnosis should be considered? A. Defensive coping B. Decisional conflict C. Risk for other-directed violence D. Impaired verbal communication

D. Impaired verbal communication Rationale: The defining characteristics are more related to the nursing diagnosis of Impaired verbal communication than to the other nursing diagnoses.

A patient begins a new program to assist with building social skills. In which part of the plan of care should a nurse record the item "Encourage patient to attend one psychoeducational group daily"? A. Assessment B. Analysis C. Planning D. Implementation E. Evaluation

D. Implementation Rationale: Interventions are the nursing prescriptions to achieve the outcomes. Interventions should be specific.

In a team meeting a nurse says, "I'm concerned whether we are behaving ethically by using restraint to prevent one patient from self-mutilation while the care plan for another patient who has also self-mutilated calls for one-on-one supervision." Which ethical principle most clearly applies to this situation? A. Beneficence B. Autonomy C. Fidelity D. Justice

D. Justice Rationale: The nurse is concerned about justice, that is, the fair treatment with the least restrictive methods for both patients. Beneficence means promoting the good of others. Autonomy is the right to make one's own decisions. Fidelity is the observance of loyalty and commitment to the patient.

A student says, "Before taking a test, I feel a heightened sense of awareness and restlessness." The nurse can correctly assess the student's experience as: A. Culturally influenced B. Displacement C. Trait anxiety D. Mild anxiety

D. Mild anxiety Rationale: Mild anxiety is rarely obstructive to the task at hand. It may be helpful to the patient because it promotes study and increases awareness of the nuances of questions. The incorrect responses have different symptoms.

A patient's nursing diagnosis is Insomnia. The desired outcome is: "Patient will sleep for a minimum of 5 hours nightly by October 31." On November 1, a review of the sleep data shows the patient sleeps an average of 4 hours nightly and takes a 2-hour afternoon nap. Which evaluation should be documented? A. Consistently demonstrated B. Often demonstrated C. Sometimes demonstrated D. Never demonstrated

D. Never demonstrated Rationale: Although the patient is sleeping 6 hours daily, the total is not in one uninterrupted session at night. Therefore the outcome must be evaluated as never demonstrated.

A recently widowed patient tells the health care provider, "I have so much epigastric discomfort. I wonder if I have an ulcer." Diagnostic tests are negative. The symptom demonstrates: A. Early reorganization behavior B. Disorganization and depression C. Preoccupation with the deceased D. Normal phenomenon of mourning

D. Normal phenomenon of mourning Rationale: Sensations of somatic distress are often experienced during the acute stage of grieving. They include tightness in the throat, shortness of breath, exhaustion, and pain or sensations such as those experienced by the dead person.

A supervisor assigns a worker a new project. The worker initially agrees but feels resentful. The next day when asked about the project, the worker says, "I've been working on other things." When asked 4 hours later, the worker says, "Someone else was using the copier, so I couldn't finish it." The worker's behavior demonstrates: A. Acting out B. Projection C. Rationalization D. Passive aggression

D. Passive aggression Rationale: A passive-aggressive person deals with emotional conflict by indirectly expressing aggression toward others. Compliance on the surface masks covert resistance. Resistance is expressed through procrastination, inefficiency, and stubbornness in response to assigned tasks.

A newly admitted patient who is acutely psychotic is a private patient of the senior psychiatrist. To whom does the psychiatric nurse who is assigned to this patient owe the duty of care? A. Health care provider B. Profession C. Hospital D. Patient

D. Patient Rationale: Although the nurse is accountable to the health care provider, the agency, the patient, and the profession, the duty of care is owed to the patient.

Which assessment finding for a patient in the community requires priority intervention by the nurse? The patient: A. Receives Social Security disability income plus a small check from a trust fund B. Lives in an apartment with two patients who attend day hospital programs C. Has a sibling who is interested and active in care planning D. Purchases and uses marijuana on a frequent basis

D. Purchases and uses marijuana on a frequent basis Rationale: Patients who regularly buy illegal substances often become medication noncompliant. Medication noncompliance, along with the disorganizing influence of illegal drugs on cellular brain function, promotes relapse. The remaining options do not suggest problems.

An example of a breach of a patient's right to privacy occurs when a nurse: A. Asks a family to share information about a patient's prehospitalization behavior B. Discusses the patient's history with other staff members during care planning C. Documents the patient's daily behaviors during hospitalization D. Releases information to the patient's employer without consent

D. Releases information to the patient's employer without consent Rationale: The release of information without patient authorization violates the patient's right to privacy. The other options are acceptable nursing practices.

A patient's nursing diagnosis is Insomnia. The desired outcome is: "Patient will sleep for a minimum of 5 hours nightly by October 31." On November 1, a review of the sleep data shows the patient sleeps an average of 4 hours nightly and takes a 2-hour afternoon nap. What is the nurse's next action? A. Continue the current plan without changes B. Remove this nursing diagnosis from the plan of care C. Write a new nursing diagnosis that better reflects the problem D. Revise the target date for outcome attainment and examine interventions

D. Revise the target date for outcome attainment and examine interventions Rationale: Sleeping a total of 5 hours at night remains a reasonable outcome. Extending the time frame for attaining the outcome is appropriate. Examining interventions might result in planning an activity during the afternoon rather than permitting a nap. Continuing the current plan without changes is inappropriate. At the very least, the time in which the outcome is to be attained must be extended. Removing this nursing diagnosis from the plan of care could be used when the outcome goal has been met and the problem resolved. Writing a new nursing diagnosis is inappropriate because no other nursing diagnosis relates to the problem.

Which scenario is an example of an adventitious crisis? A. Death of a child from sudden infant death syndrome B. Being fired from a job because of company downsizing C. Retirement of a 55-year-old D. Riot at a rock concert

D. Riot at a rock concert Rationale: The rock concert riot is unplanned, accidental, violent, and not a part of everyday life. The incorrect options are examples of situational or maturational crises.

A patient newly diagnosed with pancreatic cancer says, "My father also died of pancreatic cancer. I took care of him during his illness. I can't go through that." Select the highest priority nursing diagnosis. A. Anticipatory grieving B. Ineffective coping C. Ineffective denial D. Risk for suicide

D. Risk for suicide Rationale: The patient's statement has a subtle message of suicide. Suicide is a risk for people with major losses, including terminal disease. The nurse will need to monitor the suicide risk vigilantly. The other diagnoses may apply but are lower priority.

A soldier served in combat zones in Iraq in 2010 and was deployed to Afghanistan in 2011. When is it most important for the nurse to screen for signs and symptoms of post-traumatic stress disorder (PTSD)? A. Immediately upon return to the United States from Afghanistan B. Before departing Afghanistan to return to the United States C. One year after returning from Afghanistan D. Screening should be ongoing

D. Screening should be ongoing Rationale: PTSD can have a long lag time, of months to years. Screening should be ongoing.

Select the best outcome for a patient with the nursing diagnosis: Impaired social interaction, related to sociocultural dissonance as evidenced by stating, "Although I'd like to, I don't join in because I don't speak the language very well." The patient will: A. Demonstrate improved social skills B. Express a desire to interact with others C. Become more independent in decision making D. Select and participate in one group activity per day

D. Select and participate in one group activity per day Rationale: The outcome describes social involvement on the part of the patient. Neither cooperation nor independence has been an issue. The patient has already expressed a desire to interact with others. Outcomes must be measurable. Two of the distracters are not measurable

Select the most appropriate label to complete this nursing diagnosis: ___________, related to feelings of shyness and poorly developed social skills as evidenced by watching television alone at home every evening. A. Deficient knowledge B. Ineffective coping C. Powerlessness D. Social isolation

D. Social isolation Rationale: Nursing diagnoses are selected on the basis of the etiologic factors and assessment findings or evidence. In this instance, the evidence shows social isolation that is caused by shyness and poorly developed social skills.

Which patient would a nurse refer to partial hospitalization? An individual who: A. Spent yesterday in the 24-hour supervised crisis care center and continues to be actively suicidal. B. Because of agoraphobia and panic episodes needs psychoeducation for relaxation therapy C. Has a therapeutic lithium level and reports regularly for blood tests and clinic follow-up. D. States, "I'm not sure I can avoid using alcohol when my spouse goes to work every morning."

D. States, "I'm not sure I can avoid using alcohol when my spouse goes to work every morning." Rationale: This patient could profit from the structure and supervision provided by spending the day at the partial hospitalization program. During the evening, at night, and on weekends, the spouse could assume supervision responsibilities. The patient who is actively suicidal needs inpatient hospitalization. The patient in need of psychoeducation can be referred to home care. The patient who reports regularly for blood tests and clinical follow-up can continue on the same plan.

A patient states, "I'm not worth anything. I have negative thoughts about myself. I feel anxious and shaky all the time. Sometimes I feel so sad that I want to go to sleep and never wake up." Which nursing intervention should have the highest priority? A. Self-esteem-building activities B. Anxiety self-control measures C. Sleep enhancement activities D. Suicide precautions

D. Suicide precautions Rationale: The nurse should place priority on monitoring and reinforcing suicide self-restraint because it relates directly and immediately to patient safety. Patient safety is always a priority concern. The nurse should monitor and reinforce all patient attempts to control anxiety, improve sleep patterns, and develop self-esteem while giving priority attention to suicide self-restraint.

A terminally ill patient tells the nurse, "Life has been good. I am proud of being self-educated. I overcame adversity with willpower. I always gave my best and expected things to turn out well. I intend to die as I lived: optimistically." The nurse planning care for this patient recognizes a critical need to: A. Provide aggressive pain and symptom management B. Help the patient reassess and explore existing conflicts C. Assist the patient to focus on the meaning in life and death D. Support the patient's use of personal resources to meet challenges

D. Support the patient's use of personal resources to meet challenges Rationale: The patient whose intrinsic strength and endurance have been a hallmark often wishes to approach dying by staying optimistic and in control. Helping the patient use his or her resources to meet challenges is appropriate.

A patient is brought to the emergency department after a motorcycle accident. The patient is alert, responsive, and diagnosed with a broken leg. The patient's vital signs are temperature (T), 98.6° F; pulse (P), 72 beats per minute (bpm); and respirations (R), 16 breaths per minute. After being informed that surgery is required for the broken leg, which vital sign readings would be expected? A. T, 98.6°; P, 64; R, 14 B. T, 98.6°; P, 68; R, 12 C. T, 98.6°; P, 62; R, 16 D. T, 98.6°; P, 84; R, 22

D. T, 98.6°; P, 84; R, 22 Rationale: The patient would experience stress associated with the anticipation of surgery. In times of stress, the sympathetic nervous system takes over (fight or flight response) and sends signals to the adrenal glands thereby releasing norepinephrine. The circulating norepinephrine increases the heart rate. Respirations increase bringing more oxygen to the lungs.

A nurse is talking with a patient, and 5 minutes remain in the session. The patient has been silent for most of the session. Another patient comes to the door of the room, interrupts, and says to the nurse, "I really need to talk to you." The nurse should: A. Say to the interrupting patient, "I am not available to talk with you at the present time." B. End the unproductive session with the current patient and spend time with the patient who has just interrupted. C. Invite the interrupting patient to join in the session with the current patient. D. Tell the patient who has interrupted, "This session is 5 more minutes; then I will talk with you."

D. Tell the patient who has interrupted, "This session is 5 more minutes; then I will talk with you." Rationale: When a specific duration for a session has been set, the nurse must adhere to the schedule. Leaving the first patient would be equivalent to abandonment and would destroy any trust the patient had in the nurse. Adhering to the contract demonstrates that the nurse can be trusted and that the patient and the sessions are important. The incorrect responses preserve the nurse-patient relationship with the silent patient but may seem abrupt to the interrupting patient, abandon the silent patient, or fail to observe the contract with the silent patient.

A nurse working with a person whose spouse recently died uses cheer and humor to lift the person's spirits. At one point, the widowed person smiles. What analysis of this scenario is correct? A. The nurse's technique was successful B. Use of humor should be added to the plan of care C. Approach may prove useful in other, similar situations D. The nurse needs supervision; the communication technique was not appropriate

D. The nurse needs supervision; the communication technique was not appropriate Rationale: Clinical supervision will review the nurse's actions and thoughts and help the nurse arrive at a more therapeutic approach. Attempts at cheering up a patient who is depressed serve only to emphasize the disparity between the patient's mood and that of others. Active listening should be the technique used by the nurse. The incorrect options suggest the approach is therapeutic when it is not.

A Filipino-American patient had this nursing diagnosis: Situational low self-esteem, related to poor social skills as evidenced by lack of eye contact. Interventions were used to raise the patient's self-esteem; however, after 3 weeks, the patient's eye contact did not improve. What is the most accurate analysis of this scenario? A. The patient's eye contact should have been directly addressed by role-playing to increase comfort with eye contact B. The nurse should not have independently embarked on assessment, diagnosis, and planning for this patient C. The patient's poor eye contact is indicative of anger and hostility that remain unaddressed D. The nurse should have assessed the patient's culture before making this diagnosis and plan

D. The nurse should have assessed the patient's culture before making this diagnosis and plan Rationale: The amount of eye contact in which a person engages is often culturally determined. In some cultures, eye contact is considered insolent, whereas in other cultures, eye contact is expected. Asian Americans, including persons from the Philippines, often prefer not to engage in direct eye contact.

A child drowned while swimming in a local lake 4 years earlier. Which behavior indicates that the parents are effectively coping with their loss? The parents: A. Prohibit their other children from going swimming B. Keep a place set for the dead child at the family dinner table C. Keep their child's room exactly as the child left it 4 years ago D. Throw flowers on the lake at each anniversary date of the accident

D. Throw flowers on the lake at each anniversary date of the accident Rationale: The loss of a child is among the highest risk situations for dysfunctional grieving. The parents who throw flowers across the lake on each anniversary date of the accident are effectively using a ritual to express their feelings openly. The other behaviors indicate the parents are isolating themselves or denying their feelings or both.

A grieving patient tells a nurse, "It's been 8 months since my spouse died. I thought I would feel better by now, but lately I feel worse. I have no energy. I am lonely, but I don't want to be around people. What should I do?" What is the nurse's best counsel? A. Seek psychotherapy B. Become active in a church C. Attend a bereavement group D. Understand this is a normal response

D. Understand this is a normal response Rationale: The patient needs understanding and support that the feelings are normal. Although feelings of depression generally decline over the period of a year after the death of a loved one, the decline is not linear. Loneliness and aimlessness are most pronounced 6 to 9 months after the death. The patient should be educated about phenomena experienced during bereavement. The other options are not clearly indicated.

After completing the contracted number of visits to the crisis clinic, an adult says, "I've emerged from this as a stronger person. You supported me while I worked through my feelings of loss and helped me find community resources. I'm benefiting from a support group." The nurse can evaluate the patient's feelings about the care received as: A. Not at all satisfied B. Somewhat satisfied C. Moderately satisfied D. Very satisfied

D. Very satisfied Rationale: The patient mentions a number of indicators that suggest a high degree of satisfaction with the Nursing Outcomes Classification of patient satisfaction: psychologic care. No indicators express low-to-moderate satisfaction.

Which documentation of a patient's behavior best demonstrates a nurse's observations? A. Isolates self from others. Frequently fell asleep during group. Vital signs stable. B. Calmer and more cooperative. Participated actively in group. No evidence of psychotic thinking. C. Appeared to hallucinate. Patient frequently increased volume on television, causing conflict with others D. Wears four layers of clothing. States, "I need protection from dangerous bacteria trying to penetrate my skin."

D. Wears four layers of clothing. States, "I need protection from dangerous bacteria trying to penetrate my skin." Rationale: The documentation states specific observations of the patient's appearance and the exact statements made. The other options are vague or subjective statements and can be interpreted in different ways.

A patient tells a nurse, "I have psychiatric problems and am in and out of hospitals all the time. Not one of my friends or relatives has these problems." Select the nurse's best response. A. "Comparing yourself with others has no real advantages." B. "Why do you blame yourself for having a psychiatric illness?" C. "Mental illness affects 50% of the adult population in any given year." D. "It sounds like you are concerned that others don't experience the same challenges as you."

A. "Comparing yourself with others has no real advantages." Rationale: Mental illness affects many people at various times in their lives. No class, culture, or creed is immune to the challenges of mental illness. The correct response also demonstrates the use of reflection, a therapeutic communication technique. It is not true that mental illness affects 50% of the population in any given year. Asking patients if they blame themselves is an example of probing.

A patient is hospitalized for depression and suicidal ideation after their spouse asks for a divorce. Select the nurse's most caring comment. A. "Let's discuss some means of coping other than suicide when you have these feelings." B. "I understand why you're so depressed. When I got divorced, I was devastated too." C. "You should forget about your marriage and move on with your life." D. "How did you get so depressed that hospitalization was necessary?"

A. "Let's discuss some means of coping other than suicide when you have these feelings." Rationale: The nurse's communication should evidence caring and a commitment to work with the patient. This commitment lets the patient know the nurse will help. Probing and advice are not helpful or therapeutic interventions.

A student nurse tells the instructor, "I don't need to interact with my patients. I learn what I need to know by observation." The instructor can best interpret the nursing implications of Sullivan's theory to the student by responding: A. "Nurses cannot be isolated. We must interact to provide patients with opportunities to practice interpersonal skills." B. "Observing patient interactions can help you formulate priority nursing diagnoses and appropriate interventions." C. "I wonder how accurate your assessment of the patient's needs can be if you do not interact with the patient." D. "Noting patient behavioral changes is important because these signify changes in personality."

A. "Nurses cannot be isolated. We must interact to provide patients with opportunities to practice interpersonal skills." Rationale: Sullivan believed that the nurse's role includes educating patients and assisting them in developing effective interpersonal relationships. Mutuality, respect for the patient, unconditional acceptance, and empathy are cornerstones of Sullivan's theory. The nurse who does not interact with the patient cannot demonstrate these cornerstones. Observations provide only objective data. Priority nursing diagnoses usually cannot be accurately established without subjective data from the patient. The third response pertains to Maslow's theory. The fourth response pertains to behavioral theory.

A patient states, "I'm starting cognitive behavioral therapy. What can I expect from the sessions?" Which responses by the nurse are appropriate? Select all that apply. A. "The therapist will be active and questioning." B. "You may be given homework assignments." C. "The therapist will ask you to describe your dreams." D. "The therapist will help you look at ideas and beliefs you have about yourself." E. "The goal is to increase your subjectivity about thoughts that govern your behavior."

A. "The therapist will be active and questioning." B. "You may be given homework assignments." D. "The therapist will help you look at ideas and beliefs you have about yourself." Rationale: Cognitive therapists are active rather than passive during therapy sessions because they help patients to reality test their thinking. Homework assignments are given and completed outside the therapy sessions. Homework is usually discussed at the next therapy session. The goals of cognitive therapy are to assist the patient to identify inaccurate cognitions, to reality test their thinking, and to formulate new, accurate cognitions. Dream describing applies to psychoanalysis, not cognitive behavioral therapy. The desired outcome of cognitive therapy is to assist patients in increasing their objectivity, not subjectivity, about the cognitions that influence behavior.

A 26-month-old child displays negative behaviors. The parent says, "My child refuses toilet training and shouts, 'No!' when given direction. What do you think is wrong?" Select the nurse's best reply. A. "This is normal for your child's age. The child is striving for independence." B. "The child needs firmer control. Punish the child for disobedience and say, 'No.'" C. "There may be developmental problems. Most children are toilet trained by age 2 years." D. "Some undesirable attitudes are developing. A child psychologist can help you develop a remedial plan."

A. "This is normal for your child's age. The child is striving for independence." Rationale:These negative behaviors are typical of a child around the age of 2 years whose developmental task is to develop autonomy. The remaining options indicate the child's behavior is abnormal.

A multidisciplinary health care team meets 12 hours after an adolescent is hospitalized after a suicide attempt. Members of the team report their assessments. What outcome can be expected from this meeting? A. A treatment plan will be determined B. The health care provider will order neuroimaging studies C. The team will request a court-appointed advocate for the patient D. Assessment of the patient's need for placement outside the home will be undertaken

A. A treatment plan will be determined Rationale: Treatment plans are formulated early in the course of treatment to streamline the treatment process and reduce costs. It is too early to determine the need for alternative postdischarge living arrangements. Neuroimaging is not indicated for this scenario.

A bill introduced in Congress would reduce funding for the care of people with mental illnesses. A group of nurses write letters to their elected representatives in opposition to the legislation. Which role have the nurses fulfilled? A. Advocacy B. Attending C. Recovery D. Evidence-based practice

A. Advocacy Rationale: An advocate defends or asserts another's cause, particularly when the other person lacks the ability to do that for him- or herself. Examples of individual advocacy include helping patients understand their rights or make decisions. On a community scale, advocacy includes political activity, public speaking, and publication in the interest of improving the individuals with mental illness; the letter-writing campaign advocates for that cause on behalf of patients who are unable to articulate their own needs.

A patient who immigrated to the United States from Honduras was diagnosed with schizophrenia. The patient took an antipsychotic medication for 3 weeks but showed no improvement. Which resource should the treatment team consult for information on more effective medications for this patient? A. Clinical algorithm B. Clinical pathway C. Clinical practice guideline D. International Statistical Classification of Diseases and Related Health Problems (ICD)

A. Clinical algorithm Rationale: A clinical algorithm is a guideline that describes diagnostic and/or treatment approaches drawn from large databases of information. These guidelines help the treatment team make decisions cognizant of an individual patient's needs, such as ethnic origin, age, or gender. A clinical pathway is a map of interventions and treatments related to a specific disorder. Clinical practice guidelines summarize best practices about specific health problems. The ICD classifies diseases.

A patient has dementia. The health care provider wants to make a differential diagnosis between Alzheimer disease and multiple infarctions. Which diagnostic procedure should a nurse expect to prepare the patient for first? A. Computed tomography (CT) scan B. Positron emission tomography (PET) scan C. Functional magnetic resonance imaging (fMRI) D. Single-photon-emission computed tomography (SPECT) scan

A. Computed tomography (CT) scan Rationale: A CT scan shows the presence or absence of structural changes, including cortical atrophy, ventricular enlargement, and areas of infarction—information that will be helpful to the health care provider. The other tests focus on brain activity and are more expensive; they may be ordered later.

A nurse at a behavioral health clinic sees an unfamiliar psychiatric diagnosis on a patient's insurance form. Which resource should the nurse consult to discern the criteria used to establish this diagnosis? A. DSM-IV-TR B. Nursing Diagnosis Manual C. A psychiatric nursing textbook D. A behavioral health reference manual

A. DSM-IV-TR Rationale: The DSM-IV-TR gives the criteria used to diagnose each mental disorder. The Nursing Diagnosis Manual focuses on nursing diagnoses. A psychiatric nursing textbook or behavioral health reference manual may not contain diagnostic criteria.

A patient's spouse, who is a chemist, asks a nurse how serotonin reuptake inhibitors (SSRIs) lift depression. The nurse should explain that SSRIs: A. Destroy increased amounts of neurotransmitters B. Make more serotonin available at the synaptic gap C. Increase production of acetylcholine and dopamine D. Block muscarinic and alpha1-norepinephrine receptors

A. Destroy increased amounts of neurotransmitters Rationale: Depression is thought to be related to the lowered availability of the neurotransmitter serotonin. SSRIs act by blocking the reuptake of serotonin, leaving a higher concentration available at the synaptic cleft. They actually prevent the destruction of serotonin, have no effect on acetylcholine and dopamine production, and do not block muscarinic or alpha1-norepinephrine receptors.

A drug causes muscarinic-receptor blockade. A nurse will assess the patient for: A. Dry mouth B. Gynecomastia C. Pseudoparkinsonism D. Orthostatic hypotension

A. Dry mouth Rationale: Muscarinic-receptor blockade includes atropine-like side effects such as dry mouth, blurred vision, and constipation. Gynecomastia is associated with decreased prolactin levels. Movement defects are associated with dopamine blockade. Orthostatic hypotension is associated with alpha1-receptor antagonism.

A patient asks a nurse, "The pamphlet I read about depression says psychosocial factors influence depression. What does that mean?" Which examples could the nurse cite to support the information? Select all that apply. A. Having a hostile family B. Having an over- or under-involved family C. Having two first-degree relatives with bipolar disorder D. Experiencing the sudden death of a parent or loved one E. Feeling strong guilt over having an abortion when one's religion forbids it F. Experiencing symptom remission when treated with an antidepressant medication

A. Having a hostile family B. Having an over- or under-involved family D. Experiencing the sudden death of a parent or loved one E. Feeling strong guilt over having an abortion when one's religion forbids it Rationale: Family influence is considered a psychosocial factor affecting a patient's mental health. A hostile under- or overinvolved family is critical of the patient and contributes to low self-esteem. Religious influences are considered psychosocial in nature. Life experiences, especially crises and losses, are considered psychosocial influences on mental health. Having two first-degree relatives with bipolar disorder would be considered a factor that influences the individual's risk for mental disorder, but it is a genetic, not psychosocial, factor. Treatment with a biological agent such as an antidepressant medication is an example of a biological influence.

A psychiatric nurse addresses Axis I of the DSM-IV-TR as the focus of care but also considers the presence of other long-term, nonmedical disorders that may affect treatment. To which axis should the nurse refer for this information? A. II B. III C. IV D. V

A. II Rationale: Axis II refers to personality disorders and mental retardation. Together, Axis I and Axis II constitute the classification of abnormal behavior diagnosed in the individual. Axis III indicates any relevant general medical conditions. Axis IV reports psychosocial and environmental problems that may affect the diagnosis, treatment, and prognosis. Axis V is the GAF score.

A 4-year-old child grabs toys from siblings, saying, "I want that toy now!" The siblings cry, and the child's parent becomes upset with the behavior. Using the Freudian theory, a nurse can interpret the child's behavior as a product of impulses originating in the: A. Id B. Ego C. Superego D. Preconscious

A. Id Rationale: The id operates on the pleasure principle, seeking immediate gratification of impulses. The ego acts as a mediator of behavior and weighs the consequences of the action, perhaps determining that taking the toy is not worth the parent's wrath. The superego would oppose the impulsive behavior as "not nice." The preconscious is a level of awareness.

The therapeutic action of monoamine oxidase inhibitors (MAOIs) blocks neurotransmitter reuptake, causing: A. Increased concentration of neurotransmitters in the synaptic gap B. Decreased concentration of neurotransmitters in serum C. Destruction of receptor sites D. Limbic system stimulation

A. Increased concentration of neurotransmitters in the synaptic gap Rationale: If the reuptake of a substance is inhibited, then it accumulates in the synaptic gap and its concentration increases, permitting the ease of the transmission of impulses across the synaptic gap. Normal transmission of impulses across synaptic gaps is consistent with a normal rather than a depressed mood. The other options are not associated with blocking neurotransmitter reuptake.

A nurse assesses an inpatient psychiatric unit, noting that exits are free from obstruction, no one is smoking, the janitor's closet is locked, and all sharp objects are being used under staff supervision. These observations relate to: A. Management of milieu safety B. Coordinating care of patients C. Management of the interpersonal climate D. Use of therapeutic intervention strategies

A. Management of milieu safety Rationale: Members of the nursing staff are responsible for all aspects of milieu management. The observations mentioned in this question directly relate to the safety of the unit. The other options, although part of the nurse's concerns, are unrelated to the observations cited.

An advanced practice nurse determines a group of patients would benefit from therapy in which peers and interdisciplinary staff all have a voice in determining the level of the patients' privileges. The nurse would arrange for: A. Milieu therapy B. Cognitive therapy C. Short-term dynamic therapy D. Systematic desensitization

A. Milieu therapy Rationale: Milieu therapy is based on the idea that all members of the environment contribute to the planning and functioning of the setting. The other therapies are all individual therapies that do not fit the description.

A nurse caring for a patient taking a serotonin reuptake inhibitor (SSRI) will develop outcome criteria related to: A. Mood improvement B. Logical thought processes C. Reduced levels of motor activity D. Decreased extrapyramidal symptoms

A. Mood improvement Rationale: SSRIs affect mood, relieving depression in many patients. SSRIs do not act to reduce thought disorders. SSRIs reduce depression but have little effect on motor hyperactivity. SSRIs do not produce extrapyramidal symptoms.

A nurse surveys the medical records for violations of patients' rights. Which finding signals a violation? A. No treatment plan is present in record B. Patient belongings are searched at admission C. Physical restraint is used to prevent harm to self D. Patient is placed on one-to-one continuous observation

A. No treatment plan is present in record Rationale: The patient has the right to have a treatment plan. Inspecting a patient's belongings is a safety measure. Patients have the right to a safe environment, including the right to be protected against impulses to harm self that occur as a result of a mental disorder.

A nurse says, "When I was in school I learned to call upset patients by name to get their attention, but I read a descriptive research study that says that this approach doesn't work. I'm going stop calling patients by name." Which statement is the best appraisal of this nurse's comment? A. One descriptive research study rarely provides enough evidence to change practice. B. Staff nurses apply new research findings only with the help from clinical nurse specialists. C. New research findings should be incorporated into clinical algorithms before using them in practice. D. The nurse misinterpreted the results of the study. Classic tenets of practice do not change.

A. One descriptive research study rarely provides enough evidence to change practice. Rationale: Descriptive research findings provide evidence for practice but must be viewed in relation to other studies before practice changes. One study is not enough. Descriptive studies are low on the hierarchy of evidence. Clinical algorithms use flow charts to manage problems and do not specify one response to a clinical problem. Classic tenets of practice should change as research findings provide evidence for change.

A nurse assesses that a patient is suspicious and frequently manipulates others. Using the Freudian theory, these traits are related to which psychosexual stage? A. Oral B. Anal C. Phallic D. Genital

A. Oral Rationale: According to Freud, each of the behaviors mentioned develops as the result of attitudes formed during the oral stage, when an infant first learns to relate to the environment. Anal stage traits include stinginess, stubbornness, orderliness, or their opposites. Phallic stage traits include flirtatiousness, pride, vanity, difficulty with authority figures, and difficulties with sexual identity. Genital stage traits include the ability to form satisfying sexual and emotional relationships with members of the opposite sex, emancipation from parents, and a strong sense of personal identity.

A nurse should assess a patient taking a medication with anticholinergic properties for inhibited function of the: A. Parasympathetic nervous system B. Sympathetic nervous system C. Reticular activating system D. Medulla oblongata

A. Parasympathetic nervous system Rationale: Acetylcholine is the neurotransmitter found in high concentration in the parasympathetic nervous system. When acetylcholine action is inhibited by anticholinergic drugs, parasympathetic symptoms such as blurred vision, dry mouth, constipation, and urinary retention appear. The functions of the sympathetic nervous system, the reticular activating system, and the medulla oblongata are not affected by anticholinergic medications.

A nurse prepares to administer an antipsychotic medication to a patient with schizophrenia. Additional monitoring of the medication's effects and side effects will be most important if the patient is also diagnosed with which health problem? Select all that apply. A. Parkinson disease B. Graves disease C. Osteoarthritis D. Epilepsy E. Diabetes

A. Parkinson disease D. Epilepsy E. Diabetes Rationale: Antipsychotic medications may produce weight gain, which complicates the care of a patient with diabetes or lowers the seizure threshold or both, which complicates the care of a patient with epilepsy. Parkinson disease involves changes in transmission of dopamine and acetylcholine; therefore these drugs also complicate the care of a patient with the disorder. Osteoarthritis and Graves disease should have no synergistic effect with this medication.

An individual is experiencing problems associated with memory. Which cerebral structures are most likely to be involved in this deficit? Select all that apply. A. Prefrontal cortex B. Occipital lobe C. Temporal lobe D. Parietal lobe E. Basal ganglia

A. Prefrontal cortex C. Temporal lobe D. Parietal lobe Rationale: The prefrontal cortex, parietal, and temporal lobes of the cerebrum play a key role in the storage and processing of memories. The occipital lobe is predominantly involved with vision. The basal ganglia influence the integration of physical movement, as well as some thoughts and emotions.

Planning for patients with mental illness is facilitated by understanding that inpatient hospitalization is generally reserved for patients who: A. Present a clear danger to self or others B. Are noncompliant with medications at home C. Have no support systems in the community D. Develop new symptoms during the course of an illness

A. Present a clear danger to self or others Rationale: Hospitalization is justified when the patient is a danger to self or others, has dangerously decompensated, or needs intensive medical treatment. The incorrect options do not necessarily describe patients who require inpatient treatment.

A nurse administers a medication that potentiates the action of gamma-aminobutyric acid (GABA). Which finding would be expected? A. Reduced anxiety B. Improved memory C. More organized thinking D. Fewer sensory perceptual alterations

A. Reduced anxiety Rationale: Increased levels of GABA reduce anxiety, thus any potentiation of GABA action should result in anxiety reduction. Memory enhancement is associated with acetylcholine and substance P. Thought disorganization is associated with dopamine. GABA is not associated with sensory perceptual alterations.

Priority teaching for a patient taking clozapine (Clozaril) should include which instruction? A. Report sore throat and fever immediately B. Avoid foods high in polyunsaturated fat C. Use water-based lotions for rashes D. Avoid unprotected sex

A. Report sore throat and fever immediately Rationale: Clozapine therapy may produce agranulocytosis; therefore signs of infection should be immediately reported to the health care provider. In addition, the patient should have white blood cell levels measured weekly. The other options are not relevant to clozapine administration.

The laboratory report for a patient taking clozapine (Clozaril) shows a white blood cell count of 3000 mm3 and a granulocyte count of 1500 mm3. The nurse should: A. Report the laboratory results to the health care provider B. Give the next dose as prescribed C. Administer aspirin and force fluids D. Repeat the laboratory tests

A. Report the laboratory results to the health care provider Rationale: These laboratory values indicate the possibility of agranulocytosis, a serious side effect of clozapine therapy. These results must be immediately reported to the health care provider. The drug should be withheld because the health care provider will discontinue it. The health care provider may repeat the laboratory test, but, in the meantime, the drug should be withheld. Giving aspirin and forcing fluids are measures that are less important than stopping the administration of the drug.

A mentally ill person's current global assessment of functioning (GAF) score is 10. Select the nurse's highest priority related to this patient's care. A. Safety B. Hygiene C. Nutrition D. Socialization

A. Safety Rationale: This low GAF score indicates a consistent risk for self-harm exists; therefore the nurse's highest priority is safety.

A 40-year-old adult living with parents states, "I'm happy but I don't socialize much. My work is routine. When new things come up, my boss explains them a few times to make sure I understand. At home, my parents make decisions for me, and I go along with them." A nurse should identify interventions to improve this patient's: A. Self-concept B. Overall happiness C. Appraisal of reality D. Control over behavior

A. Self-concept Rationale: The patient feels the need for multiple explanations of new tasks at work and, despite being 40 years of age, allows both parents to make all decisions. These behaviors indicate a poorly developed self-concept.

A psychiatric technician says, "Little of what takes place on the behavioral health unit seems to be theory based." A nurse educates the technician by identifying which common use of Sullivan's theory? A. Structure of the therapeutic milieu of most behavioral health units B. Frequent use of restraint and seclusion as behavior management tools C. Assessment tools based on age-appropriate versus arrested behaviors D. Method nurses use to determine the best sequence for nursing actions

A. Structure of the therapeutic milieu of most behavioral health units Rationale: The structure of the therapeutic environment has, as its foci, an accepting atmosphere and provision of opportunities for practicing interpersonal skills. Both constructs are directly attributable to Sullivan's theory of interpersonal relationships. Sullivan's interpersonal theory did not specifically consider the use of restraint or seclusion. Assessment based on the developmental level is more the result of Erikson's theories. Sequencing nursing actions based on the priority of patient needs is related to Maslow's hierarchy of needs.

A basic level registered nurse works with patients in a community setting. Which groups should this nurse expect to lead? Select all that apply. A. Symptom management B. Medication education C. Family therapy D. Psychotherapy E. Self-care

A. Symptom management B. Medication education E. Self-care Rationale: Symptom management, medication education, and self-care groups represent psychoeducation, a service provided by the basic level registered nurse. Advanced practice registered nurses provide family therapy and psychotherapy.

An experienced nurse says to a new graduate, "When you've practiced as long as I have, you'll instantly know how to take care of psychotic patients." What is the new graduate's best analysis of this comment? Select all that apply. A. The experienced nurse may have lost sight of patients' individuality, which may compromise the integrity of practice. B. New research findings must be continually integrated into a nurse's practice to provide the most effective care. C. Experience provides mental health nurses with the tools and skills needed for effective professional practice. D. Experienced psychiatric nurses have learned the best ways to care for psychotic patients through trial and error. E. Effective psychiatric nurses should be continually guided by an intuitive sense of patients' needs.

A. The experienced nurse may have lost sight of patients' individuality, which may compromise the integrity of practice. B. New research findings must be continually integrated into a nurse's practice to provide the most effective care. Rationale: Evidence-based practice involves using research findings to provide the most effective nursing care. Evidence is continually emerging; therefore, nurses cannot rely solely on experience. The effective nurse also maintains respect for each patient as an individual. Overgeneralization compromises that perspective. Intuition and trial and error are unsystematic approaches to care.

Which patient statement would lead a nurse to suspect that the developmental task of infancy was not successfully completed? A. "I have very warm and close friendships." B. "I'm afraid to let anyone really get to know me." C. "I am always right, so don't bother saying more." D. "I'm ashamed that I didn't do it correctly in the first place."

B. "I'm afraid to let anyone really get to know me." Rationale: According to Erikson, the developmental task of infancy is the development of trust. The patient's statement that he or she is afraid of becoming acquainted with others clearly shows a lack of ability to trust other people. Having warm and close friendships suggests the developmental task of infancy was successfully completed. The third option suggests rigidity rather than mistrust. The fourth option suggests failure to resolve the crisis of Initiative versus Guilt.

A patient asks a nurse, "What are neurotransmitters? My doctor says mine are out of balance." The best reply would be: A. "You must feel relieved to know that your problem has a physical basis." B. "Neurotransmitters are chemicals that pass messages between brain cells." C. "It is a high-level concept to explain. You should ask the doctor to tell you more." D. "Neurotransmitters are substances we eat daily that influence memory and mood."

B. "Neurotransmitters are chemicals that pass messages between brain cells." Rationale: Stating that neurotransmitters are chemicals that pass messages between brain cells gives the most accurate information. Neurotransmitters are messengers in the central nervous system. They are released from the axon terminal, diffuse across the synapse, and attach to specialized receptors on the postsynaptic neuron. The incorrect responses do not answer the patient's question, are demeaning, and provide untrue and misleading information.

Two nursing students discuss career plans after graduation. One student wants to enter psychiatric nursing. The other student asks, "Why would you want to be a psychiatric nurse? All they do is talk. You'll lose your skills." Select the best response by the student interested in psychiatric nursing. A. "Psychiatric nurses practice in safer environments than other specialties. Nurse-to-patient ratios must be better because of the nature of patients' problems." B. "Psychiatric nurses use complex communication skills, as well as critical thinking, to solve multidimensional problems. I'm challenged by those situations." C. "I think I'll be good in the mental health field. I do not like clinical rotations in school, so I don't want to continue them after I graduate." D. "Psychiatric nurses don't have to deal with as much pain and suffering as medical surgical nurses. That appeals to me."

B. "Psychiatric nurses use complex communication skills, as well as critical thinking, to solve multidimensional problems. I'm challenged by those situations." Rationale: The practice of psychiatric nursing requires a different set of skills than medical surgical nursing, although substantial overlap does exist. Psychiatric nurses must be able to help patients with medical and mental health problems, reflecting the holistic perspective these nurses must have. Nurse-patient ratios and workloads in psychiatric settings have increased, similar to other specialties. Psychiatric nursing involves clinical practice, not simply documentation. Psychosocial pain is real and can cause as much suffering as physical pain.

A patient shows the nurse an article from the Internet about a health problem. Which characteristic of the web site's address most alerts the nurse that the site may have biased and prejudiced information? A. Address ends in ".org." B. Address ends in ".com." C. Address ends in ".gov." D. Address ends in ".net."

B. Address ends in ".com." Rationale: Financial influences on a site are a clue that the information may be biased. ".com" at the end of the address indicates that the site is a commercial one. ".gov" indicates that the site is maintained by a government entity. ".org" indicates that the site is nonproprietary; the site may or may not have reliable information, but it does not profit from its activities. ".net" can have multiple meanings.

A 26-month-old child displays negative behavior, refuses toilet training, and often shouts, "No!" when given directions. Using Freud's stages of psychosexual development, a nurse would assess the child's behavior is based on which stage? A. Oral B. Anal C. Phallic D. Genital

B. Anal Rationale: In Freud's stages of psychosexual development, the anal stage occurs from age 1 to 3 years and has, as its focus, toilet training and learning to delay immediate gratification. The oral stage occurs between birth and 1 year, the phallic stage occurs between 3 and 5 years, and the genital stage occurs between 13 and 20 years.

A patient in the emergency department reports, "I hear voices saying someone is stalking me. They want to kill me because I found the cure for cancer. I will stab anyone that threatens me." Which aspects of mental health have the greatest immediate concern to a nurse? Select all that apply. A. Happiness B. Appraisal of reality C. Control over behavior D. Effectiveness in work E. Healthy self-concept

B. Appraisal of reality C. Control over behavior E. Healthy self-concept Rationale: The aspects of mental health of greatest concern are the patient's appraisal of and control over behavior. The patient's appraisal of reality is inaccurate, and auditory hallucinations are evident, as well as delusions of persecution and grandeur. In addition, the patient's control over behavior is tenuous, as evidenced by the plan to "stab" anyone who seems threatening. A healthy self-concept is lacking. Data are not present to suggest that the other aspects of mental health (happiness and effectiveness in work) are of immediate concern.

A patient has delusions and hallucinations. Before beginning treatment with a psychotropic medication, the health care provider wants to rule out the presence of a brain tumor. For which test will a nurse need to prepare the patient? A. Cerebral arteriogram B. Computed tomography (CT) scan or magnetic resonance imaging (MRI) C. Positron emission tomography (PET) or single photon emission computed tomography (SPECT) D. Functional magnetic resonance imaging (fMRI)

B. Computed tomography (CT) scan or magnetic resonance imaging (MRI) Rationale: A CT scan and an MRI visualize neoplasms and other structural abnormalities. A PET scan, SPECT scan, and fMRI, which give information about brain function, are not indicated. An arteriogram would not be appropriate.

A nurse consistently strives to demonstrate caring behaviors during interactions with patients. Which reaction by a patient indicates this nurse is effective? A patient reports feeling: A. Distrustful of others B. Connected with others C. Uneasy about the future D. Discouraged with efforts to improve

B. Connected with others Rationale: A patient is likely to respond to caring with a sense of connectedness with others. The absence of caring can make patients feel distrustful, disconnected, uneasy, and discouraged.

A nurse finds a new patient uncommunicative about recent life events. The nurse suspects marital and economic problems. The social worker's assessment is not available. The most effective action the nurse can take is to: A. Ask the patient who shares a room with him or her B. Consult Axis IV of the DSM-IV-TR in the medical record C. Focus questions on the topics of marital and economic issues D. Delay discussion of these topics until the social worker's assessment is available

B. Consult Axis IV of the DSM-IV-TR in the medical record Rationale: The physician's admission note identifies psychosocial and environmental problems on Axis IV pertinent to the patient's situation, providing another source of information for the nurse. Asking the patient who shares a room with him or her violates patient privacy rights. Persistent questioning may cause the patient to withdraw. Delaying the discussion until the social worker's assessment is available is not an effective solution.

A patient is admitted to the psychiatric hospital for assessment and evaluation. Which assessment finding best indicates that the patient has a mental illness? The patient: A. Describes coping and relaxation strategies used when feeling anxious B. Describes mood as consistently sad, discouraged, and hopeless C. Can perform tasks attempted within the limits of own abilities D. Reports occasional problems with insomnia

B. Describes mood as consistently sad, discouraged, and hopeless Rationale: A patient who reports having a consistently negative mood is describing a mood alteration. The incorrect options describe mentally healthy behaviors and common problems that do not indicate mental illness.

A patient taking medication for mental illness develops restlessness and an uncontrollable need to be in motion. A nurse can correctly analyze that these symptoms are related to which drug action? A. Anticholinergic effects B. Dopamine-blocking effects C. Endocrine-stimulating effects D. Ability to stimulate spinal nerves

B. Dopamine-blocking effects Rationale: Medications that block dopamine often produce disturbances of movement such as akathisia because dopamine affects neurons involved in both the thought processes and movement regulation. Anticholinergic effects include dry mouth, blurred vision, urinary retention, and constipation. Akathisia is not caused by endocrine stimulation or spinal nerve stimulation.

Which historical nursing leader helped focus practice to recognize the importance of science in psychiatric nursing? A. Abraham Maslow B. Hildegard Peplau C. Kris Martinsen D. Harriet Bailey

B. Hildegard Peplau Rationale: Although all these leaders included science as an important component of practice, Hildegard Peplau most influenced its development in psychiatric nursing. Maslow was not a nurse, but his theories influence how nurses prioritize problems and care. Bailey wrote a textbook in the 1930s on psychiatric nursing interventions. Kris Martinsen emphasized the importance of caring in nursing practice.

A nurse uses Peplau's interpersonal therapy while working with an anxious, withdrawn patient. Interventions should focus on: A. Changing the patient's perceptions about self B. Improving the patient's interactional skills C. Using medications to relieve anxiety D. Reinforcing specific behaviors

B. Improving the patient's interactional skills Rationale: The nurse-patient relationship is structured to provide a model for adaptive interpersonal relationships that can be generalized to others. Changing the patient's perceptions about his- or herself would be appropriate for cognitive therapy. Reinforcing specific behaviors would be used in behavioral therapy. Using medications would be the focus of biological therapy.

A patient hospitalized with a mood disorder has aggression, agitation, talkativeness, and irritability. A nurse begins the care plan based on the expectation that the health care provider is most likely to prescribe a medication classified as a(n): A. Anticholinergic B. Mood stabilizer C. Psychostimulant D. Antidepressant

B. Mood stabilizer Rationale: The symptoms describe a manic attack. Mania is effectively treated by the antimanic drug lithium and selected anticonvulsants such as carbamazepine, valproic acid, and lamotrigine. No drugs from the other classifications listed are effective in the treatment of mania.

A newly admitted patient is profoundly depressed, mute, and motionless. The patient has refused to bathe and eat for a week. Which score would be expected on the patient's global assessment of functioning? A. 100 B. 80 C. 50 D. 10

D. 10 Rationale: The patient is unable to maintain personal hygiene, oral intake, or verbal communication. The patient is dangerous to self because of the potential for starvation. A GAF score of 100 indicates high-level functioning. A score of 80 or 50 suggests higher functional abilities than the patient presently displays.

Which outcome, focused on recovery, would be expected in the plan of care for a patient living in the community with serious and persistent mental illness? Within 3 months, the patient will: A. Deny suicidal ideation B. Report a sense of well-being C. Take medications as prescribed D. Attend clinic appointments on time

B. Report a sense of well-being Rationale: Recovery emphasizes managing symptoms, reducing psychosocial disability, and improving role performance. The goal of recovery is to empower the individual with mental illness to achieve a sense of meaning and satisfaction in life and to function at the highest possible level of wellness. The incorrect options focus on the classic medical model rather than recovery.

Which research evidence would most influence a group of nurses to change their practice? A. Expert committee report of recommendations for practice B. Systematic review of randomized controlled trials C. Non-experimental descriptive study D. Critical pathway

B. Systematic review of randomized controlled trials Rationale: Research findings are graded using a hierarchy of evidence. A systematic review of randomized controlled trials is Level A and provides the strongest evidence for changing practice. Expert committee recommendations and descriptive studies lend less powerful and influential evidence. A critical pathway is not evidence; it incorporates research findings after they have been analyzed.

A patient is hospitalized for major depression. Of the medications listed, a nurse can expect to provide the patient with teaching about: A. chlordiazepoxide (Librium) B. fluoxetine (Prozac) C. clozapine (Clozaril) D. tacrine (Cognex)

B. fluoxetine (Prozac) Rationale: Fluoxetine is a selective serotonin reuptake inhibitor (SSRI), an antidepressant that blocks the reuptake of serotonin with few anticholinergic and sedating side effects; clozapine (Clozaril) is an antipsychotic medication; chlordiazepoxide (Librium) is an anxiolytic drug; and tacrine (Cognex) is used to treat Alzheimer disease.

Which statements most clearly reflect the stigma of mental illness? Select all that apply. A. "Many mental illnesses are hereditary." B. "Mental illness can be evidence of a brain disorder." C. "People claim mental illness so they can get disability checks." D. "If people with mental illness went to church, they would be fine." E. "Mental illness is a result of the breakdown of the American family."

C. "People claim mental illness so they can get disability checks." D. "If people with mental illness went to church, they would be fine." E. "Mental illness is a result of the breakdown of the American family." Rationale: Stigma is represented by judgmental remarks that discount the reality and validity of mental illness. Many mental illnesses are genetically transmitted. Neuroimaging can show changes associated with some mental illnesses.

The parent of a child who has schizophrenia tearfully asks a nurse, "What could I have done differently to prevent this illness?" Select the nurse's most caring response. A. "Although schizophrenia is caused by impaired family relationships, try not to feel guilty. No one can predict how a child will respond to parental guidance." B. "Most of the damage is done, but there is still hope. Changing your parenting style can help your child learn to cope more effectively with the environment." C. "Schizophrenia is a biological illness with similarities to diabetes and heart disease. You are not to blame for your child's illness." D. "Most mental illnesses result from genetic inheritance. Your genes are more at fault than your parenting."

C. "Schizophrenia is a biological illness with similarities to diabetes and heart disease. You are not to blame for your child's illness." Rationale: Patients and families need reassurance that the major mental disorders are biological in origin and are not the "fault" of parents. Knowing the biological nature of the disorder relieves feelings of guilt over being responsible for the illness. The incorrect responses are neither wholly accurate nor reassuring; they fall short of being reassuring and place the burden of having faulty genes on the shoulders of the parents.

The nurse wants to assess for disturbances in circadian rhythms in a patient admitted for depression. Which question best implements this assessment? A. "Do you ever see or hear things that others do not?" B. "Do you have problems with short-term memory?" C. "What are your worst and best times of day?" D. "How would you describe your thinking?"

C. "What are your worst and best times of day?" Rationale: Mood changes throughout the day are related to circadian rhythms. Questions about sleep pattern would also be relevant to circadian rhythms. The question about seeing or hearing things is relevant to the assessment for illusions and hallucinations. The question about thinking is relevant to the assessment of thought processes. The other question is relevant to assessment of memory.

A nurse must assess several new patients at a community mental health center. Conclusions concerning current functioning should be made on the basis of: A. The degree of conformity of the individual to society's norms B. The degree to which an individual is logical and rational C. A continuum from mentally healthy to unhealthy D. The rate of intellectual and emotional growth

C. A continuum from mentally healthy to unhealthy Rationale: Because mental health and mental illness are relative concepts, assessment of functioning is made by using a continuum. Mental health is not based on conformity; some mentally healthy individuals do not conform to society's norms. Most individuals occasionally display illogical or irrational thinking. The rate of intellectual and emotional growth is not the most useful criterion to assess mental health or mental illness.

A patient is hospitalized for a reaction to a psychotropic medication and then is closely monitored for 24 hours. During a predischarge visit, the case manager learns the patient received a notice of eviction on the day of admission. The most appropriate intervention for the case manager is to: A. Cancel the patient's discharge from the hospital B. Contact the landlord who evicted the patient to further discuss the situation C. Arrange a temporary place for the patient to stay until new housing can be arranged D. Document that the adverse medication reaction was feigned because the patient had nowhere to live

C. Arrange a temporary place for the patient to stay until new housing can be arranged Rationale: The case manager should intervene by arranging temporary shelter for the patient until suitable housing can be found. This is part of the coordination and delivery of services that falls under the case manager role. The other options are not viable alternatives.

A nurse explains multiaxial diagnoses to a psychiatric technician. Which information is accurate? A. It is a template for treatment planning B. Nursing and medical diagnoses are included C. Assessments of several aspects of functioning are included D. It incorporates the framework of a specific biopsychosocial theory

C. Assessments of several aspects of functioning are included Rationale: The use of five axes requires an assessment beyond the diagnosis of a mental disorder and includes relevant medical conditions, psychosocial and environmental problems, and global assessment of functioning. The Diagnostic and Statistical Manual of Mental Disorders (fourth edition, text revision) (DSM-IV-TR) is not a template for treatment planning and does not use a specific biopsychosocial theory. Nursing diagnoses are not included in multiaxial diagnoses.

A patient comments, "I never know the right answer" and "My opinion is not important." Using Erikson's theory, which psychosocial crisis did the patient have difficulty resolving? A. Initiative versus Guilt B. Trust versus Mistrust C. Autonomy versus Shame and Doubt D. Generativity versus Self-Absorption

C. Autonomy versus Shame and Doubt Rationale: These statements show severe self-doubt, indicating that the crisis of gaining control over the environment is not being successfully met. Unsuccessful resolution of the crisis of Initiative versus Guilt results in feelings of guilt. Unsuccessful resolution of the crisis of Trust versus Mistrust results in poor interpersonal relationships and suspicion of others. Unsuccessful resolution of the crisis of Generativity versus Self-Absorption results in self-absorption that limits the ability to grow as a person.

A cognitive strategy a nurse could use to assist a very dependent patient would be to help the patient: A. Reveal dream content B. Take prescribed medications C. Examine thoughts about being independent D. Role model ways to ask for help from others

C. Examine thoughts about being independent Rationale: Cognitive theory suggests that one's thought processes are the basis of emotions and behavior. Changing faulty learning makes the development of new adaptive behaviors possible. Revealing dream content would be used in psychoanalytical therapy. Taking prescribed medications is an intervention associated with biological therapy. A dependent patient needs to develop independence.

A student nurse prepares to administer oral medications to a patient with major depression, but the patient refuses the medication. The student nurse should: A. Tell the patient, "I'll get an unsatisfactory grade if I don't give you the medication." B. Tell the patient, "Refusing your medication is not permitted. You are required to take it." C. Explore the patient's concerns about the medication, and report to the staff nurse D. Document the patient's refusal of the medication without further comment

C. Explore the patient's concerns about the medication, and report to the staff nurse Rationale: The patient has the right to refuse medication in most cases. The patient's reason for refusing should be ascertained, and the refusal should be reported to a unit nurse. Sometimes refusals are based on unpleasant side effects that can be ameliorated. Threats and manipulation are inappropriate. Medication refusal should be reported to permit appropriate intervention.

Operant conditioning will be used to encourage speech in a child who is nearly mute. Which technique would a nurse include in the treatment plan? A. Ignore the child for using silence B. Have the child observe others talking C. Give the child a small treat for speaking D. Teach the child relaxation techniques, then coax speech

C. Give the child a small treat for speaking Rationale: Operant conditioning involves giving positive reinforcement for a desired behavior. Treats are rewards to reinforce speech. Ignoring the child will not change the behavior. Having the child observe others describes modeling. Teaching relaxation techniques and then coaxing speech is an example of systematic desensitization.

The goal for a patient is to increase resiliency. Which outcome should a nurse add to the plan of care? Within 3 days, the patient will: A. Describe feelings associated with loss and stress B. Meet own needs without considering the rights of others C. Identify healthy coping behaviors in response to stressful events D. Allow others to assume responsibility for major areas of own life

C. Identify healthy coping behaviors in response to stressful events Rationale: The patient's ability to identify healthy coping behaviors indicates adaptive, healthy behavior and demonstrates an increased ability to recover from severe stress. Describing feelings associated with loss and stress does not move the patient toward adaptation. The remaining options are maladaptive behaviors.

A nurse administering psychotropic medications should be prepared to intervene when giving a drug that blocks the attachment of norepinephrine to alpha1 receptors because the patient may experience: A. Increased psychotic symptoms B. Severe appetite disturbance C. Orthostatic hypotension D. Hypertensive crisis

C. Orthostatic hypotension Rationale: Sympathetic-mediated vasoconstriction is essential for maintaining normal blood pressure in the upright position. Blockage of alpha1 receptors leads to vasodilation and orthostatic hypotension. Orthostatic hypotension may cause fainting and falls. Patients should be taught ways of minimizing this phenomenon.

The parent of a 4-year-old rewards and praises the child for helping a younger sibling, being polite, and using good manners. A nurse supports the use of praise because according to the Freudian theory, these qualities will likely be internalized and become part of the child's: A. Id B. Ego C. Superego D. Preconscious

C. Superego Rationale: In the Freudian theory, the superego contains the "thou shalts" or moral standards internalized from interactions with significant others. Praise fosters internalization of desirable behaviors. The id is the center of basic instinctual drives, and the ego is the mediator. The ego is the problem-solving and reality-testing portion of the personality that negotiates solutions with the outside world. The preconscious is a level of awareness from which material can be easily retrieved with conscious effort.

A person received an invitation to be in the wedding of a friend who lives across the country. The individual is afraid of flying. What type of therapy should the nurse recommend? A. Psychoanalysis B. Milieu therapy C. Systematic desensitization D. Short-term dynamic therapy

C. Systematic desensitization Rationale: Systematic desensitization is a type of therapy aimed at extinguishing a specific behavior, such as the fear of flying. Psychoanalysis and short-term dynamic therapy are aimed at uncovering conflicts. Milieu therapy involves environmental factors.

Which documentation of diagnosis would a nurse expect for a person with mental illness? A. I Rheumatoid arthritis II 100 III Posttraumatic stress disorder IV Arrested for shoplifting 2 months earlier V None B. I Mental retardation II Histrionic personality disorder III 75 IV Hypertension V Home destroyed by tornado last year C. I Schizophrenia, paranoid II Death of spouse last year III 40 IV None V Alcohol abuse D. I Generalized anxiety disorder II Avoidant personality disorder III Fibromyalgia IV Declared bankruptcy 6 months ago V 60

D. I Generalized anxiety disorder II Avoidant personality disorder III Fibromyalgia IV Declared bankruptcy 6 months ago V 60 Rationale: The option beginning with a diagnosis of generalized anxiety disorder places a clinical disorder on Axis I, a personality disorder on Axis II, a medical problem on Axis III, a psychosocial problem on Axis IV, and global assessment of functioning (GAF) on Axis V. The other options misplace and incorrectly order the clinical data.

A 26-month-old child displays negative behavior, refuses toilet training, and often shouts, "No!" when given direction. The nurse's counseling with the parent should be based on the premise that the child is engaged in which of Erikson's psychosocial crises? A. Trust versus Mistrust B. Initiative versus Guilt C. Industry versus Inferiority D. Autonomy versus Shame and Doubt

D. Autonomy versus Shame and Doubt Rationale: The crisis of Autonomy versus Shame and Doubt is related to the developmental task of gaining control of self and environment, as exemplified by toilet training. This psychosocial crisis occurs during the period of early childhood. Trust versus Mistrust is the crisis of the infant, Initiative versus Guilt is the crisis of the preschool and early school-aged child, and Industry versus Inferiority is the crisis of the 6- to 12-year-old child.

The spouse of a patient with schizophrenia says, "I don't understand why childhood experiences have anything to do with this disabling illness." Select the nurse's response that will best help the spouse understand this condition. A. "Psychological stress is actually at the root of most mental disorders." B. "We now know that all mental illnesses are the result of genetic factors." C. "It must be frustrating for you that your spouse is sick so much of the time." D. "Although this disorder more likely has a biological rather than psychological origin, the support and involvement of caregivers is very important."

D. "Although this disorder more likely has a biological rather than psychological origin, the support and involvement of caregivers is very important." Rationale: Many of the most prevalent and disabling mental disorders have been found to have strong biological influences. Helping the spouse understand the importance of his or her role as a caregiver is also important. Empathy is important but does not increase the spouse's level of knowledge about the cause of the patient's condition. Not all mental illnesses are the result of genetic factors. Psychologic stress is not at the root of most mental disorders.

A participant at a community education conference asks, "What is the most prevalent mental disorder in the United States?" Select the nurse's best response. A. "Why do you ask?" B. "Schizophrenia" C. "Affective disorders" D. "Anxiety disorders"

D. "Anxiety disorders" Rationale: The prevalence for schizophrenia is 1.1% per year. The prevalence of all affective disorders (e.g., depression, dysthymia, bipolar) is 9.5%. The prevalence of anxiety disorders is 13.3%.

A nurse assesses a newly admitted patient with depression. Which statement is an example of "attending"? A. "We all have stress in life. Being in a psychiatric hospital isn't the end of the world." B. "Tell me why you felt you had to be hospitalized to receive treatment for your depression." C. "You will feel better after we get some antidepressant medication started for you." D. "I'd like to sit with you a while so you may feel more comfortable talking with me."

D. "I'd like to sit with you a while so you may feel more comfortable talking with me." Rationale: Attending is a technique that demonstrates the nurse's commitment to the relationship and reduces feelings of isolation. This technique shows respect for the patient and demonstrates caring. Generalizations, probing, and false reassurances are nontherapeutic.

An informal group of patients discuss their perceptions of nursing care. Which comment best indicates a patient's perception that his or her nurse is caring? A. "My nurse always asks me which type of juice I want to help me swallow my medication." B. "My nurse explained my treatment plan to me and asked for my ideas about how to make it better." C. "My nurse told me that if I take all the medicines the doctor prescribes I will get discharged soon." D. "My nurse spends time listening to me talk about my problems. That helps me feel like I'm not alone."

D. "My nurse spends time listening to me talk about my problems. That helps me feel like I'm not alone." Rationale: Caring evidences empathic understanding, as well as competency. It helps change pain and suffering into a shared experience, creating a human connection that alleviates feelings of isolation. The remaining options give examples of statements that demonstrate advocacy or giving advice.

In the shift-change report, an off-going nurse criticizes a patient who wears heavy makeup. Which comment by the nurse who receives the report best demonstrates advocacy? A. "This is a psychiatric hospital. Craziness is what we are all about." B. "Let's all show acceptance of this patient by wearing lots of makeup too." C. "Your comments are inconsiderate and inappropriate. Keep the report objective." D. "Our patients need our help to learn behaviors that will help them get along in society."

D. "Our patients need our help to learn behaviors that will help them get along in society." Rationale: Accepting patients' needs for self-expression and seeking to teach skills that will contribute to their well-being demonstrate respect and are important parts of advocacy. The on-coming nurse needs to take action to ensure that others are not prejudiced against the patient. Humor can be appropriate within the privacy of a shift report but not at the expense of respect for patients. Judging the off-going nurse in a critical way will create conflict. Nurses must show compassion for each other.

The parent of an adolescent with schizophrenia asks a nurse, "My child's doctor ordered a positron-emission tomography (PET) scan. What is that?" Select the nurse's best reply. A. "PET uses a magnetic field and gamma waves to identify problems areas in the brain. Does your teenager have any metal implants?" B. "It's a special type of x-ray image that shows structures of the brain and whether a brain injury has ever occurred." C. "PET is a scan that passes an electrical current through the brain and shows brain wave activity. PET can help diagnose seizures." D. "PET is a special scan that shows blood flow and activity in the brain."

D. "PET is a special scan that shows blood flow and activity in the brain." Rationale: The parent is seeking information about PET scans. It is important to use terms the parent can understand. The correct option is the only reply that provides factual information relevant to PET scans. The incorrect responses describe magnetic resonance imaging (MRI), computed tomographic (CT) scans, and electroencephalography (EEG).

Cognitive therapy was provided for a patient who frequently said, "I'm stupid." Which statement by the patient indicates the therapy was effective? A. "I'm disappointed in my lack of ability." B. "I always fail when I try new things." C. "Things always go wrong for me." D. "Sometimes I do stupid things."

D. "Sometimes I do stupid things." Rationale: "I'm stupid" is an irrational thought. A more rational thought is, "Sometimes I do stupid things." The latter thinking promotes emotional self-control. The incorrect options reflect irrational thinking.

A critical care nurse asks a psychiatric nurse about the difference between a DSM-IV-TR diagnosis and a nursing diagnosis. Select the psychiatric nurse's best response. A. "No functional difference exists between the two diagnoses. Both serve to identify a human deviance." B. "The DSM-IV-TR diagnosis disregards culture, whereas the nursing diagnosis includes cultural variables." C. "The DSM-IV-TR diagnosis profiles present distress or disability, whereas a nursing diagnosis considers past and present responses to actual mental health problems." D. "The DSM-IV-TR diagnosis influences the medical treatment; the nursing diagnosis offers a framework to identify interventions for problems a patient has or may experience."

D. "The DSM-IV-TR diagnosis influences the medical treatment; the nursing diagnosis offers a framework to identify interventions for problems a patient has or may experience." Rationale: The medical diagnosis is concerned with the patient's disease state, causes, and cures, whereas the nursing diagnosis focuses on the patient's response to stress and possible caring interventions. Both the DSM-IV-TR and a nursing diagnosis consider culture. The DSM-IV-TR is multiaxial. Nursing diagnoses also consider potential problems.

A patient has symptoms of acute anxiety related to the death of a parent in an automobile accident 2 hours earlier. The nurse should prepare to teach the patient about a medication from which group? A. Tricyclic antidepressants B. Atypical antipsychotics C. Anticonvulsants D. Benzodiazepines

D. Benzodiazepines Rationale: Benzodiazepines provide anxiety relief. Tricyclic antidepressants are used to treat symptoms of depression. Anticonvulsants are used to treat bipolar disorder or seizures. Antipsychotic drugs are used to treat psychosis.

A nurse wants to find a description of diagnostic criteria for a person with schizophrenia. Which resource should the nurse consult? A. U.S. Department of Health and Human Services B. Journal of the American Psychiatric Association C. North American Nursing Diagnosis Association (NANDA) International D. DSM-IV-TR

D. DSM-IV-TR Rationale: The DSM-IV-TR identifies diagnostic criteria for psychiatric diagnoses. The other sources have useful information but are not the best resources for finding a description of the diagnostic criteria for a psychiatric disorder.

The spouse of a patient with schizophrenia asks, "Which neurotransmitters are more active when a person has schizophrenia?" The nurse should state, "The current thinking is that the thought disturbances are related to increased activity of: (Select all that apply.) A. GABA." B. Substance P." C. Histamine." D. Dopamine." E. Norepinephrine."

D. Dopamine." E. Norepinephrine." Rationale: Dopamine plays a role in the integration of thoughts and emotions, and excess dopamine is implicated in the thought disturbances of schizophrenia. Increased activity of norepinephrine also occurs. Substance P is most related to the pain experience. Histamine decrease is associated with depression. Increased GABA is associated with anxiety reduction.

The relapse of a patient with schizophrenia is related to medication noncompliance. The patient is hospitalized for 5 days, medication is restarted, and the patient's thoughts are now more organized. The patient's family members are upset and say, "It's too soon about the patient being scheduled for discharge. Hospitalization is needed for at least a month." The nurse should: A. Call the psychiatrist to come explain the discharge rationale. B. Explain that health insurance will not pay for a longer stay for the patient C. Call security to handle the disturbance and escort the family off the unit D. Explain that the patient will continue to improve if medication is taken regularly

D. Explain that the patient will continue to improve if medication is taken regularly Rationale: Patients no longer stay in the hospital until every vestige of a symptom disappears. The nurse must assume responsibility to advocate for the patient's right to the least restrictive setting as soon as the symptoms are under control and for the right of citizens to control health care costs. The health care provider will use the same rationale. Shifting blame will not change the discharge. Calling security is unnecessary. The nurse can handle this matter.

A nurse psychotherapist works with an anxious, dependent patient. The therapeutic strategy most consistent with the framework of psychoanalytic psychotherapy is: A. Emphasizing medication compliance B. Identifying the patient's strengths and assets C. Offering psychoeducational materials and groups D. Focusing on feelings developed by the patient toward the nurse

D. Focusing on feelings developed by the patient toward the nurse Rationale: Positive or negative feelings of the patient toward the nurse or therapist represent transference. Transference is a psychoanalytic concept that can be used to explore previously unresolved conflicts. Emphasizing medication compliance is more related to biological therapy. Identifying patient strengths and assets would be consistent with supportive psychotherapy. The use of psychoeducational materials is a common "homework" assignment used in cognitive therapy.

A patient's history shows intense and unstable relationships with others. The patient initially idealizes an individual and then devalues the person when the patient's needs are not met. Which aspect of mental health is a problem? A. Effectiveness in work B. Communication skills C. Productive activities D. Fulfilling relationships

D. Fulfilling relationships Rationale: The information provided centers on relationships with others, which are described as intense and unstable. The relationships of mentally healthy individuals are stable, satisfying, and socially integrated. Data are not present to describe work effectiveness, communication skills, or activities.

A nurse listens to a group of recent retirees. One says, "I volunteer with Meals on Wheels, coach teen sports, and do church visitation." Another laughs and says, "I'm too busy taking care of myself to volunteer. I don't have time to help others." These comments contrast which developmental tasks? A. Trust versus Mistrust B. Industry versus Inferiority C. Intimacy versus Isolation D. Generativity versus Self-Absorption

D. Generativity versus Self-Absorption Rationale: Both retirees are in middle adulthood, when the developmental crisis to be resolved is Generativity versus Self-Absorption. One exemplifies generativity; the other embodies self-absorption. The developmental crisis of Trust versus Mistrust would show a contrast between relating to others in a trusting fashion and being suspicious and lacking trust. Failure to negotiate the developmental crisis of Industry versus Inferiority would result in a sense of inferiority or difficulty learning and working as opposed to the ability to work competently. Behaviors that would be contrasted in the crisis of Intimacy versus Isolation would be emotional isolation and the ability to love and commit to oneself.

A single parent is experiencing feelings of inadequacy related to work and family since one teenaged child ran away several weeks ago. The parent seeks the help of a therapist specializing in cognitive therapy. The psychotherapist who uses cognitive therapy will treat the patient by: A. Discussing ego states B. Focusing on unconscious mental processes C. Negatively reinforcing an undesirable behavior D. Helping the patient identify and change faulty thinking

D. Helping the patient identify and change faulty thinking Rationale: Cognitive therapy emphasizes the importance of changing erroneous ways people think about themselves. Once faulty thinking changes, the individual's behavior changes. Focusing on unconscious mental processes is a psychoanalytic approach. Negatively reinforcing undesirable behaviors is behavior modification, and discussing ego states relates to transactional analysis.

A person tells a nurse, "I was the only survivor in a small plane crash, but three business associates died. I got anxious and depressed and saw a counselor three times a week for a month. We talked about my feelings related to being a survivor, and now I'm fine, back to my old self." Which type of therapy was used? A. Milieu therapy B. Psychoanalysis C. Behavior modification D. Interpersonal therapy

D. Interpersonal therapy Rationale: Interpersonal therapy returns the patient to the former level of functioning by helping the patient come to terms with the loss of friends and guilt over being a survivor. Milieu therapy refers to environmental therapy. Psychoanalysis calls for a long period of exploration of unconscious material. Behavior modification focuses on changing a behavior rather than helping the patient understand what is going on in his or her life.

A patient has anxiety, increased heart rate, and fear. The nurse would suspect the presence of a high concentration of which neurotransmitter? A. GABA B. Histamine C. Acetylcholine D. Norepinephrine

D. Norepinephrine Rationale: Norepinephrine is the neurotransmitter associated with sympathetic nervous system stimulation, preparing the individual for "fight or flight" response. GABA is a mediator of anxiety level. A high concentration of histamine is associated with an inflammatory response. A high concentration of acetylcholine is associated with parasympathetic nervous system stimulation.

An adult expresses the wish to be taken care of and often behaves in a helpless fashion. This adult has needs related to which of Freud's stages of psychosexual development? A. Latency B. Phallic C. Anal D. Oral

D. Oral Rationale: According to Freud, fixation at the oral stage sometimes produces dependent infantile behaviors in adults. Latency fixations often result in a difficulty identifying with others and developing social skills, resulting in a sense of inadequacy and inferiority. Phallic fixations result in having difficulty with authority figures and poor sexual identity. Anal fixation sometimes results in retentiveness, rigidity, messiness, destructiveness, and cruelty.

In the majority culture of the United States, which individual is at greatest risk to be incorrectly labeled mentally ill? A. Person who is usually pessimistic but strives to meet personal goals B. Wealthy person who gives $20 bills to needy individuals in the community C. Person with an optimistic viewpoint about life and getting his or her own needs met D. Person who attends a charismatic church and describes hearing God's voice

D. Person who attends a charismatic church and describes hearing God's voice Rationale: Hearing voices is generally associated with mental illness; however, in charismatic religious groups, hearing the voice of God or a prophet is a desirable event. In this situation, cultural norms vary, making it more difficult to make an accurate DSM-IV-TR diagnosis. The individuals described in the other options are less likely to be labeled as mentally ill.

A patient has disorganized thinking associated with schizophrenia. Neuroimaging would most likely show dysfunction in which part of the brain? A. Brainstem B. Cerebellum C. Temporal lobe D. Prefrontal cortex

D. Prefrontal cortex Rationale: The prefrontal cortex is responsible for intellectual functioning. The temporal lobe is responsible for the sensation of hearing. The cerebellum regulates skeletal muscle coordination and equilibrium. The brainstem regulates internal organs.

On the basis of current knowledge of neurotransmitter effects, a nurse anticipates that the treatment plan for a patient with memory difficulties may include medications designed to: A. Inhibit GABA production B. Increase dopamine sensitivity C. Decrease dopamine at receptor sites D. Prevent destruction of acetylcholine

D. Prevent destruction of acetylcholine Rationale: Increased acetylcholine plays a role in learning and memory. Preventing the destruction of acetylcholine by acetylcholinesterase results in higher levels of acetylcholine with the potential for improved memory. GABA is known to affect anxiety level rather than memory. Increased dopamine causes symptoms associated with schizophrenia or mania rather than improves memory. Decreasing dopamine at receptor sites is associated with Parkinson disease rather than improving memory.

A patient has taken many conventional antipsychotic drugs over years. The health care provider, who is concerned about early signs of tardive dyskinesia, prescribes risperidone (Risperdal). A nurse planning care for this patient understands that atypical antipsychotics: A. Are less costly B. Have higher potency C. Are more readily available D. Produce fewer motor side effects

D. Produce fewer motor side effects Rationale: Atypical antipsychotic drugs often exert their action on the limbic system rather than the basal ganglia. The limbic system is not involved in motor disturbances. Atypical antipsychotic medications are not more readily available. They are not considered to be of higher potency; rather, they have different modes of action. Atypical antipsychotic drugs tend to be more expensive.

A patient underwent psychotherapy weekly for 3 years. The therapist used free association, dream analysis, and facilitated transference to help the patient understand unconscious processes and foster personality changes. Which type of therapy was used? A. Short-term dynamic psychotherapy B. Transactional analysis C. Cognitive therapy D. Psychoanalysis

D. Psychoanalysis Rationale: The therapy described is traditional psychoanalysis. Short-term dynamic psychotherapy would last less than 1 year. Neither transactional analysis nor cognitive therapy makes use of the techniques described.

A patient begins therapy with a phenothiazine medication. What teaching should a nurse provide related to the drug's strong dopaminergic effect? A. Chew sugarless gum B. Increase dietary fiber C. Arise slowly from bed D. Report muscle stiffness

D. Report muscle stiffness Rationale: Phenothiazines block dopamine receptors in both the limbic system and basal ganglia. Dystonia is likely to occur early in the course of treatment and is often heralded by sensations of muscle stiffness. Early intervention with an antiparkinsonian medication can increase the patient's comfort and prevent dystonic reactions.

A nurse supports parental praise of a child who is behaving in a helpful way. When the individual behaves with politeness and helpfulness in adulthood, which feeling will most likely result? A. Guilt B. Anxiety C. Loneliness D. Self-esteem

D. Self-esteem Rationale: The individual will be living up to the ego ideal, which will result in positive feelings about self. The other options are incorrect; each represents a negative feeling.

Which belief by a nurse supports the highest degree of patient advocacy during a multidisciplinary patient care planning session? A. All mental illnesses are culturally determined B. Schizophrenia and bipolar disorder are cross-cultural disorders C. Symptoms of mental disorders are constant from culture to culture D. Some symptoms of mental disorders reflect a person's cultural patterns

D. Some symptoms of mental disorders reflect a person's cultural patterns Rationale: A nurse who understands that a patient's symptoms are influenced by culture will be able to advocate for the patient to a greater degree than a nurse who believes that culture is of little relevance. All mental illnesses are not culturally determined. Schizophrenia and bipolar disorder are cross-cultural disorders, but this understanding has little relevance to patient advocacy. Symptoms of mental disorders change from culture to culture.

The following patients are seen in the emergency department. The psychiatric unit has one bed available. Which patient should the admitting officer recommend for admission to the hospital? The patient who: A. Experiencing dry mouth and tremor related to side effects of haloperidol (Haldol). B. Experiencing anxiety and a sad mood after a separation from a spouse of 10 years. C. Who self-inflicted a superficial cut on the forearm after a family argument. D. Who is a single parent and hears voices saying, "Smother your infant."

D. Who is a single parent and hears voices saying, "Smother your infant." Rationale: Admission to the hospital would be justified by the risk of patient danger to self or others. The other patients have issues that can be handled with less restrictive alternatives than hospitalization.

A nurse can anticipate anticholinergic side effects will be likely when a patient is taking: A. lithium (Lithobid) B. buspirone (BuSpar) C. risperidone (Risperdal) D. fluphenazine (Prolixin)

D. fluphenazine (Prolixin) Rationale: Fluphenazine, a first-generation antipsychotic medication, exerts muscarinic blockade, resulting in dry mouth, blurred vision, constipation, and urinary retention. Lithium therapy is more often associated with fluid balance problems, including polydipsia, polyuria, and edema. Risperidone therapy is more often associated with movement disorders, orthostatic hypotension, and sedation. Buspirone is associated with anxiety reduction without major side effects.

A nurse cares for patients taking various medications, including buspirone (BuSpar), haloperidol (Haldol), trazodone (Desyrel), and phenelzine (Nardil). The nurse will order a special diet for the patient taking: A. buspirone B. haloperidol C. trazodone D. phenelzine

D. phenelzine Rationale: Patients taking phenelzine, an MAOI, must be on a tyramine-free diet to prevent hypertensive crisis.


Ensembles d'études connexes

Objective 10: Prepare an organized, developed analysis on a topic related to foundations of reading development, development of reading comprehension; reading assessment and instruction.

View Set

Ch 22 Bonding in Ionic Compounds

View Set